Está en la página 1de 277

Questions and Answers with Non Muslims Vol-8

Dialogs With Non Muslims on the Threads of www.topix.net (Vol-8)

1 of 277

Questions and Answers with Non Muslims Vol-8

Introduction:
In the Name of Allah, the Most Gracious, the Most Merciful All praise is to Allah Cherisher and Sustainer of Universe and His peace and blessings be on His Servant and Prophet Mohammad and the family and companions of the Prophet and those followed in their footsteps till the last day. Amen, Afterwards, This is the eighth volume on the Questions and Answers which I had with Non Muslims on the threads of www.topix.net. Seventh and last volume was published in the month of March 2011, and I had hoped that it would be my last. But as they say, old habits die hard, again the list of these answers went on increasing till It became of a sufficient size that I decided to upload it for the readers of SCRIBD. Reference to the earlier lists of Questions and Answers are given below: 1. Questions and Answers Vol-1 : http://www.scridb.com/ 6519912 (and repeated with change of format on 13103050) 2. Questions and Answers Vol-2 : http://www.scridb.com/12727738 (And repeated with change of format on 13262933) 3. Questions and Answers Vol-3 : http://www.scridb.com/15492127 4. Questions and Answers Vol-4 : http://www.scridb.com/18432168 5. Questions and Answers Vol-5 : http://www.scridb.com/22247840 6. Questions and Answers Vol-6 : http://www.scridb.com/27748086 7. Questions and Answers Vol-7 : http://www.scridb.com/51089589 The pattern is changed slightly with more space given for original question than what was done earlier, so the readers would have better idea of how lively were the discussions!! I would like to clarify once more that these answers are based on my limited knowledge and informations; these are not dogmatic viewpoint and not authentic Islamic position. If I have made any mistakes in these answers, I seek the forgiveness of Allah and I would request readers to inform me of the mistakes and I will be open to correction. If I am able to help solve doubts of some people have about Islam and Muslims, then I would consider my efforts successful. M.U. Qidwai Jubail, KSA, Jan 10, 2012.

2 of 277

Questions and Answers with Non Muslims Vol-8

Index:
Chapter-1 (War on Terror, 9/11 and related issues) Q-1 to Q-10 (Pages 6-15) Chapter- 2 (Atheism and related Topics), Q-11 to Q-15 (Pages 18-22) Chapter-3 (Christianity and related Topics), Q16 to Q 37 (Pages 24-44) Chapter-4 (Hinduism, Sikhism and Buddhism) Q 38 to Q 70 (Pages 46-78) Chapter 5 (Islam-1) Q 71 to Q 106 (Pages 80-111) Chapter 6 (Islam -2 ) Q107 to Q 140 (Pages 114-148) Chapter 7 (Islam -3) Q 141 to Q 172 (Pages 151-183) Chapter 8 (Islam-4 ) Q 173to Q 213 (Pages 186-226) Chapter 9 (Jews, Israel and Palestine) Q 214 to Q 235 (Pages 230-251) Chapter 10 (Misc. Subjects) Q 236 to Q 265 ( Pages 254-283)

3 of 277

Questions and Answers with Non Muslims Vol-8

Chapter-1 War on Terror, 9/11 and related Topics

4 of 277

Questions and Answers with Non Muslims Vol-8

5 of 277

Questions and Answers with Non Muslims Vol-8

Q#1

Fall of WTC

[QUOTE who="Chazmo"] 1. One comment... Maximum verbosity...

Thermite produces a tremendous amount of ultra violet radiation when burning... So muc so that it is not possible to look at the conflagration without protective eyewear... This 'flash' is very apparent when filming the burn... Even digitally... Where is this seen in the Towers?... Why is this also not seen in the second Tower?... Why is this also not apparent in WTC7?... Comments?...

2. Listen to "Demo Dave" Griffin and his crew talk about ground zero and evidence of pa caking.

"For it being two hundred and ten story buildings, the pile wasn't an enormous pile. We were expecting it to be - I think a lot of the guys were expecting it to be a lot more. I cut away a section of the wall - my gang cut into a section of the wall and we - we counted 1 floors compressed into 8 feet."

[/QUOTE] Ans. 01. Thermite and Ultra Violet Glow:

Do you think that People who planned and brought down WTC towers were Amateurs? Do you not know that Super Thermites are used routinely in the demolition Industry? Was any half witted explosion rigger was employed to put cutter charges in WTC towers? They were specialist and not any ordinary specialist but super specialist. It was a very deep rooted conspiracy, key persons were placed at key posts in many agencies, to look the other way during execution and investigation of whole episode.

Was it by chance that NIST missed to look for Super Thermites in the debris of WTC

And was it By chance or to aesthetic beauty of NYC or help ease steel shortage in USA that WTC steel was recycled so quickly? 02. Pan-caking of Floors:
6 of 277

Questions and Answers with Non Muslims Vol-8

Q#2

GITMO

Tale of a Dreaded Terrorists from GITMO PrisonBy Clive Smith

Note: The original was in English, it was translated and published into a Urdu Newspape it is being re-translated into English, if some one finds some change in words or expressions, it was un-intentional MUQ

In 2004 US Supreme Court ordered that inmates of GITMO prison should be allowed to consult lawyers, I visited the prison first time after this order. I was asked to represent on Mr. Ahmad Airachdi. When I went first time to meet him in 2005, the guards warned me to be very careful as he is a General of Osama Bin Laden.

But when I met him, I did not find anything abnormal with him and when I told him abou what soldiers had told me, he laughed and told that in reality he was a Chef and worked f 18 years in different hotels and restaurants of London. I did not believe it at first, but wh I went to each of these hotels and people confirmed about his work there, I got convinced Many people still remembered his wonderful cooking!! It was evident that he was innocent and was just implicated into being an Al Qaeda Dreaded Terrorist. He told me his story in these words.

The life in GITMO is such that a person is consistently shifted from one prison to anothe and from one cell to another continuously. I was also treated like that and after death of 3 inmates, I was shifted into camp 5, which consisted of concrete cells. They were closed from all directions, except for a glass in the back wall, from where faint light used to com There was a very small opening in one wall, like a matchstick, from where I used to see t sky and sometimes a bird flying

On the eve of 9/11, I was with some of my friends in London Caf, when we watched th news on TV. I was shocked and then every one started talking about American retaliation and how it would affect Afghan and Pakistan people. When attack started and I saw refugee running from there I decided to help these people.

I went to Pakistan and then went to Peshawar and then took a Taxi to drop me near the border of Afghanistan. On the way, I only saw trucks and busses, fill with people coming from Afghanistan to Pakistan.

I reached the border village of Pak Afghanistan and I was handed over to some man who would take me inside Afghanistan. I stayed there for 14 days and used to cook food for th refugee. One night I heard the sounds of Airplanes flying very low and there was a big Bang, I saw a bus being hit and on fire. All the people sitting in the bus were scattered he and there and no one survived.

After this side, I decided to come back to Pakistan, we were hiding from both Americans and Pakistani Army, because they were arresting people coming from Afghanistan and handing them over to American forces. Those who were from other countries were more
7 of 277

Questions and Answers with Non Muslims Vol-8

Q#3

GWB

[QUOTE who="Happy Lesbo"]<quoted text> Interesting post. I agree with you: Syria is next. Your post made a few things very clear to me but I don't think it was GWB's 'master-plan.' Let's just call it 'the powers that be' decided to gain control of the mid-east. By enforcing the 'no-fly zone' and using military force, Obama declared war on Libya (enforcing no-fly zones is an Act Of War). He declared war without the approval of Congress. That violates America's constitution. Unfortunately, 'we-the-people' of the United States had little voice in th decision. We cannot continue walking down this path. What is your solution?[/QUOTE]

Let me tell you that I am impressed with your posts and the clear insights you have about the real causes of the problem.

I chose the name of GWB the Great because he was the 'Outer face" which started this war on terror and who became "hyper active" after stupidity, inefficiency and negligence of entire US Govt. machinery for allowing something like 9/11 to happen (and that certainly raises the issue of conspiracy).

I am sure that Long term US goals and policies are not decided by the current dummy who is sitting in the White House..they are decided by Think tanks and specialized close group circles.

The Democrat and Republicans Masks which Presidents of US wear from time to time ar for public fun and they just slow down to increase the pace of these Long term US policies and goals decided by these think tanks.

The general public seems content that they have chosen a Brand New President who w wash away the sins of previous regime and lead US to a New life and closer to US dream (whatever it may be). That is why we see continuation of US policy irrespective of who is the US president. Osama has gone one step further than any president by retaining GWB the Great Secretary of Defense, I doubt if it is a unique case or has been repeated in US history. The US media and news channels and their financial institutions are in vice like grip of Jews and Zionists organizations.

I do not know when US people will get freedom from these vice gripsbut future historians (after 500 years of so) would marvel at the Total control which a very small minority group had over such a giant as US and lead it to whichever directions it sought f

8 of 277

Questions and Answers with Non Muslims Vol-8

Q#4

Nostrada mus

[QUOTE who="War is Over"]

MUQ, your English is great! I agree, many topics are not susceptible to quick or simple resolution. However, I beg to differ that such discussions have no purpose. Inevitably the lead to a sharpening of ideas. They are a crucible in which better or more thought out idea survive while others perish.

For example, you post quite often concerning the 911 tragedy. You probably have an opinion about it, about how it happened, who was responsible, and why it occurred. Do you know that the French sage and seer Nostradamus predicted that a prince from the Arabian peninsula would send two missiles to cause destruction in the "New City" which Nostradamus described as the greatest city in the New World, which had just been discovered at the time of his birth. A movie was even made about this prophecy by Nostradamus, "The Prophecies of Nostradamus," released in 1979. I recall clearly the film and wondering at the time why two missiles would be sent by the Arabian prince to destr New York, because everyone knew in 1979 that only one large hydrogen bomb would be necessary to destroy Manhattan island.

Recall that Osama bin Laden took clear and unequivocal credit for the 911 tragedy, explaining that even he was surprised that the two towers fell. He was not sad that they fe only surprised that the construction of the towers was so frail that the fire caused their collapse.

MUQ, the purpose of this forum is, among other things, to lead to the enlightenment of it posters and readers. I humbly suggest that you explore the prophecy of Nostradamus, maybe watch the film, "The Prophecies of Nostradamus," released in 1979. This might help you to understand that Osama bin Laden might indeed have been entirely responsibl for ordering the 911 tragedy to occur. .. [/QUOTE] Ans. It is really strange; I never thought that I would ever be called to comment on the predictions made by Nosterdamus!!

First of all we Muslims do not believe in such type of predictions and according to Islam, no one has knowledge of future except God Almighty. It is God Almighty who gives information about some future events to His prophets and they tell it to their followers. But even these prophesies are told in general terms and are never dogmatic in nature. As to my knowledge, this Nosterdamus was not a prophet of God, so his predictions and
9 of 277

Questions and Answers with Non Muslims Vol-8

Q#5

Pakistan

[QUOTE who=" "] Pakistan is not doing enough [/QUOTE] Ans.

What Pakistani President said was very correct that Most people in US Govt. and Congre do not know what they are talking when it comes to other countries and civilizations They do not know what are the ground situations are in India and they keep on repeating like a parrot "Pakistan is not doing enough". "Pakistan has not done enough".

I think what Pakistan has done, no other country can do. It has surrendered its sovereignty to US and NATO forces. They come with ease and bomb any place they want and no questions asked. What happened in Davis case, shows the extreme haughtiness os US people towards an independent country. A US secret service man, shoots two Pakistani nationals in broad day light.....and US "orders" Pakistani Court and Govt to "release" their man.

Can any Pakistani embassy man doing the same "crime" in US would be dealt in the sam way. This dual standard of Justice will be the failings of US and European people in the end. How can they distinguish on the basis of color of skin and nationality.

US and European people use a very shallow approach when dealing with other people an that is the main reason why there is so much mistrust and misunderstanding in their relations with other nations.

But when the issue any "orders" to Israel....it is "thrown back" on their faces with contem and they remain a moot spectator and can do nothing.

This is another anomaly in their relationships with other nations, they have a separate law to deal with Israel....much lenient than their own laws!!

10 of 277

Questions and Answers with Non Muslims Vol-8

Q#6

War on Terror

[QUOTE who="Hussein Obama Lied"]<quoted text> Obama promised several times in the debates he will capture Bin Laden ..he hasn't in fact he hasn't kept any of his campaign promises has he ?? We have deep deep corruption on both sides of the ailse.. who could ever vote for Barney Frwank ..?[/QUOTE] Why you pick up Obama alone? Before him GWB the Great started the Afghan war to capture and bring Osama for trial....he had eight years and he did not or could not do it. In fact there is a "war monger lobby" in USA which wants that USA should always be at war.

They derive double benefit from it, get big and fat defense contracts and test their weapon "live" on humans, which they could not do other wise. So accordingly USA is always at war for past 60 or so years (from the time Israel was created!!). Korean War Vietnam War. Kuwait War Afghan War. Iraq War, Libya War, Iran War, Syria war, Saudi War (planned for future) Why you pick on Obama? Why not pick on the War Monger Lobby of USA? How many trillions they have lost? and what they have gained?

US was never under threat by any country for past 250 years 9in WW-1 and WW-2, it wa USA which joined Britain and not because it was attacked first or was under threat)

11 of 277

Questions and Answers with Non Muslims Vol-8

Q#7

War on Terror

[QUOTE who="Bush"] 01. I support our soldiers 100%, they are deployed by orders from their commander in chief. I am also for eliminating terrorists attacks.

02. The question I have had for years is, is invading other countries the best way to destro terrorists and terrorism or are we just making "them" crazy and revengeful and cause more hatred and terror attacks. Also in my opinion if we loose 25 soldiers to kill 100 terrorists that is still not a win, because our soldiers lives are more valuable than thos terrorists. 03. It is time our military leaders wake up to the truth that you cannot fight an idea (hate for the west or democracy) with firepower. 04. We need to brain wash them the other way just like they are brainwashed by their religions. At their end is Radical Islam. 05. There was a time that we, the establish Christians went to central and south America and do acts of terrorism and decimated the Incas and the Mayans. And look where Spain is today. They are just a small nation. 06. Besides why rebuild Iraq and Afghanistan by destroying them in the first place. Lets rebuild America for Americans. [/QUOTE] Ans. 01. I am also for stopping terrorist activities in the world, but what if the soldiers themselves turn into terrorists?

Just following wrong orders of Superiors is no excuse in any International Court to comm excesses and inhuman acts. Remember Nuremberg Trials?

02. The Question you should ask is that why people hate your country and your soldiers? you solve this basic problem, may be you will not have to attack so many nations. The root cause is that your country interferes in the internal affair of every country.

03. You are right in saying that you cannot fight any Idea with firepower (that is why W or Terror was such a misnomer! A terror to remove other terrors?)

04. You are trying to brainwash them already for past 200 years. And for past 25 years or
12 of 277

Questions and Answers with Non Muslims Vol-8

Q#8

War on Terror

[QUOTE who="John_Schuylkill County_Pa"] 01. Just like George Bush bringing stability to Iraq Obama's not getting Qaddafi can neutralize Al-Qaeda. 02. Egyptian Zawahiri ,Al-qaeda #2 man, in his latest video wants to take on Qaddafi. They will diminish each other. 03. If gasoline prices don't come down soon President Trump will take over the oil fields both Iraq and Libya. 04. The 2 WTC towers became like 2 acetylene torches with the big gaping holes in the towers. WTC7 fell due to the damage from collapse of the towers.

05. You and Steven Jones cannot prove me wrong. I have seen the whole thing on live TV [/QUOTE] Ans.

01. You are right, Just what GWB the Great did to Iraq, his pupil Obama is trying to do to Libya.to destroy the country and then try to rebuild it from scratch. 02. This is golden policy, when you do not have any one else to blame , blame it on Al Qaeda.I do not know how Al Qaeda is surviving despite having no office and no fund for past 10 years and being banned in every country of the world.

03. Do you think where GWB the Great and Obama failed , President Trump will succeed This is only your wish and has nothing to do with reality. 04. Have you seen the size of WTC-towers and size of Jet liner? There was no Big gaping hole in WTC towers due to strike, in fact the damage was not visible from other three directions at all.

To assume that such strong steel structured tower would collapse on their bases in mere 9 minutes is denying almost all laws of physics and engineering and metallurgy.

And to simplify the fall of WTC-7 that it fell due to falling debris is another very non scientific fact. It was a 47 story building, if it can fall to ground just by falling debris then you should dismantle all those sky scrappers in NY city, they are very unsafe!!

05. The truth often comes out in the punch line and your truth came out in the last sentenc I saw it all on TV!! So whatever you see on TV is real? WWF wresting? Superman and batman and Spider Man flying in the sky? And all the stunts and magic you see on TV screen?

The 9/11 show was stage managed to look so convincing on TV screens and that is you
13 of 277

Questions and Answers with Non Muslims Vol-8

Q#9

War on Terror

[QUOTE who="RSOF"]

01. The US spent how much time, money and troops to protect Muslims from genocide a starvation in the 1990's? Kuwait, Bosnia, Somalia, Kosovo and what thanks did we g for it?

02. 9-11 well thanks but no thanks. And if you really cared about your "innocent civilians you would not use them as shields or spotters! Muslims brought this on themselves. 03. You tried for 10 yrs to get our attention with terrorist attacks, after attacks. WELL, after 9-11,you have had our attention and not in a good way.

04. Islamic terrorist could stop ALL of this right now if you would put down your guns a stop fighting. But that just isn't in your nature now isn't it! [/QUOTE] Ans. 01. We do not say that US has not done any good deeds, but they never did it to help Muslims and Islam in general. They always had their hidden agenda to propagate their own interests. 02. The thing which really antagonizes Muslims all over the world is US total and unconditional support to Israel and its every action. If you remember US has used MOST of its Vetoes in UN Security Council to shield Israel from any harm.

03. Most of these Terrorist activities whom you call it, have their direct and / or indirect link with the problem with Israel. Either US should break its ties with Israel or shou act as Neutral in this conflict and you will find that most of the terrorist problems would be solved.

04. But I am sure, US would act its hand, allow it to be dismembered and suffer every typ of humiliation. But it would never abandon Israel. And it has nothing to do with their LOVE for Jews or Judaism. It is because of the Vice Like Grip which Jewish Lobby and Zionists have on US media and US financial Institutions. 05. Most of US people do not even realize that they have taken for a sucker and they are fighting proxy wars for Israel and no one else. If US people realize this, they will become Free in the real sense and would take actions that are solely in the US interest.

06. Even the 9/11 has its root in the Israeli problems, but instead of solving the real issue, for past 10 years US is busy in hunting down Terrorists , with the result that violence and terrorism has spread to most parts of the world and has multiplied.

14 of 277

Questions and Answers with Non Muslims Vol-8

Q#10 Who did it [QUOTE who="Red Flag"]

MUQ, as a keen student of US Air Force tactics and awareness, it puzzled me briefly as t why the hijacked airliners on Sept. 11, 2001 were not apprehended by US air defense networks. It seems that the US air defense umbrella has always been designed around the assumption that the attack from Soviet/Russian or Chinese offensive combat aircraft would, of course, come from OUTSIDE the borders of the United States. The US Air Force never imagined a "foreign force" launching aircraft from airfields with the borders of the continental United States.

Imagine fifty Tupolev-95 Bear heavy strategic bomber aircraft taking off from twent or so American commercial airports, mixed in with standard, everyday commercial airline flights. Those combat bomber aircraft would be able to bomb America with impunity, if their bomb runs were closely coordinated temporally.

Note how the fourth airplane on 911, due to a delay in takeoff, was not coordinated with the other three aircraft attacks and thus was defeated in the air by the passengers and it crashed in Pennsylvania. Coordination of attack is critical.

So, MUQ, it appears that at the time of the 911 attacks, the US Air Force planners in charge of US air defense never conceived of an internal air attack. Now, of course, there are contingency plans in place. Great question and issue for this forum, the War Forum. [/QUOTE] Ans.

I do not suspect your sincerity and your bewilderment as to what really happened on 9/11

Every new terrorist attack has its own surprises and in almost every case the authorities a procedures are found lacking.

But if you really think deeply, you will find that 9/11 was really a special day for terrorist They could have done anything that day, if they wished!! There is no end of anomalies , and any thing by itself becomes a subject of discussion. I point out a few of these:

a. Idea of some one hijacking commercial planes and hitting a target with it, was not that far fledged. Right on the day of 9/11 an exercise was being conducted in that very area where Planes were to hit!! Was it a coincidence or a divertive tactic to fledge of any whistle blower?

15 of 277

Questions and Answers with Non Muslims Vol-8

Chapter-2 On Atheism and Atheists and related Subjects

16 of 277

Questions and Answers with Non Muslims Vol-8

Q# 11

Atheist

[QUOTE who=" "] Who is a Die-Hard Atheist? [/QUOTE] Ans. It is obvious that his name was picked up from his lead role in those "die hard" films. I doubt that the person who posted that post also knew him personally.

So far no one has come forward to define what a "die-hard Atheist " is....so I want to com forward and put a definition:

"A die hard Atheist according to me who will keep on denying God even on his death bed when some things of the next world appear to him....and he or she keeps on rejecting them"......such is a "Die Hard" Atheist!!

And a "Die Hard" believer is one who sticks to his faith, irrespective of whatever "proofs the opponents bring that there is no God! And we have samples of both of them in front of our eyes.

17 of 277

Questions and Answers with Non Muslims Vol-8

Q# 12

Creator

[QUOTE who="Dragnet"]

my answer will have to bit elaborate but that is because of the nature of the question. i wi type it anyway as it is up to you to read it or not.

you are presenting your arguments about god as if they are axiomatic or self evident truths. since you see that there is creation then you deduce that there must be a creato hence the nature of your understanding about god can also be called as apriori knowledge because you are essentially saying that only when there is god there can b creation. ..

in a priori knowledge about god there is a gulf between the creator and his creation while in a posteriori knowledge there is a seamless connection between the creator and his creation. [/QUOTE] Ans. Thanks for your Philosophical Approach on this subject. That is the problem with the Philosophers, if you ask a Child What is one plus one The Child will answer without thinking Two. And if you ask the same question to a Philosopher, he would spend may be one or two weeks on thinking about it and then come with this Masterpiece Answer.

I hear people saying one plus one equals two, I thought and I thought and it seems quite logical answer, but what is one and what is another one and how they can be added so after a lot of contemplation. I am inclined to say two, but I am still not sure!! The same I can say about the origin of God the way you explained it.

The thing is so clear that if you ask any child Can things come about without a Maker the answer will be straight No.

And this is so true in every thing we see about it, when we see a painting lying in street, n one has to convince us that it was made by a painter. It is our inner self which answers the question.

Paintings do not materialize from this air, they HAVE to be painted by a painter.so eve if we do not know him, he must exist. That is why the concept of Creator comes up to the mind of every one who sees this
18 of 277

Questions and Answers with Non Muslims Vol-8

Q# 13

God

[QUOTE who="Really"] I would change that slightly. The most common type of atheist says "I don't believe in god, but I can't prove it (nor should I)

Replace god with Easter Bunny, Santa Clause, whatever and it still works. All logically consistent.

What you are saying is that there is definitively no Easter Bunny, but there might be a go even though there is no evidence for either. Logically inconsistent. [/QUOTE] Ans. Atheists are on a very wrong track in their rejection of God. They try to convince themselves by giving false logic that since some thing cannot be Proven by Physical evidence, it must not exist.

The comparision of God with Easter bunny is all the most amusing, since it is comparin hallucinations and imaginations against some thing which is the most solid fact of this universe. And the biggest stumbling block in their path is this Universe!! Any one who sees the size of this Universe and the way it is being controlled, would not fail to realize that it is based on very fine design and very firm basic laws. How these laws came into being and how this whole Universe came into being, is a question which they cannot answer and they hide behind the excuse we do not know at present, but will find it out in near future.

That nothing comes into existence by itself, without a Creator is an established fact. Tha nothing worthwhile comes into existence by random actions is also an established fact.

Then how come this huge Universe came into existence, without a Creator, a Planner, and a Designer is a question which no Atheist can answer.

In Modern scientific world, it would only be a man without understanding who will claim that this Universe came into being by itself or by random selection. I think what drove to Atheists into rejecting God is the Models which religions like Judaism, Christianity and Hinduism etc are projecting.
19 of 277

Questions and Answers with Non Muslims Vol-8

Q#14 Animals [QUOTE who="Garamba"]<quoted text> 1)Animals do pray BECAUSE it is written? Show me just one camel waiting for judgement day... Above all, if animals like monkeys etc have no free will then it is obvious even for believers to understand that animals don't pray... 2)Worrying about myself? I really don't get that religious dogma about god punishing people for not obeying his laws. It sounds like a dictator who wants us to do something he forgot to fix in the first place (free will).

03. If that god really wanted beings to obey him and worship him why wasn't he satisfied with the angels who already do anything he commands them to do. Ever thought abo that? I guess not.

04. Apparently at the end of times the good will be allowed to heaven and the bad will perish in fire. According to the choice they made while living. What did the omniscient, omnipotent god need to know, what made him start that experiment, test [/QUOTE] Ans.

01. Animals are not waiting for the judgment day, because they have done no sin for whic they would be punished.

There 'crimes" could be only the use of excessive force against one each other and for tha there would be a very simple recompense. 02. What is the value of any law, if there was no punishment or reward for those who follow it and those who reject it. So what is this test for humans if no punishment or reward was associated with it? 03. And you are wrong if you assume that God gains anything when people worship or pray to Him. They do it for their own goods.

His Kingdom and His Majesty and His Dominion does not change by what these humans or all other creatures do.

So the acts of prayers and worships which humans do is for their own good and it keeps them on right path and saves them from the whispering of evil one and their own bas selves.

04. God does not need to know anything about the last day His knowledge is perfect. However it is to humans that all that record keeping and proofs are being prepared, s
20 of 277

Questions and Answers with Non Muslims Vol-8

Q#15 God

[QUOTE who="AB s"] 01. Is this the same creator who flooded the (FLAT) earth and destroyed all heathen earthlings except a few in a large boat? 02.Or is this the same creator that declared Israelites of jewdom as his chosen people?

03. Makes me wonder why a creator of such infinite power to split moons has an issue wi a woman being treated equally as a man, any idea? [/QUOTE] Ans. 01. Yep it is the same Creator, who punished humanity for disobeying His Prophet and Messenger Noah and saved only the people of Ark and let all others drown to be a permanent warning to all humans.

02. The chosen status for Jews was not a sort of blank check. It was a conditional title. So long as they continued to walk in the way of their Lord, they will be chosen above other nations. But if they turn away, that title would be given to other people.

Jews were warned again and again, and God gave them chance once after the other to repent. But when they rejected Jesus Christ, the last prophet to come from them and tried to kill them, they were suspended form that chosen status.

However when they rejected the Last and Final prophet, they were permanently removed from the Chosen People list and that title was given to Muslims (conditional again and no a blank check). Makkah was replaced as new direction of prayer for entire humanity in place of Jerusalem. 03. Creator has no problem with women. He created them and He knows what is good of them. He created men and women as complimentary to each other and not a direct competitor. He is really amused at the efforts of Modern Materialistic society to say that men and women are Equals and not Complimentary!!

It is amazing that these people think that they know More than their own Creator knew about them!!

21 of 277

Questions and Answers with Non Muslims Vol-8

Chapter-3 On Christianity, Christians and Relations with Muslims

22 of 277

Questions and Answers with Non Muslims Vol-8

Q# 16

Arab

[QUOTE who="Hugh Be"] Just wondering, is there such a thing as Arab Christians? [/QUOTE] Ans. Surely there are people who deserve to be called Arab Christians. Before the start of mission of our Prophet, Christianity was well established in Syria, Lebanon, Palestine, Egypt, Morocco, Tunisia, Abyssinia etc. The language of many of these nations was Arabic or some dialect of Aramaic / Arabic / Hebrew. When Arabs conquered these countries after the death of prophet, most people accepted Islam, but there still remained many people who did not accept Islam and continued their old religion. These are people who are Arab Christian and Arab Jews and they are following these religions since Many Generations. Copts of Egypt also belong to this group Now their language has become pure Arabic. But they have that ancient connection to Judaism and Christianity. They deserve to be called as Arab Christian and Arab Jews. Allah Knows Best.

23 of 277

Questions and Answers with Non Muslims Vol-8

Q# 17

Bible

[QUOTE who="SFN"] This is good discussion MUQ my friend. Thanks for your input on Islam.

1. I am defending the Bible and GOD said HE will never again destroy the world by wate and HE gave the sign of the Rainbow. Split The Moon is your belief, Not Jews or Christians. 2. I totally agree with you MUQ. 3. Yes it is because of the disobedience of Adam and Eve. .. 4. MUQ your sins and MY sins killed JESUS. ..? 5. Maybe it is not fit for Muslims to be on Saudi soil because of the way Muslim leaders mistreat your citizens? It is not only the Jews who need forgiveness. It is Christians, Muslims, and all humans. 6. NO, Arafat rejected Clinton and the 95% so the Palestinians got ZERO. 7. 37 Jesus replied: Love the Lord your God with all your heart and with all your soul and with all your mind. .. [/QUOTE] Ans. No problem , I also face the same problem with 4000 character limit! 01. You may defend Bible, but first you should know that Bible is not Spoken Word of Godso we just cannot attribute ANY statement in Bible to God.

God might have promised never to destroy humankind by flood, but the Rainbows did no start appearing from that day. God does not need any rainbow reminders because He never forgets His promise. It is an insult to God.

Splitting moon was a One time affair which was witnessed by those who were present, does nor matter, whether they were Jews, Christians, Muslims or Pagans. 02. No comment. 03. Adams and Eves sins:

You are just exaggerating that incident. It is the logic of St Paul and not of God. When y have free will, you might use it wrongly also. Adams and Eves were deceived by Satan an then they repented and God forgave them.
24 of 277

Questions and Answers with Non Muslims Vol-8

Q# 18

Bible

[QUOTE who="JAN"]

She made many great points, you come back with translations of the Bible as if that leads to corruption. For one translations are not corruption, secondly the same argument ca be made of the Quran you are aware I assume they have different translations of the Quran as well? Ancient scribes compared to modern prove the Bible verses the same, sorry the corruption claim is debunked by hundreds of thousand of artifacts. [/QUOTE] Ans. I am not talking about different translations, I am talking about difference in the content.

We also have different translations of Quran in many languages, they use different words to convey the meaning of same Arabic words, they are different translation, not different versions of Quran. In case of any doubt, people can go back to original Arabic.

But the case of Bible is totally different. The different versions vary in content also. I told you RC Bible has 73 books and Protestant Bible has 66 books, are they different translations?

Many verses from KJV have been thrown out from RSV are they different translations

Every version of Bible differs from another version in content, the original manuscripts d not tally exactly with each other.

And most people do not KNOW anything about the original languages in which thos books are written.

They are being soothed by the statements issued from Church and confirmed by Popes an Archbishops that God took special care to protect Bible and not one word was lost or changed. And people go back to sleep, thinking that Popes and Archbishops do no lie!! As a Christian Priest, you should know the Inside Story, should you not?

And by the way, I do not know that this skeptic is male or female!! And I do not that abo you either!!

25 of 277

Questions and Answers with Non Muslims Vol-8

Q# 19

faith

[QUOTE who="Sweet 2360"] The bible says if you have the faith of size of a mustard seed, you can move a mountain, rip a tree from the ground and toss it into the ocean. It is a proof of your faith. Apparently you talk a good faith, but never put it to a test.

At least not with unbiased witnesses to record the event. Do you know what that would d for your religion? Do you care? I have asked Christians the same thing. Nobody in an religion can do anything with prayer. Because there is no god listening. [/QUOTE] Ans.

Yes Bible makes that statement and you should ask Biblical Scholars how they interpret i In my view it was a qualitative statement and not a quantitative one.

I will not say that you can achieve nothing with prayers, it depends how you pray and how strong is your faith.

So many things have happened because of prayers that we need a book of many volumes compile it. And you will realise the power of prayers, when you are yourself caught in a situation, where there is no physical help from any one and you are sure that no physical help can reach you from any direction. That is when your heart automatically turns towards your Creator and you start praying, without any one telling you or asking you.

26 of 277

Questions and Answers with Non Muslims Vol-8

Q# 20

Faith

[QUOTE who="Redoran"] So, you too believe in "afterlife". In this country we have the fanatical "Born Again Christians", who also believe in "afterlife" so long as they invoke the name "jesus Christ". I don't know if you know it, a Christian could have murdered /butchered 10,000 people, and so long as they profess belief, before they die, they will sit with their god and become one in the same.

Also, if a Christian has believed in the Christ all his life and is shot in the head, and dies instantly without invoking Christ, that so-called Christian will spend an eternity in th hellfire and suffer agony 24/7. This is a fact, I am not making this up. Comments? [/QUOTE] Ans.

Yes Christians (Most of them any way, who call themselves Christians) do believe in the Hereafter. You are right in depicting the beliefs of most of Christians that it is only faith that is needed for salvation.

If they just Believe that Jesus died for their sins, they do not have to worry what they do i this life.

It does not matter how many persons they killed or how many they cheated, on the last da Jesus will save all of them.

In Islam it is not the same, both your faith and actions shall be judged. I true belief in Isla must produce good actions. Otherwise you are a Hypocrite. In Quran there is always mention of Faith and Righteous deeds, one after the other. So Islamic approach is more balanced that what Christians believe in. PS:

What Christians say of only faith will save them is as per their own inventions and not as per teachings of Jesus. Who said Unless your righteousness exceed the righteousnes of Pharisees and SEDUCEES, you shall by no means enter the Kingdom of Heaven

27 of 277

Questions and Answers with Non Muslims Vol-8

Q# 21

Good People

[QUOTE who="Sophie"]

To be fair...there ARE good christians. Some better people than Muslims, and visa versa. We gotta give credit where credit is due here... and claiming everyone who associates themselves with an institution is bad is wrong. sorry dude. love your posts usually. You seem to know a lot of what you know. but sometimes these grandiose statements you mak against a whole group of people break down your credibility. [/QUOTE] Ans.

It would be very wrong on my part if I said that there are No Good Christians in the worl or that there are no good Jews in the world or that there are no Good Hindus, Buddhists or Sikhs in the world. There are good people in every community and there are bad people also in every community. But in these threads people try to criticise only the bad members of the society and the tones and wordings are such that seem as if we are criticizing every member of that society. You also scan the anti Islamic posts and they will give impression that all Muslims are dregs of the highest order and there are no good people amongst them.

Islam and Qurans approach is not like that, it always holds a balanced approach. After criticizing Jews for their rebellious behavior it ends with All (Jews) are not like that, amongst people of the book (I.e. Jews and Christians) there are those who stand during night rehearsing the word of God and enjoining what is Good and preventing what is evil. And at another place it says Amongst them (i.e. Jews) there are people standing firm on equity while most of them are perverted transgressors. Regarding Christians, Quran has special place it says that Closest in love and understanding to Muslims you will find those who say We are Christians, because amongst them you will find men devoted to learning and those who have renounced the world and they are not arrogant

And at another place, is praises the disciples of Jesus in this way We have placed in thei hearts love and mercy. (That is why we see Christians always very active (even to this date) in the fields of humanitarian aids and services to the poor).. So by no way, I am criticizing all Christians and telling that there are no good people amongst them. In fact Christians will become very close to Muslims, if they leave
28 of 277

Questions and Answers with Non Muslims Vol-8

Q# 22

Governme nt

[QUOTE who="Skeptic"] I don't think you understand our government. Although American citizens rights are given to us by God under our Declaration of Independence our Government is not run by any religious run religion. It is secular.

It recognizes the rights of all citizens regardless of their religious or non-religious beliefs. Its nuclear power is to protect and defend the citizens from the aggression of foreign powers. Religion is based on individual beliefs and is separate from the government. Christians do live their life following Jesus Christ. As do Jews follower their own religion and muslims follow their own religion in America. America is a pluralistic society made up of every race color and creed on this planet [/QUOTE] Ans. A. Christians Govt. being Secular: So you want to say that American Govt. in Not Christian. And none of the Govt. in Europe is Christian either.

Can you tell me in your 2000 years old History of Christianity ANY Govt. which was run on Christianity? B. History of Europeans Tryst with Christianity:

And please tell me WHY all these Mighty USA and European Govt. that consist of majority of Christian people have become Secular and not run according to principle of Christianity? I will tell you the answer. The last time European nations tried to live according to the principles of Christianity, that period is called Dark Ages in their history books. There was restriction on freedom, restriction on science, learning of knowledge and every thing needed for progress of human civilization.

The Popes ruled as tyrants in the name of Jesus and committed every types of mischief an oppression in the name of Jesus, Church and Christianity. European people had to fight for many centuries to get rid of that tyranny.
29 of 277

Questions and Answers with Non Muslims Vol-8

Q# 23

Hoax of 21 May 2011

[QUOTE who=" "] [/QUOTE] Ans. An "official Communiqu" from the Office of Son of Man: Dearly beloved,

I wanted to keep my "revisit plan" hidden from people, so that it should be a surprise to all....that is why I did not tell it to any one, but hide it in code words in "my book"....and I also told very clearly the following: "Matt. 24: 37 But as the days of Noah were, so shall also the coming of the Son of man be. 38 For as in the days that were before the flood they were eating and drinking, marrying and giving in marriage, until the day that Noah entered into the ark, 39 And knew not until the flood came, and took them all away; so shall also the coming of the Son of man be. "

Only ONE MAN found out (after many false starts and hit and trials) and, decoded m cipher, WHEN is the Coming of Son of? And THAT man is Sir Harold Camping, Esq. Reverend of Church of Lay Preachers.

And now THAT man has betrayed this Secret Knowledge to a bunch of his dedicated followers....so that the "return plan" of 'Son a Man" is no longer a Secret and Mystery any more!!

And therefore I have changed my plan and will no longer return on May 21, 2011. M return date will therefore will continue to be a secret (as originally planned) and would not be announced any more. All data s that exists in the existing "official book" stands obsolete and ineffective, with immediate effect. Yours Lordly, Signed: H.H.H.H.E. The Son of Man (Official Seal of Son of Man)
30 of 277

Questions and Answers with Non Muslims Vol-8

Q# 24

Jesus

[QUOTE who="Peter"]

01. Christ is called Mighty God, Wonderful Counselor, the Everlasting Father, Prince of Peace, the Lord our righteousness, the Creator, Redeemer, King of Kings and Lord o Lords and declared to be the Son of God by His resurrection from the dead. Muhammad said Jesus was only a prophet and not the Son of God.

02. Christ accurately predicted the destruction of the temple and Jerusalem in Matt 2 Muhammad said he was Allahs prophet and apostle, but he never prophesied anything which came to pass. 3. Christ preached the Kingdom of God confirmed with signs, wonders and miracles. Muhammad spread Islam with the sword without any confirming signs.

[/QUOTE] Ans. Let me again patch up and remove the hate content 1. Followers of Jesus in their ignorance and zeal bestowed on Jesus the Titles which are Prerogative of God and God alone, no created being can hold any of them.

The Highest Title for a Created being is to be called a Messenger of God, some one whom God chooses be to the Custodian of His message to human kind.

Jesus was such an honored man, so was Mohammad and se were so many prophets mentioned in Jewish and Christian scriptures (Gods peace and Mercy be on to all of them). Just being a prophet is the best and Higest title for any Created being. And you really insult some one when you bestow on him the Titles which are not his.

If you start calling a street beggar and pauper O Mighty Billionaire, O holders of keys o treasure, O who can give Millions to whom you wish you are either insulting him or pulling his legs. 03. All prophets of God make predictions and they all prove true. Jesus made many predictions and they proved true. Our prophet made many predictions (they are recorded in Quran) and I have written a monograph about it).

04. Jesus could only preach and promise and give Good news about the nearness of Kingdom of Heaven (because he lacked the manpower and resources to establish it)
31 of 277

Questions and Answers with Non Muslims Vol-8

Q# 25

Jesus

[QUOTE who="BMZ"] Please read my very important post, once again and let me have your silly comments, please: Hello, MUQ

I would like to add the following important points, which the Christians have missed form day one:

1. If you read Acts very carefully, you would know that the disciples of Jesus kept on praying in the Temple. Peter is on record for saying that.

2. Who did the disciples worship in the Temple? Of course, God Almighty, not Jesus 3. Where was Paul sent to for offering repentance? The Temple. 4. Paul, the self-loathing sinner, even did not worship Jesus. Neither did he consider him God. He just considered Jesus as a man and a mediator between God and men.

5. There was not a single gospel in the market or on the streets, when Paul was teaching. Even Paul did not have a gospel. There was no Mark, no Matthew, no Luke and no John.

6. Long after Jesus, Paul and all were gone, came in the weird gospel by John, who tried to write like a Jewish prophet and produced a Gnostic gospel, just to show divinity of Jesus, because the other three gospels simply looked like a children's story book. 7. All the problems arose after the appearance of John's silly and unsubstantiated gospel. 8. John was most likely not written by the real John, because that John was also illiterate or unschooled, just as Peter was, as confirmed by Acts.

Instead of finding Jesus' Way to the Father in Heaven, they started to find who Jesus was, etc., and ended up finding the abomination known as the triune God and the alleged son of God. Salaams BMZ [/QUOTE] Ans.

32 of 277

Questions and Answers with Non Muslims Vol-8

Q# 26

Jesus

[QUOTE who="Skeptic"] 01. It is very clear in scripture. Jesus explained it all through the New Testament. And God revealed Himself and the Holy Spirit at Jesus Baptism.

02. In John Jesus explains that He is the bread of life that came down from heaven an he is the giver of eternal life. 03. Jesus said the Father and I are one. They are two different persons but have the same thought pattern of each other. 04. As Jesus says in scripture I only do what I see the Father doing. And in John 17:5 Jesus say that before the world He and the father shared the same glory

5. And now, O Father glorify thou me with thine own self with glory which I had with the before the world was. So Jesus was with God in the beginning before the world was.

It is the actions and doing of God in Scripture that the Holy Fathers had to explain as God being a Triune God. "The Father, Son, and Holy Spirit" 06. The Father couldn't come down from heaven and live among us and be touched.

So God prepared a body for His Son Jesus so that He could live among us and be touched and the person would not die.

It is Jesus Himself that reveals that He was with God the Father in the beginning before th world was. 07. The point is you can not accept Jesus as prophet without accepting what he taught in the Gospels. And what Jesus taught in the Gospels is not in the Quran. [/QUOTE] Ans. 01. It is Very Clear in scripture: a. You have used a very strange phrase here that Jesuss divinity is very clear in the
33 of 277

Questions and Answers with Non Muslims Vol-8

Q# 27

John

[QUOTE who="Think"] 01. John was an eye witness and Jesus told John from the cross to care for Mary His mother. 02. How come the Quran never mentions who took care of Mary after Allah took Jesus back to heaven? . If Allah was a witness to it all why doesn't Allah tell what happened to Mary? 03. . How come Jesus was physically taken up to heaven and lives in heaven while Muhammad's human body is still lying in the grave? [/QUOTE] Ans.

01. It is now universally agreed amongst Biblical Scholars that writer of Gospel of Joh is not the same John who was one of the Chosen Disciple of Jesus.

So your whole picture of Jesus telling on the cross to John to take care of Mary vanish into thin air!!

02. Quran deals with important subjects and not with every subjects and trivial subjects like who take care of Mary after Jesus. Did Jesus took care of Mary when he was alive? No if you read the records of Gospels!!

When Mary and his brothers came to meet Jesus, and it was told to jesus, that your mothe and your brothers wait outside, he said: Who is my mother and who is my brothers? Then pointing to the multitudes , he said These are my mother and these are my brothers!! Mary had Other sons also beside Jesus. James was one of them , why would an Outsider like John will take care of mary when her own son was living!! 03. How was Jesus taken up into heaven with body? It was God who did that and He is Powerful enough to do this small feat. The same way He created Jesus Without Human Father!!

Yes Jesus lives in the heaven while our prophet lies buries in ground..just having partia superiority does not mean that Jesus is superior in ALL respects.

34 of 277

Questions and Answers with Non Muslims Vol-8

Q# 28

Paul

[QUOTE who="Matthew"] I don't disagree as much with the idea of Paul being a false prophet because some of his doctrine did cause followers to stumble. But the idea of the Comforter being Mohammed is wrong.

Jesus spoke, "I am going to send you what my Father has promised; but stay in the ci until you have been clothed with power from on high." Luke 24:19

The disciples where given the Holy Spirit "from on high" and they were able to spea in other languages. This allowed them to preach the good news to peoples of all nations. "Therefore go and make disciples of all nations, baptizing them in the name the Father and of the Son and of the Holy Spirit," Matthew 28:19 [/QUOTE] Ans. A. Apostleship of Paul: I thank you very much for even considering that Paul could be one of the false prophets. Most of Christians are so enchanted with his philosophy that even the idea of him being a false prophet is an Anathema to them.

If they could have pondered over his claim of being an Apostle of Jesus, there were plent of tell tale signs to prove the falsehood of his claim, some of which I give below:

a. He never met Jesus while Jesus was alive, so how he have been appointed an Apostle b Jesus? b. Even after Jesus left the earth, he was enemy of his followers and chosen disciples of Jesus and did everything to stop the mission from spreading. c. Then it was his own claim of self conversion which transformed him completely according to him. There is no confirmation that his claim was true.

d. Then after his conversion, instead of going and learning the teachings of Jesus from his chosen disciples, (which is a common process for a new convert) he immediately started his own preaching. e. He claimed direct access to Jesus and claimed Apostleship without any proof or confirmation from Jesus.

f. He was a strange apostle of Jesus, who never mentions any quotations from Jesus in all his arguments.

35 of 277

Questions and Answers with Non Muslims Vol-8

Q# 29

Rainbow

[QUOTE who="SFN"] Excellent Answers MUQ and does make sense. I have a little different take. 01. The sign of the Rainbow was given that the world will never be destroyed by water again. 02. All humans are made in the image and likeness of GOD. But all humans are sinners and are in need of salvation because of Adam and Eve disobeyed GOD. GOD does not break HIS promises to Abraham, Isaac, and Jacob. The Jews are still HIS chosen people. Mecca is the real holy place then why does the whole Arab and Persian world want Jerusalem? Bill Clinton and Israel gave Arafat and the Palestinians 95% of everything he wanted except for Jerusalem. Check points, better living conditions, etc. etc. 03. Husbands and Wives ( Men and Woman ) should love and care for one another the same way GOD loves and cares for one another. [/QUOTE] Ans. Thank you for your response.

It seems that you have a Hangover from your Biblical class, let me comment on some o them: 01. The sign of Rainbow: It is very unscientific you know! Rainbows are caused by the laws of refraction of light, when it encounters water drops. Do you mean to say that these laws did not exist before Noah time? Or it was only after Noah that water drops came into skies? God does not have to be Reminded by any signs, because He never forgets anything! That is why Quran is silent on any Sign of Rainbow. 02. Humans made in image of God: It is true, but it is not this Physical Image, which is shared also by monkeys and gorillas. It is the spiritual image. Knowledge, understanding, mercy, kindness, reasoning etc.
36 of 277

Questions and Answers with Non Muslims Vol-8

Q# 30

Return of Jesus

[QUOTE who="A voice"]

But He did give us signs before the coming of The Son of Man. The most important one being the coming of the Antichrist who will fool most of the people into believing that he is Christ. After 5 months of the rule of antichrist, then th Son of Man will return with the shout of angels and an event not seen since the beginning and will not be seen again. No one knows when GOD will tell Michael to kick satan out of Heaven so no one knows when the moment comes. JUST REMAIN ON DUTY. [/QUOTE] Ans.

What you have said is very correct. The return of Jesus Christ is linked with the Rise of Anti Christ. And after he has ruled for some time, Jesus would come down on eart to destroy the Anti Christ. And no one exactly knows the time when Anti Christ shall come. So all the believers should be on Guard. This is what Islamic teachings are also and we call Anti Christ by the Arabic name MASEEH DAJJAL, which mean false or Lying or Evil Christ, which is same as Anti Christ.

And I think that Most Christians are not even aware that Muslims do believe in second coming of jesus Christ and that he will come near the end of the world after DAJJAL has come to earth. You can see , how we Muslims feel, when every alternate year, some one comes out and fixes the date of Jesus return and End of the World!!

37 of 277

Questions and Answers with Non Muslims Vol-8

Q# 31

Return of Jesus

[QUOTE who="SS"] 01. Poor Muq. Doesnt even believe in a second coming of Christ.

02. But a man DID bring forth the Quran in the first place, did he not, Muq? Unless you d not believe that Quran was dictated to Muhammad word-for-word for him to record. Which is Muq?

You either DO believe that the Quran was dictated to Muhammad to record which includ the return of both Jesus and Muhammad or you do NOT believe that the Quran was dictated word-for-word from God's mouth to Muhammads ear. You either DO belie that Mohammed was revealed the Quran and then distributed it or you do NOT belie that. Which is it?

03. (by the way, excellent work on following Harold Camping. If more Christians follow such chicanery as closely, a lot less damage would be done to individual lives.) But which is it?

04. Either Muhammad was a liar, or he was telling the truth. I can't believe you espouse t Hadith, by the way. If there is anything anywhere that has done more damage to the reputation of Islam in the world, it has been the Hadith which are completely contradictory to almost every word that was ever printed in the Quran. [/QUOTE] Ans. 01. Who said Muslims do not believe in second coming of Jesus Christ.

The truth is Islam is the ONLY Non Christian faith of the face of earth which as its artic of faith believes in Jesus as a prophet of God and in his second coming. However Muslims reject this buffoonery which is played every few years by predicting that Jesus will return on such and such date.which is a favorite past time of many Christians!! 02. Are you not aware how Quran was revealed and composed?

Quran was revealed directly to prophet by the Archangel Gabriel, and he memorized the wordings. When Gabriel left the prophet would call one of his many scribes (becaus prophet himself could not read or write) to write down the latest revelation.

He would ensure that the writing was correct by asking the scribe to read it back to him, s that to ensure that he made no mistake in writing it. Such was the care and precaution by which whole Quran was preserved in written form
38 of 277

Questions and Answers with Non Muslims Vol-8

Q# 32

Sects

[QUOTE who="Old Pom"]

Manners and etiquette? That would eliminate a significant number of posters and all troll Nice but boring. Christianity has been a mess almost from the start, with the various groups each putting their own interpretation on the teachings. Concentrating on the peripheral whilst ignoring the main message. Constantine tried to bring these together, along with some other pagan groups, at the council of Nicaea in AD 325.

People being what they, those who should never lead got to the top and corruption spoiled it. We will have to wait till the end of time to find the truth. [/QUOTE] Ans. A. Manners and Etiquettes:

I know that being polite and following etiquette would eliminate many people and it migh even be a little boring. I do not mind a few entertainers on the thread either, but when majority becomes like that, then it becomes like a fish market and free for all.

Then no one can post a descent message and it gets overwhelmed by the garbage message B. Christianity and Sects: To say the truth, Jesus had a very short time period for his mission and he could not organize his followers into any tight knit society.

After his quick and sudden demise, what you said about sects being formed in Christianit and infighting between the sects is very true. The main reason for division of Christian into sects was because of St. Paul. He was the first man, who created his party within the Christian community.

You might be aware of the Jerusalem Council held between Paul and Chosen Disciples of Jesus.

39 of 277

Questions and Answers with Non Muslims Vol-8

Q# 33

Trinity

Who Invented the Trinity? (Part-2) by Aisha Brown A Formal Doctrine is Drawn Up

When controversy over the matter of the Trinity blew up in 318 between two church men from Alexandria - Arius, the deacon, and Alexander, his bishop - Emperor Constantine stepped into the fray.

Although Christian dogma was a complete mystery to him, he did realize that a unified church was necessary for a strong kingdom. When negotiation failed to settle the dispute, Constantine called for the first ecumenical council in Church history in order to settle the matter once and for all.

Six weeks after the 300 bishops first gathered at Nicea in 325, the doctrine of the Trinity was hammered out. The God of the Christians was now seen as having three essences, or natures, in the form of the Father, the Son, and the Holy Spirit. The Church Puts Its Foot Down The matter was far from settled, however, despite high hopes for such on the part of Constantine. Arias and the new bishop of Alexandria, a man named Athanasius, began arguing over the matter even as the Nicene Creed was being signed; "Aryanism" became catch-word from that time onward for anyone who did not hold to the doctrine of the Trinity. It wasn't until 451, at the Council of Chalcedon that, with the approval of the Pope, the Nicene/Constantinople Creed was set as authoritative.

Debate on the matter was no longer tolerated; to speak out against the Trinity was now considered blasphemy, and such earned stiff sentences that ranged from mutilation to death. Christians now turned on Christians, maiming and slaughtering thousands because of a difference of opinion. Debate Continues

40 of 277

Questions and Answers with Non Muslims Vol-8

Q# 34

Trinity

Who Invented the Trinity? (Part-1) by Aisha Brown The three monotheistic religions - Judaism, Christianity, and Islam - all purport to share one fundamental concept: belief in God as the Supreme Being, the Creator and Sustainer the Universe.

Known as tawhid in Islam, this concept of the Oneness of God was stressed by Moses in Biblical passage known as the "Shema" or the Jewish creed of faith: "Hear, O Israel: The Lord our God is one Lord." (Deuteronomy 6:4) It was repeated word-for-word approximately 1500 years later by Jesus when he said: "...The first of all the commandments is, Hear, O Israel; the Lord our God is one Lord." (Mark 12:29)

Muhammad came along approximately 600 years later, bringing the same message again "And your God is One God: There is no God but He, ..." (The Qur'an 2:163)

Christianity has digressed from the concept of the Oneness of God, however, into a vague and mysterious doctrine that was formulated during the fourth century. This doctrine, which continues to be a source of controversy both within and without the Christian religion, is known as the Doctrine of the Trinity.

Simply put, the Christian doctrine of the Trinity states that God is the union of three divin persons - the Father, the Son and the Holy Spirit - in one divine being.

If that concept, put in basic terms, sounds confusing, the flowery language in the actual text of the doctrine lends even more mystery to the matter:

"...we worship one God in Trinity, and Trinity in Unity... for there is one Person of the Father, another of the Son, another of the Holy Ghost is all one... they are not three gods but one God... the whole three persons are co-eternal and co-equal... he therefore that wi be save must thus think of the Trinity..."(excerpts from the Athanasian Creed)

41 of 277

Questions and Answers with Non Muslims Vol-8

Q# 35

Violence

1000 Years of Carnage & Barbarity in the name of Christ 10th Century Obscenities: Vile Princes of the Papacy:

"Popes maimed & were maimed, killed & were killed... Without question, these pontiffs constitute the most despicable body of leaders, clerical or lay, in history. They were, frankly, barbarians. Ancient Rome had nothing to rival them in rottenness." Peter d Rosa (Vicars of Christ, p48) John XII (955-964).:

Born from an incestuous relationship between Pope Sergio III and his 13-year-old daught Marozie. John, in turn, took his mother as his own mistress.

Pope at 18, he turned the Lateran into a brothel. He was accused by a synod of "sacrilege simony, perjury, murder, adultery and incest" and was temporarily deposed.

He took his revenge on opponents by hacking off limbs. He was murdered by an enraged husband who caught him having sex with his wife. 11th Century Horror: Church lords over ignorant squalor of millions 1095 - Pope Urban II calls upon the Franks to invade the more civilized Muslim world. Begins five centuries of warfare.

"Let those who have hitherto been robbers now become soldiers." Urban II addresses hi gangsters. 1009: Rivalry from Islam prompts eastern churches to break with idolatry. This 'iconoclasm' begins breach with idol-worshipping Catholic west. Centuries of bloodshed ensue. 1079: The Council of Rome: Persecution of Berengarius & his followers who cannot stomach the dogma of 'transmutation of bread & wine into Christ.' 12th Century Criminality: Christian Church ally of murderous kings & rogue princes

"Warrior Monks" - Muslim heads catapulted into the besieged city of Antioch by Christia Knights (Illumination from Les Histoires d'Outremer by William of Tyre 12th centur Bibliotheque Nationale, Paris).

42 of 277

Questions and Answers with Non Muslims Vol-8

Q# 37

Zionists

[QUOTE who="UM O"]

The sentiment of Christians towards the Zionists is already changing and new alliances w be formed. The time is ripe for a Christian-Muslim alliance so long as Muslims also embrace peace and stand against radical and militant Islam. True peace-loving Christians are standing up against Christian Zionists as are the peaceful Jews who condemn Zionism within their religion. It is time that Islam also purges their evil elements so that the three Abrahamic religions can once again be brothers and stand one.

It starts with the individual. I am a Christian who has befriended many Jews and Muslims and it is a very powerful friendship once you understand each others differences and realize that we all should have a common goal, and that is peace and brotherhood. So go and become friends with a Jew and a Christian, they are your brothers and to refuse to do so only means you are not for peace.

[/QUOTE] Ans. I thank you for your very beautiful post. We would love to see this Zionist- Christian nexus being broken as it bodes nothing but hatred and violence between the three Monotheistic religions of the world. Islam is all for co-existence and dialog and understanding each others position. I think Islam still has the most regard for jewish and Christian prophets than any other religion of the world. There are extremists and zealots in all three religions and we should see that they do not have a sway over their people.

So far we have seen that Western Media is solidly pro Jew and Pro Zionist and anti Islam and anti Muslims. And the main reason is that they are in control of Jews and Zionists. The common people have no idea when the views are interest alongside news and they think that they are same.

The study of Jewish Media and its activities will throw some light into it and hope people will realize as to what harm this jewish and Zionist lobby is causing to USA, Europe and the world at large.

43 of 277

Questions and Answers with Non Muslims Vol-8

Chapter-4 On Hindus, Sikhs and related Subjects

44 of 277

Questions and Answers with Non Muslims Vol-8

Q# 38

Buddhism

[QUOTE who="Whoops"]

I am convinced secular but you are right that the religion was just called religion by Hind themselves. Dharma. Toe millennia ago, even the India's Buddhist faith used the wor "Dharma" in their teachings. Again the basis of Buddhism was distinct and the close similarity was with Jainism. Many "Hindus" of that time practised both faiths in tandem and some took up Jainism or Buddhism exclusively. In India (pre 1947) Buddhism declined more sharply only in AD 1300. [/QUOTE] Ans.

The Name Buddhism, Jainism and Hinduism are all Human given name and none of th original prophets or originators called these people by these names.

That is why I said that name of religion that was from the very beginning is called SANATAN DHARMA i.e. the old, true or pure religion (in whatever sense you us the word SANATAN).

Lord Buddha or Lord Jain might be the prophets who came to restore the religion, since i was corrupted by its followers.they might not have brought any new religion. But much of this is hidden in mystery as we do not have original teachings of these old prophets. But is stands to reason and logic that when our Creator is one, how can He send these conflicting religions.

That is why Quran says that we should all agree to Basic Fundamentals which are commo in all religion. Decline of Buddhism and Jainism in India started with Idol worshippers Hindus going in offensive mode and destroying their places of worship and converting them back to their fold. Some of this is recorded in history books and most of this is hidden. Many Buddhists might have converted to Islam for that reason. He H

45 of 277

Questions and Answers with Non Muslims Vol-8

Q# 39

Caste

[QUOTE who="Dragon"]

the Islamic concern for the Hindu lower castes is fake and smacks of vendetta against the Hindus. you Muslims have a habit of fishing in troubled waters. just read the works o ambedkar and why he didn't become a Muslim. he found the Islamic faith full of hatred for the other, the non Muslim group. when Muslims are in minority they are always looking for fissures within the majority community. but once they become majority they simply are not bothered whether someone is lower caste or higher caste all are kafirs or infidels, period. Muslims just profess fake concern for the lower cast people so that they can exploit the disunity of the Hindus to their own advantage. firs you Muslims should stop drinking camel piss and stop practicing other retrograde practices before advising the Hindus. for Hindus, Islam is just an Arabic religion of intellectually deprived Arabs and nothing else. it's just a foreign religion of the Arabs [/QUOTE] Ans. Most of your post is filled with hatred and bias and rhetorics. The only two points of interests are Why Muslims did not do enough for Dalits during their ruling type and the second is Why Ambedkar did not accept Islam. I will address both of them shortly. A. Why Muslims did not do enough for Dalits when they ruled India:

There might be many reasons, my thinking is that they considered it as an Internal mann of Hindu religion and since they did not want to interfere in their religious affairs, th ignored it.

Another could be because they did not know Indian language and Hindu High caste peop would have warned them of religious bias for this caste system.

But there is no doubt that bulk of Hindus who converted to Islam were from Lower caste. And over the centuries Indian Muslims picked up some bad habits by living alongside Hindus and one of them was this virus of caste system.

It is my feeling that if Muslims had put a healing hand to the suffering of these Dalits, the situation in India would have been totally different.

But this was as opportunity lost and we can only seek forgiveness from our Dalit brothers for not doing enough. B. Why Ambedkar Did not accept Islam:
46 of 277

Questions and Answers with Non Muslims Vol-8

Q# 40

Cow

[QUOTE who="Dragon"]

01. Wasnt the cow piss drinking part in your post not an example of hate agenda? the difference between islam and Hinduism is that islam was political right from the beginning and that is why islam talked about islamic brotherhood while Hinduism ha become political only in recent days where they are talking about hindu brotherhood 02. but this is not to condone casteism. there are varying interpretations of casteism by Dalit scholars themselves. some Dalit scholars are saying that leaving Hinduism will be tantamount to accepting defeat. these Dalit scholars' ideology is similar to naxalites who are saying that feudalism in the form of money lenders and landlords should be wiped out and the land and its resources should be given back to the peasants.

03. In a similar way these Dalit scholars are saying that Brahmanism should be wiped out and Hinduism should be reclaimed back by the Dalits. these people are talking of Dalits becoming priests and defeating the Untouchability concept by remaining with Hinduism. 04. In fact it is Hindus whose practices got corrupted by islamic ideas. in rural areas the concept of women covering their faces while talking to males other than their husbands is a practice that was influenced by islam. even in islam, there are differences based on skin color, caste and other considerations. in Pakistan the punjabis treat muhajirs with contempt based on some minor color difference. how many fair complexioned north Indian muslims will marry with a dark complexioned south Indian muslim? Urdu speaking muslims have a contempt for vernacular speaking muslims and so on and so forth. [/QUOTE] Ans.

01. After hearing so many hate messages from you people on Islam and Muslim, I though it was only fair that you will ignore this Cow Urine Drinking slang, but you wen right after it!!

Hindus do have a strange relationship with cows, which goes beyond the reason. Arabs also owe so much to camel for surviving as a people but they never took to worshi of camels , I do not know how it started in India.because in Vedic time, Cows me was a delicacy to be served on special occasions.

And I know that still Cow Urine is used in MOST of the Ayurvedic Medicines produced by Famous Aurvedic Medicine making companies and that Cows Urine is also available in Packed and certified Bottles guaranteeing that it is 100 % cow urine!! ( if some one will try to adulterate it my mixing it with the urine of Buffalo!!)

47 of 277

Questions and Answers with Non Muslims Vol-8

Q# 41

Discrimin ation

[QUOTE who="CS"] Of course these names are not Given by God BUT THE FALSE GOD OF ISLAM. and that is the main cause of hatred towards Islam. No religion creates discrimination among people but Islam.

"As per God" ??? who told you this. Muhammad told this is in the quran. and what is the credibility of Muhammad, it is mentioned in the Quran that Muhammad is proph Is this your logic? Everybody is muslim??, who has given this name??? Your imaginary God ( created by muhammad ). Thats why i said, All those foolish questi raised by you had these consideration. that was the shit of wisdom which Muhamma transformed into you people. [/QUOTE] Ans. No religion creates discrimination among people but Islam.

I never thought that I would hear this from a Hindu!! A religion which is based on casteis and permanently divided human being into four castes with graded inequality calling Islam as the Only religion that discriminates between people!

Hinduism discriminates between people on things about which they have No Control. I is not in any one power to be born into a Brahmin family or a Dalit family. Islam at least discriminates based on things which are in human own control and not beyond control. Islam discrimination is issue based and discrimination in Hinduism is arbitrary. Regarding your comments about our prophet, I need not have to answer them.

By saying these you are in fact showing your own ignorance and proving by your own words that your name should indeed would have been MOORKH SINGH AND NOT CHATUR SINGH!! PS: And I think that it is time to stop answering your posts. I do not correspond with people who use such type of foul language against our prophet.

48 of 277

Questions and Answers with Non Muslims Vol-8

Q# 42

God

[QUOTE who="Shesha"]

1. a lingam is a male aspect/principle while a yoni is the female compliment. an omphalo is a lingam. note the phalo in omphalos; similiar to phallic. 2. it is believe that the abrahamic god was actually a lingam; which has it's roots in hinduism and shiva worship. the stone in the kaaba is the one which is associated with ishmael and hagar? 3. it is also believed that the story of jacob wrestling with the angel is due in part because he used a lingam to rest his head upon before meeting his brother esau.

4. what is even more enlightening is the fact he called the placed peniel(face of god); which is very similiar to the word pineal. the pineal gland; which is the 7th chakra of shakti, or 7th seal of meditation.

5. this is why the hindus wear the bindu to denote the third eye. this is why the egyptian gods are shown with the cobra/asp resting on their foreheads. this is why buddha is shown sometimes with his hair braided; which looks very similiar to a pine cone. [/QUOTE]

1. Thank you for telling me what is a Lingam and What is a Yoni but what THAT has do with the black stone placed in Kaaba? The purpose of that stone is only to indicat the point from which TAWAF (i.e. going round the Kaaba is to start and finish). If there was no such mark, since Kaaba is a rectangular house, people would get confused. Now they can count the number of rounds this is the simple and straight forward explanation. 2. And WHO says that Abrahams God was a Lingam? What rights you have to place YOUR OWN ideas on some one else? The stone in Kaaba is associated with Abraham and Ishmael. Hagar has or had no role in it from where you invent such stories?

Black stone at the time of our prophet was nothing but a single stone fixed in one corner o Kaaba..after many centuries the protective cover of silver was placed to save the stone from physical damage (no femininity of silver and no special shape like Yoni e have any ground for misrepresentation here)

3. And WHO believes that story of Jacob wrestling with god has any thing to do with Lingam or Yoni? He placed a rock at the place and what is so mysterious about it Why people go for looking for hidden meaning in every nook and corner
49 of 277

Questions and Answers with Non Muslims Vol-8

Q# 43

Gurus

[QUOTE who=" "] [/QUOTE] Posted without comment- MUQ Hindu guru convicted of molesting girls in US PTI | Mar 7, 2011,

WASHINGTON: An octogenarian (80+- MUQ) Hindu religious leader, head of a Hindu ashram, one of the largest temples in the US, has been convicted of molesting two young girls.

The girls, many of whose family members serve at the ashram 'Barsana Dham', a 200-acr Hindu ashram located in Austin. alleged that they were molested by Prakashanand Saraswati, who is known to his devotees as Shree Swamiji, when they grew up there in th mid-1990s. His conviction at the ashram,, which attracts a large number of Hindu Americans from across the country every year, has sent shock waves among the community members.

"We are very disappointed with the verdict. We are certain that Swamiji is innocent. This is not the end of the legal process," said Aman Agrawal, spokesman of Barsana Dham. The sentencing is scheduled for today. Convicted in all of the 20 counts, he faces a maximum penalty of 20 years on each of the counts.

In their charges, two women Shyama Rose, 30, and Vesla Tonnessen Kazimer, 27, allege that Prakashanand Saraswati tried to exploit them on numerous occasions over the course of several years, beginning when they were as young as 12. The two lived in the campus of the ashram along with their other family members. The charges by Rose and Kazimer against Prakashanand were first levied in April 2008 following which he was arrested. He was later released on a USD 1 million bond. A third woman Kate Tonnessen, 31, too made similar allegations recently. Barsana Dham was established by Prakashanand in 1990. http://timesofindia.indiatimes.com/world/indians-abroad/Hindu-guru-convicted-ofmolesting-girls-in-US/articleshow/7644917.cms PS:

50 of 277

Questions and Answers with Non Muslims Vol-8

Q# 44

Hinduism

[QUOTE who="DAKO"]

Dr. Robert A. Morey has been my spiritual guide and he in his intercourse gave me the following discourse by introducing Hinduism as he saw it. See for yourself if it make any sense. I do agree with anything in here.

Hinduism is one of the oldest pre-Christian pagan religions still viable in the world today. While we think of it as the faith of Mother India, it actually traces it origins to mysterious tribe of Europeans called the Aryans who invaded and conquered Northe India from 1500 BC to 500 BC. The light-skinned Brahmins of Northern India claim to be their physical and spiritual descendants.

The Aryans brought with them their sacred writings called the Vedas. They were originally fire worshippers and this is why they believed in cremation instead of burying their dead. They also invented the theory of soul-transmigration in which at death you do not go to heaven or to hell but you are reborn into another body on eart This next body could be animal, vegetable or human depending on whether you were good or bad. Your past behavior catches up with you in your present life due to the law of karma.

You could in your next reincarnation end up a clam, a carrot, a bush or a human bein The highest rebirth you could wish for was to be born as one of the white-skinned Brahmins who by virtue of their color were considered the "higher" class. The Ugly Reality of Racism

The inherent racism of historic Hinduism is thus blatant. You were judged by the col of your skin, not the content of your character, skills or talents. The darker your skin the lower your caste and rank in Hindu society. The whiter your skin, the higher you caste and rank. The Brahmins prided themselves on their white skin while despising the darker skinned untouchables who were often viewed and treated as sub-humans.

This explains why Hindu gurus are more than willing to travel to the West to convert rich white Europeans to Hinduism BUT never travel to black Africa to make convert The truth is, they don't want black people whose skin color is an indication of bad karma. As long as they can sucker rich white people into giving them money ("Mone is evil. So give it all to me.") why bother with darker skinned people? Do you agree with me, MUQ? 2. Muq, Let us now move to caste system.

The terrible caste system was invented in order to protect the white Brahmins from polluting their sacred whiteness with black blood. You had to marry and to labor in t caste into which you were born. The lines were clearly drawn and on one was allowe
51 of 277

Questions and Answers with Non Muslims Vol-8

Q# 45

Idol

[QUOTE who="Yuodhya"] 1. Then why do you not come India and teaches us as a Hindu ?

2. Hindus have many PANTHA so they can not believe in superiority of any book but AL of them believe in FREEEDOM of THOUGHTS and BRHMM . 3. They all love to God due to love and faith but not due to any 'fear ' or ' greed '. .. 4. Idol worship is not a bad thing . Scientifically it has been proved by

5. Please , do not explain the Kalki Avatar in your favor because you don't know that it's time and place is also described in these scriptures which was ignored cleanly by you . Th place is south India not Mecca . 6. Perhaps Mohammad misguided Arabians and others like so you should come in Hinduism and should wait for KALKI . [/QUOTE] Ans. 01. Propagation Work:

I do my job while in India and while I am working outside India. There are Muslims who are doing the work of propagating Islam to Non Muslims. 02. Having Different PANTHA:

And each having conflicting and diverging opinions on the Basic Faith issues is not a goo sign but a bad sign for any religion.

When two things are conflicting and contradicting each other, either both are wrong or on of the two is wrong. Both cannot be True. 03. Diverging Hindus:

So it is really a weakness of Hinduism and not its strength that Cow worship as well as Cow eating both be considered as truth (And we find both Cow worshipping and Cow eating Hindus). (Same way we have Ram worshipping and Ram hating Hindus. That also goes for Krishna and Hanuman and other Hindu gods.

And we have supporters of Vedas and Manu Smiriti Hindus and those who are opposed t
52 of 277

Questions and Answers with Non Muslims Vol-8

Q# 46

India

This Crazy, Crazy, Crazy India- MUQ (Things which cannot be explained, but we must keep quite so as not to hurt the feeling of others) A. The Protector of Hyderabad:

1. In Hyderabad there is a medium size lake near the border with Secunderabad, knows a Tank Bund if I remember correctly. A modern road runs between the two half of the lake and it is a very popular picnic spot in the evening time. 2. During 1980 s, some people thought to beautify the area and install some imposing object in the land.

3. What is a better thing than to install a statue of some god in the Secular India? So the decided to install Biggest statue of Lord Buddha in the middle of lake, to be a big attraction for the public. 4. So the statue was made and a big pedestal was installed in the middle of lake and the statue was to be shifted to that location on a barge and put on the pedestal. 5. But something went wrong, when the statue was being removed from the barge and it slipped and went to the bottom of lake!! 6. It remained there for one or two years and then an effort was made to retrieve the statue and this time contract was given to a big and reliable company.

7. It so happened that I was in Hyderabad on the day when the statue was taken out from the lake bottom. After a lot of cleaning and ceremonial washing, it was finally put on the pedestal.

8. When I went to Hyderabad, a few years back, I saw a sign Lord Buddha the Protector of Hyderabad!!

9. How strange, when the same Lord Buddha could not save himself from falling into the lake and was helpless to come out of the lake, unless pulled by human. How can h protect Hyderabad from any calamity? It would be the first object to fall down int the lake when some earthquake etc take place.

10. But we must keep our mouth shut and do not speak a word, so as not to hurt religious sentiments of some people. B. Standing and reclining Hanuman at Allahabad.

1. A few years back, I was returning from Bhopal to Allahabad and it was early morning when the train reached NAINI, next station was Allahabad, so every one woke up an started arranging for getting down.

2. I overheard two gentlemen talking by the window and one explaining to other Do you
53 of 277

Questions and Answers with Non Muslims Vol-8

Q# 47

Muslims

[QUOTE who="Yuodhya"]

Nietzche said this in spiritual sense but here is a war on net and in daily life both . Liber and tolerant culture [ Hinduism , Buddhism , Jainism Sikhism / culture of Terrorists [/QUOTE] Ans.

Sometimes the truth slips from the mouth and tongue of some people, irrespective of how much they try to hide it!!

By combining all the Tolerant religion (Like Hinduism Buddhism, Jainism, Sikhism, Judaism and Christianity) on one side and , Islam and Muslims on the other side, the have simplified our position. All the forces of falsehood are on one side and truth is on another side. This fight and struggle between truth and falsehood is not new, it is as old as when the first human prophet (Adam) set foot on this earth. And it shows that falsehood irrespective of its variety, color and shade always unites against the truth as we have seen in this bracketing affect And second point is that there is never compromise between truth and falsehood.they shall always remain separated from ach other as oil and water.

The falsehood will not rest till it has extinguished the light of truth..and it may seem th all the odds favour the forces of falsehood. And it is another strange thing that irrespective of whatever odds the forces of falsehood have against truth, it is the Truth which always wins. And the falsehood is the one which is defeated. There is a very beautiful Sanskrit phrase about it SATYA MEIVE JAYATE Truth always wins and Quran says the same thing:

QUL JAAL HAQ WA ZAHAQ AL BATIL, INN AL BATILA KAANA ZAHOOQA

i.e. Say The truth has come and falsehood has perished and falsehood is there to perish! I thank you for speaking the truth, though not intentionally!!

54 of 277

Questions and Answers with Non Muslims Vol-8

Q# 48

Muslim

[QUOTE who="ITRE"] KAABA A HINDU TEMPLE TAKEN OVER BY MUSLIMS KING VIKRAMADITYA INSCRIPTION ON A GOLD DISH HUNG INSIDE THE KAABA

In pure scientific study about the Historical Muhammad raises basic questions concerning the prophet's role as a moral paragon; the sources of Islamic law; and the God-given nature of the Koran. The scientists even doubt the existence of Muhamma Scientists say that the Koran is a not a product of Muhammad or even of Arabia, but collection of materials stitched together to meet the needs of a later age. There was n Islam until two or three hundred years after the traditional version at around 830CE. The Arab tribesmen who conquered in the seventh century vast territory were not Moslems, but were persons who worshiped idols and are scientists call them pagans.

Even though Prophet Muhammad was born in the full light of history the earliest document date about a century and a half after his death. Not only does this long laps of time cast doubt on their accuracy, but internal evidence strongly suggests the Arab sources were composed in the context of intense partisan quarrels over the prophet's life. The earliest sources like papyri, inscriptions, and coins on the prophet's life, contradict the standard biography. An inscription and a Greek account fix Muhammad's birth in 552, not 570. Muhammad's career took place not in Mecca but hundreds of kilometers to the north. Yehuda Nevo. The classical Arabic language wa developed not in today's Saudi Arabia but in the Levant.

Long before Islam came in to existence, Kaaba, in Mecca in Saudi Arabia was a pilgrima site. The word Kaaba might have come from the Tamil Language which originated around 1700BC. In Tamil Nadu Kabaalishwaran temple is Lord Shivas temple and Kabaali refers to Lord Shiva. The black stone at Kaaba is held sacred and holy in Islam and is called "Hajre Aswad" from the Sanskrit word Sanghey Ashweta or Non white stone. The Shiva Lingam is also called Sanghey Ashweta. So what is in Kaaba could be the same what Hindus worship. The pedestal Maqam-E-Ibrahim at the centr of the Kaaba is octagonal in shape. In Hinduism, the pedestal of Brahma the creator i also octagonal in shape. Muslim pilgrims visiting the Kaaba temple go around it seve times. In no other mosque does the circumambulation prevail. Hindus invariably circumambulate or Pradakshina, around their deities. This is yet another proof that th Kaaba shrine is a pre-Islamic. In Shiva temples Hindus always practice circumambulation or Pradakshina. Just as in Hinduism, the custom of circumambulation by muslim pilgrims around the entire Kaaba building seven times shows that the claim that in Islam they dont worship stones is not true.

Allah was one of the deities in Kaaba long before Islam was founded. It might come as a
55 of 277

Questions and Answers with Non Muslims Vol-8

Q# 49

Muslims

[QUOTE who="ITRV"] 01.Hindus inflicted the first major and decisive defeat on Muslims at the battle of Baharaich in 1033 showing how Jihad can be countered. Here is the story in brief: 02. This time, the invader was Mahumd Ghazni's son, Masud Ghazni, who following his father's footsteps invaded India with a large army.

03. Hindu kings asked him to go back. On his refusal, Masud's huge army was besieged b the even greater Hindu army. .. 04. After this decisive and ruthless Hindu victory, peace prevailed in the country for a century and a half; till the next wave of Muslim invasions started under the leadership of Mohammed Ghori. .. 05. This interlude of one hundred and fifty years from 1033 up to 1187 had made the Hindu forget the ..

06.Hindus were under a delusion that the Muslims were like any other invader who woul settle down in India and be absorbed into Hindu society, as had happened earlier with the Greeks, Huns, Kushans, [/QUOTE] Ans. History and lessons learned from History: History is a two way street. People use history for their own purpose and draw wrong conclusions from it. That is why Bandits and outlaws in one nation s history are regarded as Patriots and Freedom Fighters in another nations history.

Indian Independence movement is a ready proof of it. Read at how British used to describ those people and how Indians used to describe these people. So I will not use to argue much about which side I am on, I will just comment on the significance of this battle and show what it really meant. 01. Location of Bahraich? Look at the map of India and then look at Afghanistan and then look again at Bahraich. How a Muslim army of a few thousands people, crossed the whole length of India and reached at the center, without encountering any resistance on the way?

Was the whole land not ruled by any Hindu ruler? Why did they gave passage to Muslim
56 of 277

Questions and Answers with Non Muslims Vol-8

Q# 50

Muslims

[QUOTE who="AH"] muk,

may i remind u Indian muslim population is not less than 23 percent according to cia fact file that wud mean every fourth Indian is a muslim and India is the most populous muslim containing country, MAY I REMIND ALL still Indians official stand is for grant of FREEDOM OF PALESTINE, India had the most harmonious relationship with yaser Arafat and almost all arab country except Iran [/QUOTE] Ans.

Thank you for your info. I know general Indian public and most Hindus have no problem with Muslims and Arab countries and Palestinians and their support to their cause.

But there is another group in India which is under the clout of RSS and its affiliates. This group is very vocal against Muslims and always tries to plant hatred between Hindus and Muslims of India and this group wants to have very close ties with Israel and learn from them how to fight with Muslims. I was speaking about this group in my post and not about majority of Hindus in India.

And for your info India has a very close ties with Iran also. It is only after US pressure and the carrot of Permanent Security Council seat that India voted against Iran. Iran ha always helped India and I do not know, why should India be against Iran? The two countries have ties that date thousands of years. By following US advice, India will do itself more harm than gain benefits. Any one following US advice is doomed for failure!! For example Pakistan!!

57 of 277

Questions and Answers with Non Muslims Vol-8

Q# 51

Muslims

[QUOTE who="Dabang"] Everyone knows what happened in Ahmedabad was the reaction to the Godhra carnage by Muslims.

That Butt kept quite for all these years and now he is accusing Mr. Modi. We all kno the evidences and witnesses get corrupted in such a long time. Its not the time yet to judge any one.

This Muq, mentions Gujarat riots in his every post. I can understand his heart pains f his Muslim brothers.

But what about his Hindu brothers (if he is really a religious person), did he shed eve one tear when there was Hindu ethnic cleansing from Jammu and Kashmir. Every da there used to be mass murders of the whole Hindu families. .. What about Mumbai train bomb blasts, Jaipur temple firing, 26/11. .. [/QUOTE] Ans. 01. To shield and justify a Guilty person is worse than committing a crime yourself. If you are for justice, you should make him an example rather than protecting him.

02. In every Anti Muslim riots, there is involvement of Police, Admin and Politicians to small and large extent. But Anti Sikh riots of 1984 and Gujarat riots of 2002 are especial because of direct involvement of Govt. and the large amount of killings that took place.

03. It is duty of State Chief Minister to protect all citizens of his state and not to act as a partisan person . that is why Gujarat riots are being dragged on and on every level the state Govt and its chief Minister is at the receiving end. 04. It is a tragedy of India, that people are supporting Modi because he is an able administrator and has done a lot of development for Gujarat. India should reject the best administrator and best developer if his hands are so dipped in the blood of innocent people. Where is the rule of law and justice.

05. I do not know how much tears you have shed on hundreds of thousands of Muslims men, women and children who have died by the hands of Indian Military and police personnel in Jammu and Kashmir?. You shed tears for a few Hindus which are killed there and no one should mourn those hundreds of thousands.

Every day scores of Hindus are not killed in Jammu and Kashmir, but innocent Muslims
58 of 277

Questions and Answers with Non Muslims Vol-8

Q# 52

Muslims

[QUOTE who="Yuodhya"]

1-- What a stupid criteria to prove a book that is it revealed or not ? There is no such type of claim inside the Vedas therefore will you try to say that Vedas are not revealed books also . .. 2-- Total stupid claim . What does it means ? one person will claim and you will always rejected that . Example

3-- Was your prophet dumb also and for your kind information quran did not write b prophet himself and it's verses were collected and edited during the time of califa Osman . ..

[/QUOTE] Ans.

1. There are claims and then there are proofs. Any one can claim any thing, but he has to give proof.

In the case of Guru Granth Sahib, no one knows who wrote it, it is a collection of sayings of so many people..some even Muslim Sufisand then this amalgamation you pronounce as a revealed book without any proof whatsoever. I doubt if Guru Nana even looked at this Guru Granth Sahib.

In case of Vedas also, it is a popular belief amongst Hindus that it was a revealed book it lacks authenticity and accuracybut since it is a very ancient literature, people do not ask for too much proofs.

Many portions of Bible are based on revelation and many more are written by historian a folklore story tellers.there is hardly any scholar which believes that Bible from A t Z is written from revelation.

It is only Quran which is pure revelation and nothing else. Sayings of prophet are recorde separately in other sets of books called Hadith.

In Islam every thing is clear and organized, with complete biographies and dates of death of its narrators, so these can be cross checked for authenticity.

2. Prophet did not write Quran with his own hand, he did not know how to read or write. Whenever he received any new Revelation, he would read it to his companions and a the same time he, would call one of his scribes to write it down.

When it was written down, he would ask the scribe to read it to him, to verify that he wro it correctly. So Quran was being preserved on the hearts of people as well as in writte
59 of 277

Questions and Answers with Non Muslims Vol-8

Q# 53

Muslims

[QUOTE who="Yuodhya"]

1-- When any Mohammedan specially your type of traitor who survives on only religious hate for others , gives lecture on truth and nonviolence then one image only which comes in mind , wolf in sheep's skin .

2-- If Holy 'Shri Guru Granth Sahib ' is not a revealed book then how can any Muhammadan claim that holy Quran is a revealed book . If ' Shri Guru Granth Sahib is a collection of different saint's statement then what is holy Quran ? 03. Is it not a collection of ' Semitic religio - social traditions ' which was condimented with Arabian - Persian laws by Prophet Muhammad ? [/QUOTE] Ans.

01. You are accusing me of spreading hate, while I do not criticize any religion or people blindly.

My criticism is issue based and not personal. While people like you, use generalized term and even launch personal attacks. In this post you called me a Traitor, without any proofs whatsoever. 02. What point you said about Guru Granth sahib, is very important and valid for all so called scriptures available on the face of this earth.

At no where inside Guru Granth Sahib, it says that This is a book which was revealed to humans.this is a claim by people who consider it a Holy Book and revelation from God. While in the case of Quran, it is stated in more than 100 places that Quran is a book of revelation from God. So it is not Muslims which give it the title, but the book itself.

And Quran challenges whole human kind that If you are in doubt about (revealed nature of this book, then you should also bring a book like it.that Quranic challenge is open still after 1400 years and there are no takers.

03. And your comments about Quran that it is a collection of Semitic religionwhich was contaminated by Arabian-Persian Laws shows your own ignorance about Qura and Islam.

Do you know our prophet did not know how to read and write, and he had no human teacher.how could such a person combine all that knowledge in one book, which f 1400 years have been memorized by millions and stays as removed from other world
60 of 277

Questions and Answers with Non Muslims Vol-8

Q# 54

Muslims

[QUOTE who="DAKO "]

Congress did not plan or intend the poverty of muslims, or else there would not be successful muslims at all. Poor muslims had been told by their mullahs that it was th duty to have large families and madrassa education would suffice them to be worthy muslims. We know that Sanjoy did try to do them good during his mum's emergency Most well educated muslims in India do recognise the problem but are ineffective when lot like you are about. [/QUOTE] Ans. Congress party was an amalgamation of different people, many of them were secular minded and wanted overall development of India irrespective of religious or caste outlooks.

Side by side in Congress there has always been a hardliner lobby who is against Muslims right from its very inception.

During freedom Movement, Congress presented itself as party for all people of India Many Muslims joined congress party and helped in the freedom struggle.

But congress could not hide its Anti Muslim bias even during the freedom struggle and after India became independent. They tried to harm Muslims and deprive them of th chances to grow and develop. But they did it silently and in slow motion. It was not possible for them to openly stop Muslim development totally, especially when Muslims were not as uneducated as SC / ST people and they already had sufficient share in Govt. jobs pre- partition.

But congress Govts. Saw to it that Muslims representation in important Jobs kept declinin over the years. It was a sort of slow poison, and not easy to detect. But as I said, it is not possible for them to totally neglect such a large population, especially when they needed their votes during each election to stay in power. So it was congress which did real damage to Muslims and did it in slow process, while maintaining its secular faade. At present there is hardly any ideological difference between Congress and BJP. People could easily leave one party and join the other. Their fight is over the power and not over the ideology as such.

61 of 277

Questions and Answers with Non Muslims Vol-8

Q# 55

Muslims

[QUOTE who="Yaudheya"]

Was MAQUABOOL FIDA HUSSAIN who created offensive paintings of Hindu Godess Sarswati , Laxmi etc. and on which matter ' many hypocrites '[ some of them are still present on this forum ] made a large noise and declared Hindu organization as ' Fasci ' while they were protesting peacefully against these paintings and were demanding apology by M.F.Hussain . [/QUOTE] Ans. There are two answers to this post: First:

a. Islam is against drawing of any human picture or statue of any person, male or female, human or God. So this act of M.F. Hussain was against Islamic principles.

b. M.F. Hussain is no Islamic Scholar or any authority on Islam, and such film stars, painters, singers etc. are not representing Islam in India. Their names being Hindu or Muslim have no significance whatsoever.

c. These paintings were drawn in 1970 s and provoked no response whatsoever. It was on after 30 years that they were re-discovered and an agitation was launched, just because the name of artists looked like a Muslim.

d. That shows the Honesty of the media and persons like you, supporting this agitation an throwing your weight behind those who are protesting. Second: You should practice first before you preach to others:

a. All these stone statues of male and female god and goddesses show them in nude form The sexual parts of both males and females are clearly visible.

b. Ancient Hindu temples are filled with figures of male and female gods in intricate sexu positions, why do you not consider them offensive to your religion and bring them down. Why you proudly show them as part of your ancient culture.

c. Why you make fun of your gods like Ram, Krishna, Hanuman, Ganesha and every othe gods in numerous films and dramas and stage shows, There is hardly any day when such remarks are not passed by Hindus. Why your hatred and apathy comes to surface, when any Muslim does any of these?

d. There are many cartoon characters developed of Hindu gods like Hanuman and other gods, they make fun of these gods every day and you do not feel any thing. But if an
62 of 277

Questions and Answers with Non Muslims Vol-8

Q# 56

Muslims Tricks played by Pseudo Historians:

Another trick which these Pseudo Historians do, is to Quote from the writing of people th have Muslim sounding names whom no one knows.

Even a child can distinguish that no Muslim Selim or Khan could write those generaliz statements about Islamic rule and conquest in India. I do not even know if such persons do exist or not or some one is using these names to misguide some people. A. Ibn Batuta and his Memoirs:

I have read the full translation of Ibn Batutas memoirs about his visit to India. He travel to India during the times of Sultan Mohammad Tughlaq and stayed for many years in India. He was even appointed Qadhi of Delhi by the Sultan and served for many years. He has written in detail about what he saw in Delhi and India in general.

He was full of praise for the wisdom, power, and actions of Sultan , yet he has written tha Sultan is very cruel in exacting his punishment and he does not hesitate to shed huma blood.

He has given many examples, of his actions in such cases, but they were not based on any religious bias. He killed and punished Hindus as well as Muslims who went against h wishes and dictates.

The scene which our esteemed writer has produced might have something to do with people who rob the travelers in the highways. Every one knows that there were certa groups of people, whose job was to rob travellers and trading caravans going thru the areas. Every one including British have named such people and took actions to control their activities. If such people were caught they were given exemplary punishment, one such scene might match which our writer has shown. But is was not because of any Hindu Muslims bias. Which our biased historian has inserted in his report to pollute the minds of common people. For them the entire 1000 years period was nothing but favourite sports for Muslims to kill, plunge and destroy Hindus and their places of worship.

Such a biased view of History will only do damage to India and will never help in getting strong.
63 of 277

Questions and Answers with Non Muslims Vol-8

Q# 57

Muslims

[QUOTE who="ITRE"] u r no different than any other person. u use sober language but u support atrocities of islam without shame.

[/QUOTE]

[QUOTE who="Yaudheya"]

If my memory is not wrong , it was you who denied from the responsibility of Mohammadans in the destruction of Viharas , Monasteries , burning of Nalanda library and demanded fool proof . When i gave the quotes from ' Tabkat e Nasiri-- by Minhas us Siraj . O.K., At-least train of MUQ has come on desired track . [/QUOTE] Ans. To: Yaudheya and ITRE: I went thru your posts and I am putting a common rejoinder to both of you. My views and my posts are balanced and I try to put what is Islamic teachings on each issue.

You people get confused and try to force that actions of Muslims are per see the direction and teachings of Islam. So let me once again point that out to you, as to what are Islamic Teachings to deal with Non Muslims in general: Islamic teachings in dealing with Non Muslims: A. The Status of Non Muslims in General: 1. The first thing is that Islam believes that ALL of the worlds revealed religions had divine origins and were based on teachings of prophets sent by the same God.

64 of 277

Questions and Answers with Non Muslims Vol-8

Q# 58

Sayings

[QUOTE who="My Experience"]

Its like Dalai Lama said today in an article as he is visiting Canada right now...No religio should be condemned for having rotten people representing them...we all have the capability of being evil...its up to our RELIGIONS to make sure we prevent it......and NO RELIGION has yet prevented its evil! I love and adore this man!

I say Buddha all they way...I like the Hindu thinking...its more about who I am! God bles this man and his truth [/QUOTE] Ans. It is very nice for Dalai Lama to say such beautiful thing. And there is no dearth of people saying such beautiful things..they look very pleasing to ears and mind is filled with joy when we hear it. But let me ask you one question, Has Dalai Lama been able to remove evil from his followers and his religion.

All are Buddhists clean like snow? The only people whom we Think to be clean as sno would be the hermits who have renounced the world and live is one Saffron clothe inside caves. But once you join their fraternity, you will find out how clean they really are..

What I want to say is that, man has evil tendencies and good tendencies also. The aim of religions is to nurture good tendencies and suppress the evil tendencies. But irrespective of how much you try, you cannot eliminate one or the other. There shall always be good people and bad people in every society and in every religion. And that is what is expected of any system whose members are humans. It is only practical things and practical systems that should be followed. You have freedom to become Buddhist or Hindu or Sikh, it is your choice and I have nothing to say about it. But if you are under the false impression that they make angels out of humans, you are wrong. May God help you finding the correct way.

65 of 277

Questions and Answers with Non Muslims Vol-8

Q# 59

Sikhism

[QUOTE who="Someone"] 01. Well who is what level every reader can read it you and me are not the judge of our own posts. when you post a point of Azaan call to prayer .and I said told you my side .now you said you dot care then who cares if call to prayer is happening every where in the world . 02. If you don't care about others then people don't care about you 03. Sikhism is part of Sanatan dharam .like Islam is an abrahamic religion . 03. Now you deny cry or do what ever you want the truth is not going to change .Hindus can claim want ever they want they are not even close to sanatan dharam .there own vedaas say not to do idol worship .do they listen no they don't .

04. By the way most Muslims I have seen in India are semi pagans .they go to graveyards and pray is that not something forbidden in Islam . Dargah are all over in India and Pakistan any where you go you see grave with green cloth on it .

05. Karma is pretty simple makes sense what it means yes we believe. In rebirth accordin to karma .there is no heaven or hell .it right here you reap what you sowed . .. According to karma the punishment for bad deed is limited and again you will be given a chance .no unlimited burning in hell [/QUOTE] Ans.

01. Yes we cannot be judge of our own posts, that is why we leave it to the judgment of people. And by the way, if majority of people favour one view, it does not mean tha it becomes Truth.

Truth remains truth, even if whole world rejects it. When the whole world believed that Sun rotates around the earth, it was a wrong thing and a single man who said No Earth moves around the sun was speaking the truth, no matter how was he ridiculed and made fun of.

02. If people do not care about my religion, then why are more than 99 % of posts are fill in this thread about Islam.

03. So Sikhism is part of Sanatan Dharma , same way as Budhism and Jainism are parts o Sanatan Dharma?

66 of 277

Questions and Answers with Non Muslims Vol-8

Q# 60

Sikhism

[QUOTE who="Someone"] 01. What do you mean by la ilaha a il Allah . 02. Do you think when Sikhs agree with you that la ilaha il Allah .

03. When Sikhs say yes Allah is the creator which means god is creator .it does no represent the god of Muslims only .your Allah and my Allah or ram is not different . 04. Written in beginning of our holy book .which does not contain name of any prophet even gurunanak himself . .. . 05. . Any kind of rituals were denounced by gurudnanak . 06. I am sorry to say Sikhs have shed the rituals of Hinduism and islam .after gurunanak they created there own rituals . .. 07. What is the need of guru or prophet . .. 07A. Muslims will kill anyone who will not agree that Mohammad was prophet . [/QUOTE] Ans.

01. By LA ILAHA ILLA ALLAH, we mean that there is no deity worth worshipping except Allah. He is the sole Creator, Cherisher and Sustainer in this Universe and we should surrender ourselves whole heartedly to His Commands and Orders.

02. I do not know where Sikhs even understand this full meaning of this LA ILAHA ILL ALLAH. Islam is the religion of all prophets since the first human, so there is nothin strange if Vedas do not recommend Idol worship.

03. What I said earlier that Sikhs do not fully understand the meaning of LA ILAHA ILL ALLAH is clear from your this comment. The second part of the KALIMA is also from the same Allah and not invented by our prophet.

So belief in RASOOL ALLAH is also a part of same KALIMA and by rejecting the other half, you are rejecting the first half also.

67 of 277

Questions and Answers with Non Muslims Vol-8

Q# 61

Sikhism

[QUOTE who="Someone"] 01. a Sikh doesn't like a Muslim or hate a Muslim .he does not follow his own religion .because Sikhs hate only radicals . 02. The bridge between Sikhs and Muslims collapsed when wazir khan bricked alive the sons of Guru gobind Singh .

03. But Muslim stress on Mohammad rasool Allah , They have forgotten la illahi il Allah 04. I would like you discuss more on this point If you said circumcision is not in Quran then how did it creep into Islam . 05. First four guru did know about it sorry you are wrong .hyms of first four guru were collected and compiled by 5th guru into guru granth sahib . 05A. What Sikhs did during partition was completely political .i wish Tara Singh should have gone for separate country but he agreed to stay with India 06. Sufi shrines are attacked on Pakistan .. . I wish moderate Muslims should step

08. Unfortunately some scholars of Islam do not consider sheik farid as an important sain [/QUOTE] Ans.

I would like to comment on your post, which has some good points and has many based o your misunderstanding about Islam.

01. First you said that a Sikh does not hate a Muslim and does not love a Muslim, These are two separate religions. I have no problem with that, we can co exist with normal human relations between us. 02. Now you are taking too much of a Historical incident when you say that Bridge between Muslims and Sikhs were broken when Sons of Guru Gobind Singh were killed by a particular Muslim. I said many times and again that it was a political struggle between Mogul Empire and Sikh Leaders it was not a religious war. As for that incident being a permanent bridge broker , is like saying No bridges can be
68 of 277

Questions and Answers with Non Muslims Vol-8

Q# 62

Sikhism

[QUOTE who="Someone"] 01. So you don't even agree with others believe , 02. Mohammad going to heaven on mule doesn't sound funny. - Fever is from hell fire - There is tree in heaven under which travelers can travel for years - He met Jesus and others prophets in heaven - and met Allah got reduced the number of prayers - Splitting the moon (that is a big one )

03. I feel you don't have any knowledge about guru granth sahib what is written in it.

04. Let talk about the only true religion of Islam ,about the people devoted to gods w .why do they circumcise why every parent is making his boy suffer isn't that going against will of god . [/QUOTE] Ans. 01. No I do not object to what others believe in, but when they bring it to me, I certainly have a right to question them. I am not against any human body getting dematerialized, provided we have reliable witnesses who have seen it. That is why I asked you who the witnesses who saw his body getting dematerialized were.

02. Our prophet being called by God to visit as an Honored Guest is not at all funny. He was being visited by the Angel Gabriel almost daily, if he was also taken into heaven once, then what is wrong in it? (For your info, Christians also believe that Jesus Chri ascended to heavens with his human body). Now let me tackle a few funny Hadith which you come across: - Fever is from hell fire: MUQ: It is an allegorical statement, reminding people of the hell fire and people should always remember hell fire, whenever they feel hot or have fever - There is tree in heaven under which travelers can travel for years
69 of 277

Questions and Answers with Non Muslims Vol-8

Q# 63

Sikhism

[QUOTE who="Someone"]

01. If you say you are in race with Christianity good luck keep going .I don't have problem with that .

02. In his last moments guru Nanaks followers Muslim and Hindu started quarreling abo what to do with his body after death .guru said when I am dead please lay 2 flowers o each side of my body and which side flower is fresh they can take my body do what ever they want to do .. 03. The tomb of guru Nanak is in Pakistan still Muslims visit that place and offer respects .and the other one a Gurudwara is built . 04. If you want proof then you wont be able to provide s proof of Mohammad visit to heaven on a mule

05. If Muslim emperors did not have issue with Sikh religion then why did they brick alive the two young sons 7 and 5 years old in sirhind

06. If it was just political then why did massa rangad was disrespecting the golden temple and himself being a Muslim he was drinking alcohol inside golden temple 07. These kind of proofs are all written in our books and now people are translating and various websites are operating help yourself to google for proofs [/QUOTE] Ans. 01. Thank you for your best wishes, soon Islam will overtake Christianity as World # 1 religion, even as on now there are more practising Muslims in the world as there are practising Christians.

Many Atheists in Europe and Americas call themselves Christians in names and census ju to boost the figure of Christians in the world.

02. So you believe that Guru Nanaks body was turned into followers? It is a funny story and I have heard similar story about SANT KABIR also (India seems to be place for turning dead bodies into flowers!!) How many eyewitnesses you have, who his body turn into flowers? I think some people during the night took away his body and secretly buried or cremated it (depending upon who were they) and left some flowers to confuse people.

70 of 277

Questions and Answers with Non Muslims Vol-8

Q# 64

Sikhism

[QUOTE who="Someone"] 01. well when I mentioned about population I meant to say 4out of 5 don't believe Mohammad is last prophet .even some sect. of Islam for example ahmadiya sect 02. Muslims are in race with christianity good luck to them .

03. If you know little about life of guru Nanak you should try to know .it is strange to me that without even knowing about his life No body was found only two flowers we left Hindus burnt the flower and Muslims buried it .a tomb is built by Muslims in Pakistan . 04. Prophet are coming with a mission to world very true .so was the case of guru Nanak too he also used to say I have a Job to do. 05. If Sikhism was false religion then it might have been wiped out . 06. Who is a Muslim you answered it correct .how many are Muslims out of 1.2billion .please can you tell me . 07. Here is guru Nanak said about Muslims please follow the link http://www.sikhs.org/relig_qm.htm Just calling yourself a Muslim or Sikh is not enough 08. By the way I have said earlier too I very much like Sufi Muslims . [/QUOTE] Ans. 01. You were boasting that out of five persons in the world 4 do not believe in Islam, but when it comes to Sikhism you change your tone and say that it does not matter, if Sikhs are in minority, why this double standards?

Our prophet came to Arabia first, today Arabs are only 20 % of World Muslim population rest 80 % of Muslims are non Arabs.

What about Sikhs? How many of them are Non Indians and not related to Punjab? You d not have any idea about what a Global religion is.

02. Yes we are in race with Christianity, they had 600 years of head start, but Muslims a catching up with them very fast.
71 of 277

Questions and Answers with Non Muslims Vol-8

Q# 65

Sikhism

[QUOTE who="Someone"] 1 population of world is 6.9 billion muslims 1.2 billion.

2, .Buddhist who have read history of India will disagree with muslims .there is Sik temple in Tibet 3,prophet dont need to claim themselves as prophet people recognize them . 4,yes you are right there was no fear from Sikhism to Islams existence in India . 5,ok if you say that guru nanak copied from islam and Hinduism what about Mohammad why was pagans celebrating month of Ramadan . 6. MUQ please tell me what is a Muslim what does it mean ? [/QUOTE] Ans. Here are short answers to the questions you posed.

1. What is the Sikh population in the world today? Is it not strange for you to say that ONLY 1.2 Billions Muslims are in the world today. That means every fifth person in the world is a Muslim. What is that percentage about Sikhs?

And who says our prophet did not live like a True Muslim? In fact he is the Role Model f us and each one of us is trying to lead our life in his footsteps.

2. Most of life of Guru Nanak is unknown, people know very little about him. Most of tha is guess work and written much after he had died. I have a hunch that even his photo which is shown is his real photo. It seems the guess work of artists who drew his picture at a much later date!!

So what is great if there is a Sikh Temple in Tibet, will it make Sikhism a global religion? 03. A prophet is sent with a mission, if he does not declare his mission then what is the purpose of his coming? People do recognize prophets during his life time by his character and truthfulness and by the message he preaches. 04. It is wrong for you to say that Mughals wanted to finish Sikhism. Their fight was political and that too regarding sovereignty. 05. Guru Nanak and Sikhs borrowed from Islam and Hinduism without giving any credit from where they borrowed.

72 of 277

Questions and Answers with Non Muslims Vol-8

Q# 66

Sikhism

[QUOTE who="Someone"] 01. You have mentioned your job is to save as many from hell . 01A. Are you positive you have reserved your place in heaven ?

02. I heard is that only those make it to heaven who die in jihad .when are you planning t blow yourself i mean join the jihad [/QUOTE] Ans. 01. Not only me, it is every Muslims job to spread and preach the true message of Islam as many people he or she can reach up to.

01A. No Muslim is arrogant enough to claim that he or she has a reserved place in Heave (this is the usual traits of Jewish and Christian evangelists). We Muslims are humble people and we say that if we try our best to live our lives in accordance with the Laws and Guidance sent by our God, we have very high chance enter Heaven thru the Grace and Mercy of our God. 02. You have wrong concept about Jihad. In fact every Muslim is engaged in Jihad 24 hours a day thru out his or her life.

Jihad means to struggle and to strive and every Muslim man or woman is constantly on th struggle to decide as to which path to take, in his or her daily activity. To follow the dictates of Quran or Prophet or to follow his own desires or act against the principles of Islam. This is a continuous Jihad in the daily life of every Muslim male or female A special sort of Jihad is when a Muslim has to fight against those who have come to attack him or his country or pose danger to his faith.

Every Muslim should be ready to participate in this type of Jihad also If the situation arises. And I am also hopeful that if such a situation arises, Allah will give me coura to defend my faith against these enemies of Islam and Muslims, even at the cost of m life and my belongings.

73 of 277

Questions and Answers with Non Muslims Vol-8

Q# 67

Sikhism

[QUOTE who="Someone"] Muq 01. There is simple answer to your questions about why no judgment day and what our guru said .the answer is karma . .. 02. If your ancestors were tired of worshipping stones then why did they choose to kiss. Black stone . ..

03. Aurangzeb was offering gifts ,lands and leasing who were converting to Islam .people knew that the only way to get closer to the king is convert to Islam . ..

04. Indians are well wished of Bangladesh infact India played big role to free Bangladesh from Pakistan . ..

05. But you never commented on jail of Mir mannu I wonder why? Because you thes are dark pages of Islamic book which you don't want to open it up to the public . [/QUOTE] Ans. 01. What is this Karma which you Sikh people believe in?

From which Hindu book Sikhs have borrowed this Karma concept? Come on, Come o dont be shy? It seems that Sikhs did not shove off all things from Hinduism when they made their new religion. I would like you to explain what is this Karma theory and how does it answers my questions, which I posed to you.

02. The Black Stone was only one and to kiss it, one had to go to Makkah.while Hindu gods and goddesses and cows were to be found everywhere and in every streetthat why my ancestors got tired of worshipping so many god and drinking urines of so many cows.

They exchanged all these mini or micro gods with One Powerful Creator of Universe, Wh was More Powerful than all these millions of mini and micro gods combined! And you did not answer, why your Gurus turned away from Hinduism and concocted a new religion for themselves? 03. So now Aurangzeb was offering money and lands to those who converted to Islam.what I heard from Hindus was that he would kill every one who refused to
74 of 277

Questions and Answers with Non Muslims Vol-8

Q# 68

Sikhism

[QUOTE who="Someone"]

A. I am not shy at all to post from guru Granth sahib .well first of all there is no last day o Judgement day in Sikhism adding believe in that crap . ..

B. Yes we are told to keep (sword) to defend our self .if someone forcefully tries to conve us . This is my lord said .. C. The sword of islam and sword in Sikhism has two different reason . Islam used it spread the religion .and Sikhs used it to defend the religion . ..

D. Aurangzeb wanted to spread Islam in India he ordered Hindus to convert or pay jaziya .it will click in some people mind they killing infidels is ok .like the E. Sunni group killed Shia Muslims on the eid day .the bomber was thinking that [/QUOTE] Ans. I will touch the important point raised in your post; A. Sikhism and the Day of Judgment:

So according to Sikhism, there is no Day of Judgment and it all ends when you die. Then please answer these questions: 1. Why it matters that you become Sikh or do not become Sikh to do not follow any religion? 2. If death is the end of all , why people should do good deed and lead a poor life? Why should they not cheat and kill and lead a happy life on this earth?

3. And what will happen to these rulers (Like Aurangzeb in your view) who killed so man people and were not punished in this world? Will they and those who did not kill any one become same once they die? 4. Where are your gurus after they were martyred for the faith and what is difference between them and those who fought and killed them 4. I think it is the biggest lacunae in Sikh religion if it does not answer these questions.
75 of 277

Questions and Answers with Non Muslims Vol-8

Q# 69

Spirit

[QUOTE who=" "] [/QUOTE] Ans. Gas and Spirit: Have you seen the half cooked theory which this False Yogi has put forward about Gas and Spirit being Similar?

Such are methods of these people with small information and smaller Imaginations..their logic is not real logic but reverse logic which called KUTARK i Sanskrit language.

Similarities between Spirits and Gas are mentioned by the false yogi in his post, now let us see some of the Differences between the two:

a. The gas consist of the same atoms from which solids and liquids are made. Under certain conditions, these atoms become loose and move freely, they fill the space and sha in whatever containers they are kept.

b. But the gases are still material, they can be weighed and they can be separated and they can be compressed and they can be shifted from one container to another containers.

c. Many gasses have distinct colors and odors, so they can be distinguished from each oth by color or odor. d. If compressed many gases become liquid, so they become visible.

e. If cooled sufficiently then all gasses become liquid and can be seen by naked eyes. If w cool them further, they can even become solid and we can even hit one another with these gasses.

g. Gasses react with each other and have other chemical properties same as their liquid a solid counterparts have, they are just different state of the same matter.

Needless to say that NONE of these are applicable to Spirit, which is not made of materia

Just because we cannot see it or define its shape will make spirits equal to gas. Similarity in one aspect will not make it similar in all other aspects. Creator of Spirits and Spirits: We can extend the same logic to distinguish between Spirit and The Creator of Spirit!!

76 of 277

Questions and Answers with Non Muslims Vol-8

Q# 70

Tolerance

[QUOTE who="Yuodhya"]

Nietzche said this in spiritual sense but here is a war on net and in daily life both . Liber and tolerant culture [ Hinduism , Buddhism , Jainism Sikhism / culture of Terrorists [/QUOTE] Ans.

Sometimes the truth slips from the mouth and tongue of some people, irrespective of how much they try to hide it!! By combining all the Tolerant religion (Like Hinduism Buddhism, Jainism, Sikhism, Judaism and Christianity) on one side and , Islam and Muslims on the other side, they have simplified our position. All the forces of falsehood are on one side and truth is on another side. This fight and struggle between truth and falsehood is not new, it is as old as when the first human prophet (Adam) set foot on this earth. And it shows that falsehood irrespective of its variety, color and shade always unites against the truth as we have seen in this bracketing affect And second point is that there is never compromise between truth and falsehood.they shall always remain separated from ach other as oil and water.

The falsehood will not rest till it has extinguished the light of truth..and it may seem th all the odds favour the forces of falsehood. And it is another strange thing that irrespective of whatever odds the forces of falsehood have against truth, it is the Truth which always wins. And the falsehood is the one which defeated. There is a very beautiful Sanskrit phrase about it SATYA MEIVE JAYATE Truth always wins and Quran says the same thing:

QUL JAAL HAQ WA ZAHAQ AL BATIL, INN AL BATILA KAANA ZAHOOQA

i.e. Say The truth has come and falsehood has perished and falsehood is there to perish! I thank you for speaking the truth, though not intentionally!!

77 of 277

Questions and Answers with Non Muslims Vol-8

Chapter-5 On Islam-1

78 of 277

Questions and Answers with Non Muslims Vol-8

79 of 277

Questions and Answers with Non Muslims Vol-8

Q# 77

Quranic challenge

[QUOTE who="Mahmood"]

MUQ keeps bringing up the "Surah like it" challenge when I have debunked it several times over. Can anyone produce the anything like Shake pear or Picasso? Any autho good and bad, leaves behind his own figure print which is very difficult to imitate. [/QUOTE] Ans. The writer is making a false claim and giving a false color to the Quranic Challenge to bring a Surah like it.

I would like to take the issue with this gentleman (I use the word gentleman in very loose sense, the person with his mentality is no gentleman) A. Shakespeare: 1. Did Shakespeare made any claim that his writings are absolutely unique and no one could bring any thing like it? 2. Are Shakespeare Drama and books the last thing in English literature and above censor?

3. Have there been no Dramas or stories written after Shakespeare that are comparable to it?

4. How many English knowing people in the world can understand Shakespeare? He and his style of English have gone from the world. B. Picasso: 1. Who says that Picasso has been the greatest artist the world has ever seen or shall ever see? 2. Can we say that Picasso is superior to all the artists of past generations? 3. Have not better pieces of arts have come out after Picasso? 4. Has it become the end of Drawings and pictures after Picasso?

5. And did Picasso made the claim that he is the best artist the world has ever seen or no one can make any picture better than him. So all these claims are which people make about their heroes and not the heroes themselves.
80 of 277

Questions and Answers with Non Muslims Vol-8

Q# 78

Alcohol

[QUOTE who="Frijoles"]<quoted text>

1. I don't disagree with your position, as you basically admit it is based upon your opinion and values system. Your opinion and value system is that the Koran has something to offer. That is an honest opinion and application of your own values system - hard to see the illogic in that. 2. Where I live, the purchase of alcohol is outlawed for minors (under a certain age), and is illegal to drive your car while intoxicated. The POSSESSION of alcohol is legal. The focus is on the behavior.

3. There are all types of educational programs being promoted at any time encouraging moderation and control .. . Our country tried outlawing alcohol in the prohibition era and it was a huge failure.

4. Currently we are realizing the follies of a similar outlawing approach to pot (Marijuana and it is slowly becoming legalized as well. This approach seems to work and everyone seems to be happy. There is no discussion I know of banning alcohol. . [/QUOTE] Ans.

1. If we say and believe that Quran has something to offer. Then why not listen as to what Quran has to offer. How come it gives guidance to us , but will not guide you?

You try to ponder on the Guidance of Quran, think about it and then accept or reject if yo find it worthy or unworthy. But this pre-judgment on Quran and not listening to its message and rejecting it on face value is not what I would call a Civilized thing. 2. You have tried beautifully, to apologise for your society fight against Alcohol, and how to control its negative effects. You have given too much emphasis on drunken driving, as if that is the ONLY wrong effect of Alcohol.

If you analyze carefully, you will see that Alcohol causes many medical problems, and most of violent crimes and incest cases are done under the influence of Alcohol. You society has to spend Billions on medical costs and arresting criminals and getting the convicted

You said that your country did try banning Alcohol thru legislation and failed miserably so now they are meekly trying to reduce its ill effect (and failing miserably there too)
81 of 277

Questions and Answers with Non Muslims Vol-8

Q# 79

Alcohol

[QUOTE who="DAKO"]

1. Your Prophet banned alcoholism on the spur of the moment and not due to its social or personal harm to the faithful believers. He did so because some came to the mosque drunk and disturbed the prayers. You know that!?? MUQ!

2. I know you wanted to play another futile game whereby hadiths are not considered scriptures but then hadiths have full force in Sharia law. OH! MUQ! [/QUOTE] Ans.

1. Arabs were so fond of Alcohol, that it was considered impossible that they will ever be able to leave this habit. It is no less than a miracle the Islam was able to ban Alcoho and all type of intoxicants in so short time and so effectively. a. USA tried to impose prohibition using force of law and police in 1920 s and faled miserably. b. Abolition of Alcohol in mentioned in Indian constitution as one of the duties of Indian state. Our late Father of Nation was so against it, But no Indian Govt. was able to enforce it. They do that show of banning Alcohol from time to time, but they fail miserably. This evil is expanding and expanding.

C. During Hindu festivals like Holi, Diwali, Dussehra, Ganesha Chaturthi etc. consumption of Alcohol increase many folds. Govt. gets hundreds of Crores in dutie from sale of Alcohol and Indians drink this forgetting their father of Nation

d. Muslims remain the biggest group in world that do not drink this Alcohol. Their might be a few Muslims who drink here and there, but as a group they are away from this, and what did it? Quran and sayings of our prophet and nothing else, no force could d it.

2. But you are wrong, Alcohol was not banned at a spur of moment and it could never hav succeeded that way, it was done in stages and there are four such stages.

a. First stage: It was mentioned in a Verse that relate to Makkah period And from the fruits of the date-palm and the vine, you get out wholesome food and also intoxicants and in it is a sign for those who are wise (16:67).

Many Muslims realized that by mentioning toxicants separate from Wholesome food, means that Allah does not like Alcohol. So they stayed away, but many continued th habit. b. Second Stage: Then this verse was revealed during Madina They ask you concerning
82 of 277

Questions and Answers with Non Muslims Vol-8

Q# 80

Allah

[QUOTE who="Scholar"]

The Ilah - Allah thing is pretty similar to El. I mean it's not a secret that El means God, an it was used in ahead of the proper name of many deity, but in the same time El was t name of a supreme God. The same thing happened with Ilah. The two words are obviously linked.

Before the birth of Islam there were many Gods (Ilahs) that meccans worshiped, but also existed a Supreme God by the name Ilah with 3 daughters - that one, was your God Allah, that later Mohammed to underline this God over the others, used to call him A Ilah that subsequently became Allah, and declared that this God was the only one. [/QUOTE] Ans.

You are very correct in your assumption that Ilah is same was what was known in Hebrew as El, and which Jesus Pronounced as Eli, Eli or Elohi, Elohi.

All these words sound similar, only a change of dialect over centuries resulted in differen pronunciation.

That is why I said that Any Jewish prophet would recognize the world Allah of Arabs and would never confuse it with a moon god. Because it is same as there own El, Eli, Elo or Elohim!! Pagans of Makkah misused the word Ilah and took for themselves many more Ilahs than Allah And they made many daughters of Allah. All this was true.

But the word Allah was separate from Ilah, that is clear because in Quran, our prophet is told to ask these Makkah people Who Created this heavens and earth and who gives life and death (and then it answers) and they will say Allah.

If Allah was not known to Makkah people and was coined only by our prophet, they wou have objected. It was as interesting post.

83 of 277

Questions and Answers with Non Muslims Vol-8

Q# 81

Allah

[QUOTE who="Oracle"]

You mean "Allah". Real muslims don't use the word God. But I understand that you want the rest to know what you are talking about.

You're wrong when you say no power can oppose it. The devil tries to oppose it all th time. You call him Shaytan? More accurately said would be that "no power could ev oppose him and win." [/QUOTE] Ans. A. Allah Vs. God: In reality Allah in Arabic is the same deity which is called God in English, YHWH by Jews and Ishwar by Hindus. He has many names in different language, but they all point to the same Unique Creator of this Universe and every thing in it.

The Arabic word Allah has many unique qualities, which make it superior to other words used in different language. To discuss more on this subject is not really useful at th stage. But we can pursue it if you be interested B. Satan Opposing God? You are mistake, Satan has ZERO Power on this earth. He has no Power to thwart Gods decrees.

He has no say in creation and day to day running of the universe. How and when the sun will rise, where and how much rain fall will be there. When and how many people w be born and how and when they shall die. So he is in no way any FORCE compared with the God. The ONLY power which has been given to him by God is to whisper his evil thoughts into the human mind (and to counter that every human has been given the gift of conscience, which whispers the correct path).

Since God gave free will to humans so they choose between the true path (as shown by th Prophets of God and what told to them by their conscience) and the evil path (which is whispered to them by Satan and their own self). So the choice is for the man to take which ever path he chooses. Satan cannot force any human to act as he wishes this is the true picture of what are Satans power on this earth.

84 of 277

Questions and Answers with Non Muslims Vol-8

Q# 82

Animals

[QUOTE who="TRD"] 1. Did you mean to suggest that inanimate objects and animals known to have acted on instinct alone actually exercise free will? That's just silly...almost as silly as the concept of Islam superseding the preexisting religions.

2. Here's a tip: Dream up a replacement for Jerusalem and refrain from responding to rhetorical statements. Merry Christmas. [/QUOTE] Ans. Yes, I mean that EVERY Creature in this Universe follows the laws which their Creator made for them. None of them have any choice but to follow their instinct. Not only Animals, most of our body is also Muslim. It follows the course set by the Creator. Our seeing, our hearing, our talking, our respiratory system, our digestive system.they all are Muslims!!

Only limited actions, on which we have been given choice are non Muslims and they also have to be made Muslims by our own free will and that is the test of life!

2. Replacement of Jerusalem is already there, it is called Makkah. It has replace the cente for all human races and nations to assemble and sing prayers to the lord. It happened almost 1400 years back.

The New Jerusalem has far surpassed the glory of old Jerusalem by many degrees.even when Jerusalem was at its peak.

But we do not rest our claim on Old Jerusalem. You just do not discard things like that simply because they are old. These are relics of our past prophets of God.united in Islam, worship on One True Lord and God of the Universe.

85 of 277

Questions and Answers with Non Muslims Vol-8

Q# 83

Apostasy

[QUOTE who="KI"] 01. By the same token you don't have all the stats of people who left Islam. Correct ?

02. Conversely the people who left Islam may have all the knowledge about it and decide to leave Islam ? 03. Why do you assume ignorance in their part. Can you give facts (not opinion) on the "conversion stories"?

04. You throw in a lot of cut paste here, pad it with your opinion and then try to establish as your universal truth. Can you be unbiased for a change? [/QUOTE] Ans.

01. I do not have all the statistics about all the people who entered Islam and I also do not have all the statistics about all the people who left Islam either. I am not an encyclopedia of this subject.

02. It is your assumptions that people who left Islam did it after lots and lots of study abo Islam and then changed it for other religions. Whatever conversions story I see on these threads and at other places, seem otherwise. Whatever charge these people put for leaving Islam, the same things are available in the religions they went into.

03. If some one thinks that Islams One True God Theory is very unreasonable and Hinduism 24 Million gods is more reasonable and broad minded.that shows his level of study!

Or One in Three and Three in One is a better and more reasonable than Islam absolute Unity principle.that also shows his level of study.

And If God has a Begotten Son is More logical and reasonable than Islam God does not beget and not begotten, that shows his level of study. And if some one thinks that Islam has a bloody past..then he or she does not know the past history of other religions Islam climes to be the Straight Path that links humans to God. And Straight Path (by definition) is the Shortest and Simplest path to the destination. If some one leaves the Straight Path and wants to follow a crooked or a twisty or risky path, it is his own risk. There are many more chances that he will never reach his destination.

86 of 277

Questions and Answers with Non Muslims Vol-8

Q# 84

Apostasy

[QUOTE who="Mahmood"] Dear Brother MUQ,

You really don't like me do you? No need to hate me so much as I wish no harm should come to you even though we are theologically in opposition. Anyway, lets cut to the chas Here is what Ibn Kathir has to say about 76:30-31: .. Ibn Kathir corroborates what I have been claiming all along. Allah does not want me embracing Islam or else he would have pre-programmed by brain to be somewhat like yours. [/QUOTE] Ans. My Dear Brother Mahmood, You have said it correctly that I do not like you, but on the other hand I do not hate you either. The emotion that I have most, is to pity you and feel sorry for you. I feel sorry for you, because all that gifts which God bestowed on you, are being used in exactly opposite directions to what they were ended.

I feel sorry for your parents too, that they saw that their son, whose parents were Muslims to many generations, is lost to apostasy. And just not an apostate but has turned into a bit critic of Islam and its teachings.

And believe me, I had every reason to hate you, because you have used words and expressions in respect to our prophet which no Muslim can tolerate to hear from some on

And by doing that, you have shown what mean and uncouth personality you have. We come across so many Christians and Hindus and Jews and Buddhists and people from oth faith.who renounced their old religion and accepted Islam, but none of them speak against their former religion and their founders of faith in the language and tone which yo use against our Dear and Beloved Prophet.

And what is more, NONE of your arguments are based on your own research, you want t study Islam from the eyes of so called Western Scholars, whose only aim is to criticise an find fault with Islam and paint it in the blackest color they can.

You drink their words and logic like Mothers milk and come and speak in the same tone. If you had done your independent research and had come to some conclusion, it wa a different thing.
87 of 277

Questions and Answers with Non Muslims Vol-8

Q# 85

Apostasy

[QUOTE who="COR"] Tell me MUQ, on what basis do you support the death penalty for apostasy? The basis of the opinion regarding the barbaric death penalty of an apostate is not in any verse of the Qur'an.

"The Qur'an has referred to the issue of apostasy at more than one place (for example see Al-Baqarah 2: 217, Al-Baqarah 2: 108, Aal Imraan 3: 90, Al-Nisaa 4: 137 and A Nahl 16: 106). But at none of these instances does the Qur'an mention the punishmen of death for such people, Moreover, the Qur'an clearly mentions in Al-Baqarah 2: 256 that: "There is absolutely no compulsion in religion" Sounds like you are on shaky ground MUQ. .. [/QUOTE] Ans. Thank you for asking a very good and pertinent question. My answer will be a little long and I hope you will bear with me. A. Sources of Islamic Laws: 01. You have forgotten that in Islam, Quran is not the ONLY source of law.

Quran is the Prime and Foremost source, then we have Hadith, i.e. sayings and actions of our Prophet and then we have the actions of First Four Caliphs and then the Unity of all Muslim Scholars.

02. These are the four sources of Islamic laws, and Islamic Scholars have written books o Islamic Jurisprudence using these four sources. 03. You are very correct that in Quran, Allah has not mentioned any punishment for Apostasy, only warning the person of punishment on the hereafter. B. Source of law of Apostasy:

01. But sayings and actions of prophet and his companions show that anyone leaving Isla has to be killed. 02. The punishment is not however for the crime of apostasy, because the person will be punished in the hereafter, this is done only to save the society from trails and
88 of 277

Questions and Answers with Non Muslims Vol-8

Q# 86

Apostasy

[QUOTE who="Mokhammad"]<quoted text> MUQ the moron Saudi girl executed for becoming Christian A young girl in Saudi Arabia was brutally executed by her Muslim father this week after he learned his daughter had converted to Christianity. Middle East business news website Zawya.com reported that the man, who is a prominen member of a "virtue committee," first cut out his daughter's tongue and held a onesided religious debate with her. He then burned his daughter alive. Observant Muslims hold that their Prophet Mohammed taught that Muslims who convert to any other religion must be killed, often in extremely brutal fashion. [/QUOTE] As per my knowledge of Islam, no Muslim is allowed to take life of another Muslim on whatever pretext.

Whatever the crime might be, it has to be punished thru the court after due process of law No one is exempt from this rule, no father can kill his son or daughter or wife or anyone for whatever things they have done. The punishment for apostasy is death, but ONLY AFTER an Islamic Court hears the culprit and pronounces the Judgement.

I am not sure that the story as mentioned in the link you gave is truth, but even if it be tru the father is at fault and he has to be punished as per Islamic laws. PS: By the way, why the mistakes of Individuals are hung around the religion of Islam?

I hear so many parents in India kill their sons or daughters as offering to some idols, or fo getting some treasures.... No one blames Hinduism for their crimes. Why you people invent every rule for Islam only? Is there shortage of extremists and weird people and criminals in any religion?

But when it comes to Muslims, you start blaming Islam without first checking and findin out what is the Islamic law about it? I see such tendency in most people, and you are a prime example to promote hate and hatred based on these false reports.

89 of 277

Questions and Answers with Non Muslims Vol-8

Q# 87

Arabs

[QUOTE who="Halfapatriot"]<quoted text> I faced it in the UAE too. Most Arabs actually look down upon Indians, Muslim or not. But there are a few who are real gentlemen.[/QUOTE] Ans. You are right in that. There are very good people amongst Arabs and there are really bad people also amongst them. But compared to people in other nations, Arabs have more good people than bad people.

But outside workers mainly have to deal with bad people (who are their employers) that i why they get a bad general opinion about these Arab people. And who says Saudis are Ideal Muslims? In fact after getting Oil wealth, Saudis have don more harm to damage and malign the image of Islam and Muslims than any one else It is the only time in Islamic History that so many Non Muslims came to work in Arabia and so less were impressed by Islam and accepted islam. I am not praising Saudis or Saudi Arabia....I am praising Islam and its principles. Just because Saudis or other Arabs (or all Muslims for that matter) are hypocrites, Islam and its teachings will not become false.

That Bangladeshi Driver got the meaning of Islam wrong...a poor Muslim Bangladeshi Rickshaw puller who works honestly for earning his living and following Islam , in t eyes of Allah could be more worthy than 1000 of richest Arabian Sheikhs who are hypocrite Muslims.

Allah does not look at our faces and our wealth and our nationalities....He looks at our fai and what is inside our hearts.

90 of 277

Questions and Answers with Non Muslims Vol-8

Q# 88

Brahmin

[QUOTE who="Ibrahmin"]

There is a strong reason why i say Quraishis and Brahmins are one and the same. I have done 20 years research on this topic, discussed with numerous Islamic scholars and arrive at this conclusion. ---------1. All Qurayshis didn't accept Islam, including Abu Talib, the uncle of Prophet(pbuh). Where did they go?

2. In India and Pakistan, Qurayshi is a very common family name. I waorked in Gulf for years and i NEVER met any Arab with Qurayshi family name. When i asked my Arab friends, they said "Qurayshi family name was dropped by Arabs because it reminds all th bad things they did for our Prophet(pbuh)". May be few are there still, not sure. This is a strong proof that Qurayshi families migrated to Indian subcontinent along with some of their family members who didn't accept Islam.

3. The striking similarity between the Hajj worship rituals and Tirupathi rituals by Achary Brahmins. No where else it exists.

4. Every Brahmin claims that Kaaba was a Hindu temple before Islam. No other idol worshiper from any part of the world has claimed any such relationship with Kaaba temp till date.

5. Above all, Dr.Zakir Naik quotes amazing evidences from Vedas about the prophecy of Prophet Muhammad(pbuh). He says that the prophecy of "Kalki Avatar" is nothing but Prophet Muhammad(pbuh).

If we believe the Prophecy in Vedas as per the research work of Dr.Zakir Naik, it leads to the conclusion that Prophet(pbuh) was born in the tribe of noble Brahmins.

Vedas belong to noble Brahmins. Kalki Avatar can't be a Pariah or Dalit or a low caste untouchable. It has to be a noble Brahmin, as per Vedas. So, why can't Dr.Zakir Naik say boldly that Prophet Muhammad(pbuh) was born in the tribe of noble Brahmins?. http://www.irf.net/index.php... ----------

I am honoring 80 million Brahmins by this proof and inviting them to accept Islam. Even 1% accept Islam, all Hindu Muslim problems will vanish. Indian subcontinent will becom prosperous and most powerful leader for Muslims. .. [/QUOTE]
91 of 277

Questions and Answers with Non Muslims Vol-8

Q# 89

Brahmins About Brahmins: We have heard enough boasts about Brahmins and their Super Human powers to transform any society they lead.

I would have ignored all of that, had not been this consistent claim was not made; that 7 million Brahmins of India can lead and cure all the problems of 1.7 Billion Muslims in th world.

I will certainly like to find out what is the truth of this claim, but let me first point out som Hard facts about Islam and Muslims. A. According to Islam, all humans are created equal: As per Quran all humans are children of same parents, Adam and Eve. So there is no inherent superiority of any human born into any clan or sub-clan. Quran says O mankind, We have created you from same (pair of) male and female and divided you into nations and tribes, so that you may know each other.

Our prophet said There is no superiority of an Arab over a Non Arab or a Non Arab ove an Arab. There is no superiority of a white man over black or a black man over a white. You are all children of Adam and Adam was created from dust

So our prophet never justified any one being born into any family as inherently superior or intelligent over one that is born into another. Our prophet was as much at ease with his clan people as he was with Bilal the African , Suhaib the Roman, Salman the Persian or Abu Musa from Yemen. And racial discrimination is against the basic teachings of Islam. B. Who are these Brahmins:

Just because Brahmin sounds similar to Abraham, these people jump to claim that they ar descendents of Abraham. From which branch of Abrahams family they are? From Isaac or from Ishmael? I am to ask them, because they cannot be from both. But if they originated from Abraham, then what about Hinduism, which was the oldest religion on this earth? Or they claim that Abraham was also a Hindu? C. Did these Brahmins ruled Makkah for 2500 years?
92 of 277

Questions and Answers with Non Muslims Vol-8

Q# 90

Calamity

[QUOTE who="Dragnet"] 01. Your arguments are illogical to say the least. Others can also say that your god didn't protect muslims from the war that western nations are waging on some muslim countries. ..

02. Another argument of yours that Hindus are praying to a dead god has no validity 03. Why is it given in the Koran that muslims must take the laws into their own hands if anything connected with islam is disrespected? Is 04. Why do mullahs regularly issue fatwas? 05. And what about the verses which say that Muhammad rode on a winged horse to heaven? .. [/QUOTE] Ans.

Your post is based on a number of false assumptions and false logics, let me see how man can I clarify: A. Muslims and protection from Natural Calamities and wars etc:

I do not see any where in Quran and Prophet sayings that Muslims will be protected from natural calamities or in wars by Allah. On the contrary it is mentioned in Quran that We (i.e. Allah) shall test you with something of fear and hunger and some loss in goods, lives and the fruits (of your toil), but give Glad tidings to those who patiently persevere (2:155)

If Muslims were exempt from these Natural calamities and war casualties , then who will not choose to become a Muslim?

The promise of Allah to protect and preserve Muslims (who do good deeds also) from all fear and mishaps is for the next world, and not this world. 02. Do Hindus Worship Dead God: I did not say that Hindus worship a dead God. But I said that these idols and statues whom they worship are dead and have no life whatsoever.

They can neither protect themselves nor those who worship them, and are not even aware if someone is worshipping them or cursing them.

93 of 277

Questions and Answers with Non Muslims Vol-8

Q# 91

Calendar

[QUOTE who="Voice.."]

ONCE again, read your BIBLE and find that when moons are used its used in reference o satan. We are children of the light,, not darkness...

Once again you show your ignorance about life and for your information, a month is NOT an equal amount of days in moon phase. Why do you try so hard to show your ignorance? It isnt hard for the rest of us to see it without you pressing the presence. READ YOUR BIBLE! [/QUOTE] Ans.

I was speaking about the Calendars and how the moon was used as perpetual calendar b all ancient civilizations. Because Moon was the ONLY thing for them to mark the time, its changing phases and full moon and crescent moon were the phases to mark their days and events. I know that in India Hindus used Moon to fix the dates of their festivals. All of their festivals are linked to phases of moon even today. So there was nothing wrong in my statement when I said that Moon was a perpetual calendar fixed by God so that humans can mark days and months according to it. And it is ridicules to say that Lunar Months keep changing from month to monthsome times they are 29 days and sometimes 30 days. I want to ask you are all solar Months equal? Why some months are 30 days and some months of 31 days and what about Februarywhy it keeps changing from 28 to 29 days?

Do you know the History of these Solar Months and how many twists and turns they have taken place over the centuries?

If you use Lunar Calendar, you calculations cannot go wrong by more than one day and they get corrected , and if you use only Solar Calendar, then you do not realize your mistakes untill many centuries are over and then you have to remove many days and months from your Calendar to readjust. That is the history of Solar Calendar versus Lunar Calendar go and read it.. You ask me to readwhile your own study is very incomplete.
94 of 277

Questions and Answers with Non Muslims Vol-8

Q# 92

Carlyle

[QUOTE who="Mahmood"] I am not sure what Carlyle's ulterior motives were. I doubt if he read Ishaq or Tabari because I doubt if their works were even translated then. This was a man who didn't do an honest days work when he migrated to Medina. A man who lived off the Spoils of War. ..

Another point I forgot make. It's not just about Mohammad. It's about the teachings of Islam and it's desire for world domination. Islam at it's zenith of power, did not fully impose Shriah on the masses. The caliphs & kings all led double lives. Opiates, harem and farticide were common practice among the Persian and Turkish rulers. I will tou upon this later. [/QUOTE] Ans. A. Carlyle And his depth of knowledge:

I do not like to venture a guess, because I have no real knowledge about it. But from his lecture, I can see that he was fairly well informed about the life history of our prophe The age in which he lived was the one, when cobweb and distortion and bias and hatred created in Europe as a result of Crusade Wars was getting removed.

Muslims were no more a Threat to the European nations and many Muslim countries wer under European control. The Europeans had time to study Islam and Muslims in more detail and many of the old myths and wrong conceptions about Islam were being removed. Carlyle as a man of letter and philosopher and thinker would not be unmindful of all this and I expect him to be well read. But as I said , he did not know Arabic so he could not study these books first handed and had to rely upon the faulty and biased translations of European Orientalists.

That despite these handicaps he could find out the truth, speaks a lot about his personality and fairness of mind and that his heart on the right side B. Michel H. Hart and The 100:

And that was the same reason why Michael H Hart chose our prophet to lead the list of 10 Most influential persons thru out the History.

95 of 277

Questions and Answers with Non Muslims Vol-8

Q# 93

Carlyle

[QUOTE who="Mahmood"]

01. Did Thomas Carlyle embrace Islam? And did he read the shahadah & abide by it's fiv pillars? 02. Hitler, Ganghis Khan, Stalin, Hanibal, Alexander the Great were all great men, but were they evil? The answer is yes.

02. Mohammad in his own deluded mind considered himself a prophet of god, but could never provide evidence for his claim. That is why he hardly gained a following during his first 13 years of preaching in Mecca. It's only after he went to Medina, fought battles, engaged in violence, that made him a great hero. No doubt he was gre man. [/QUOTE] Ans. 01. Unfortunately Thomas Carlyle did not embrace Islam, the real reasons are known to him only and Allah , Who Knows Best.

But my guess is that he did not know Arabic so he could not get the real message of Islam and he had to rely on faulty translations done by biased Western Scholars like Sale & co. And the second reason was his feeling of racial superiority over Muslims, because by that time Europeans had taken control of most of Muslim nations and his views of Muslims was that of a victor over the ones whom they conquered. But he was honest enough to put in writing the truth as far as he could go.

I prey to Allah have mercy on his soul, because his lecture was a trend setter and it cause many, many people to accept Islam and have a favorable opinion about Islam and its prophet (Gibbon also had done it before Carlyle, but id did not have that much effect

02. Yes Hitler, Genghis Khan, Stalin , Hannibal & co. were great men that is why their empires and their thoughts are only to be found in the books of history! and most o their country men do not like their names to be repeated. Prophets of God are Great men in a different sense. Their names and their teachings and their followers keep on increasing with time and their esteem and honor keeps on increasing with time. The pygmies like you casting doubts on his personality and character will not make slightest dent in his esteem, in his honor or the love that his followers have for him.

96 of 277

Questions and Answers with Non Muslims Vol-8

Q# 94

Christianit y

[QUOTE who="WN"]

01. We can and do research to satisfy ourselves. When we see that something is immoral unethical and evil we can walk away at that moment without any more research. 02. Are you saying because Christians once stoned each other in the streets it's okay for Muslims to do so? That's not a very good reason for believing.

I find Christianity to be just as evil as Islam. I'm not a Christian, either. But at least here i the US, neither of your groups are able to carry out your murders in the name of your god You want people to research past the evil of your religion...it's not necessary.

03. By the way, I have never said I agree with killing in any way. Not by missiles, bombs or any other manner. It happens on a scale which I as an individual cannot control.

04. While I do not agree with the wars my country may wage, I support the _people_ who are sent there. I support their families. I wish no one ever had to die in a war, but my individual power does not reach that far.

05. For you to claim that every American (or Frenchman, or Englishman) agrees with wa is not only foolish but very ignorant on your part. [/QUOTE] Ans. 01. The fruits of your research are available for every one to see. They are always going the negative direction. You are lowering the scale of morality after each research, so that "lesser and lesser" number of your people can be considered "Immoral" Your research has resulted in: - "Legalizing Bastard Children", - "Free sex between cones - Legalizing Homo sexuality" - "Lowering the age of consent of marriage".etc. I do not know a Single case where your Researches have provided any "Higher" standard.and this is "very natural" too! If you turn away from Divine Guidance.then it is your own vanity and Satan who are
97 of 277

Questions and Answers with Non Muslims Vol-8

Q# 95

Christianit y

[QUOTE who="SS"]

1. None of the Jewish prophets except Jesus started a new religion which has made it a quest to wipe out the original religion. Mohammad tagged onto Christianity, so by defaul he would not be accepted as a prophet to the Jews. He also started a new religion. 2. Besides that, what do YOU know about Jews or Messianic Christians to make you an authority?? I doubt that you've ever met either one, or studied what they believe. [/QUOTE] Ans.

01. All of the Jewish prophets (post Moses) were followers of the Moses laws. They wer only what we can call a revivalists. They were reminding and warning people when they were exceeding the limits. And that is very clear when we read their sermons and their teachings in the OT books. You are right; Jesus did not come to bring a new religion. His sermons and teachings do not indicate that he came to only reform the Jewish practices which had become too ritualistic.

Our prophet did not tag on to either Judaism or Christianity, he came with the authority to perfect and protect the Universal religion of all prophets. He came with a New Code of Law as Moses had done before him (Remember Deut. 18:18, a Prophet like Moses, and Deut 34: There did not rise any prophet in Israel like Moses)

02. This is a very interesting question, how come I became an authority on Jews, Christia and Messianic Christians and so many sects!!

Have you been in lock and key business? There are thousands of locks, each with differen combinations, but there is a MASTER key, which opens all locks. Quran is that Master key which solves all these small little problems which Jews and Christians and other faith people have. People may think that they have invented new problems , but in fact these are Old problems presented in a modified form.

If you have knowledge of Quran, you can pinpoint the mistakes people are making in the arguments.

98 of 277

Questions and Answers with Non Muslims Vol-8

Q# 96

Christianit y

[QUOTE who="WN"]

01. MUQ, the more you study any holy book the more it shows itself to NOT be the word of any God, but the words of human beings alone. 02. You have told me several times that I am 'wrong', but now tell me that you're not pushing your religion but tell me that studying the Bible will make you an atheist but studying your holy book will not. 03. And, I am not an atheist. 04. You are here as a Muslim who has never been a Christian nor studied the Christian Bible deeply for years ..

05. I am here as an ex-Christian of over 5 decades who knows the Bible well enough to tr to talk to people about their own book and their own belief system BECAUSE I was a Christian.

06. When you realize that portions of your religion are so repugnant to human life as to b rejected immediately by any moral human being, come back and tell us how wrong we ar [/QUOTE] Ans.

01. I am sure that you have never ever seriously studied Quran from cover to cover. After reading Bible from cover to cover, I can certainly say that one would not need to do that again, (but only read selected portion). 02. Quran is totally different, the more you recite it in Arabic, the more it grows on you. Even of you hear Quran being recited in Arabic you are attracted towards it. Every verse Quran speaks that it is not written by any human.

Deep study of Bible will lead you into disbelieve, because there are mistakes, there are contradictions and so many filthy stories and acts of deception that keep your mind wondering, if these people were really prophets of God and do prophets of God behave li that.

In Quran you will find a totally different scenario, you find here the prophets as righteous servants of God, with no distortions and kinks in their personalities.

After reading Quran, you develop respect for prophets and their mission and that can nev lead you to atheism. 03. If you are no Atheist, then what is the name of person who has no religion,?
99 of 277

Questions and Answers with Non Muslims Vol-8

Q# 97

Christianit y

[QUOTE who="peter"] 01. Christ practiced and preached faith in Israels God. Muhammad practiced fear and force by Islams god. 02.Conversion to Christ is voluntary. Conversion to Islam is compulsory. Is this true Muq ? or false.....?

[/QUOTE] Ans. Again let us see, can we bring the two sides together.

01. Both Jesus and Mohammad and all prophets around the globe practiced and preached to believe un the same Universal Creator, Sustainer and Cherished on this Universe. He is essentially the same, but due to change in language different nations know Him by different names. In some languages e is known as God, Deus, YHWH , Eli. Elohim.. In some languages , He is known as Ishwar, Wahe Guru, Parameshwar.. In Arabic language, He is known as Allah All different names pointing to the same Unique Personality, True God of Universe, the same whom all prophets believed in and asked their followers to worship. 02. Believe in Christianity is optional for those who want to get Eternal Life And

Believe in Islam is compulsory for those who want to save themselves from Hell fire an Enter Paradise. Are we not talking about the same thing guys?!!

100 of 277

Questions and Answers with Non Muslims Vol-8

Q# 98

Christianit y

[QUOTE who="Peter"] The truth is the truth, we know you are enraged by it . 01. Christ is in the midst of believers who gather in His name. Muslims must pray facing the Kaaba in Mecca, but is Muhammad there?

02. Christs tomb is empty because He was raised from the dead. The preponderance of proof is overwhelming. Muhammads tomb is still occupied. Islamists say he ascended to heaven on a white horse, but no one saw him rise from his grave and ascend. 03. Christ said Whoever believes in Me will not perish but have everlasting life. Muhammad said Deny Christ and convert to Islam or die. [/QUOTE] Ans. I am not enraged in the least, I am enjoying and having fun. But people falsehood in the garb of truth and try to convince people by false logic and false reasoning.

I am seeing that people are trying their best to create hate and hatred and I am trying to patch up and try to join. I do not know who said Blessed are the peace makers, for theirs is the Kingdom of heaven. Who is assuming the role of peace maker, people should decide. 1. Neither Christ, Nor Mohammad are present in material form at any place. It is just an empty claim and boast that has no reasonable or scientific base.

2. Jesus s so called tomb was always empty, because it was never occupied by Jesus. He was taken to heaven bodily even before he could be arrested by Jews. He left his mission unfinished leaving it for his Natural Successor he will return towards th end of time to kill that Anti Christ

Our prophet led a happy life, completed his mission and died a peaceful death and is buri and millions of Muslim visit his grave every year and present their salutation to him and his soul. What better ending can a man desire.

3. Yes Jesus said, follow me and get Eternal Life as every prophet before him made th same claim.

101 of 277

Questions and Answers with Non Muslims Vol-8

Q# 99

Christianit y

[QUOTE who="Peter"] 1. Christ was and is a Deliverer, Muhammad forced people into legalism and bondage. 2. Christ never killed anyone and He saved many, Muhammad and his followers murdered millions. 3. Christ healed the sick, the lame and the blind. Muhammad never healed any sick person. 2. 4. Christ said, Love your enemies, Muhammad said, Slay the infidel Christians and Jews. 5. Christ overcame by the Sword of the Spirit saving mens souls, Muhammad conquered with the sword killing his enemies. 6. Christ, Gods Eternal Word, was made flesh and dwelt among us. Muhammad said the Koran was Allahs word sent down from heaven. [/QUOTE] Ans. Let me again try to patch up, what this eternal hate monger wants to rent asunder. I do not know when he will start comparing Jesus with Moses or even David!!

1. Jesus said I am not come to abolish the law but to fulfill it, and his Natural Successo came with the latest version of law to be used till the end of time.

2. Christ was powerless to put his words into actions, due to lack of resources and shortag of manpower.. But his natural successor the promised comforter established th Kingdom of Heaven and rule of law on Arabia first then to the rest of world.

(And the followers of Jesus soon forgot the command of Jesus to not kill anyone and they went on killing spree and have killed more humans than all other religions combined

The number of humans killed only during WW-1 and WW-2 (which was mainly fought between Christians) far exceeds all humans killed in all wars from the first day till th time.)

3. Christ healed many of bodily sickness, but could not change their hearts (none of those he healed joined his side and helped him when he was arrested, tried and crucified lik
102 of 277

Questions and Answers with Non Muslims Vol-8

Q# 100

Christianit y

[QUOTE who="Peter"] 01. Jesus Christ is the Prince of Peace Muhammad was a prophet of war. 02. Christs disciples were killed for their faith in Jesus. Muhammads disciples killed for the faith of Islam. 03. Christ was and is the giver of life. Muhammad was a taker of life.

04. Christ practiced and preached faith in Israels God. Muhammad practiced fear and force by Islams god. 05. Conversion to Christ is voluntary. Conversion to Islam is compulsory. 06. Masses believe in Christ and live, Masses accept Islam or die.

[/QUOTE] Ans. Yours was a very biased sort of comparison and you putting jesus and our prophet as adversary of each other. All the prophets of God came to deliver the same message but their roles were different.

The role of Moses was different from the role of David and the role of David was differen from the role of jesus, and the role of our prophet was different from that of Jesus. So we should not put these prophets as adversaries of each other. So I would modify your couplets as follows:

01. While Jesus preached peace to the world, he could not establish it. It was Mohammad who established peace and religion of truth in the land.

02. Christs disciples were few in numbers and they were overwhelmed by their powerful enemies while Mohammad made his disciples so numerous and so strong that they overwhelmed their enemies and established the rule of truth and justice in the land.

103 of 277

Questions and Answers with Non Muslims Vol-8

Q# 101

Circumcis ion

[QUOTE who=" "] [/QUOTE] Ans. Male circumcision is very healthy and hygienic for humans. It has no or very little side effects.

Medical and hygienic advantages of circumcision far outweigh its side effects or harms if any. But it is not a dogma, it does not give you one way ticket to heaven. And not having circumcision will not get you one way ticket to hell either. So we should have a balanced approach on it, which Islam has,. It is a recommended practice and way of prophets of God, that is all.

Jews and Christians have taken extreme position on it based on their religious scripture... And Quran by the way, does not has a "Single verse" on it!! I think this is the best way to sum up all these pages upon pages of meaningless discussions.

104 of 277

Questions and Answers with Non Muslims Vol-8

Q#

Circumcis ion

[QUOTE who="FEN"] Good is not a dogma.. I'm happy about that. I just wanted to tell you that I'm talking about this just in a medical POV.

I really keep beside religion and traditions, because Jesus was circumcised, Jews and Muslims are too...it seems being the only common point about the 3 monotheistic religion And as Atheist I am I'm not searching answers in theology.

Just wanted to tell you that you're all wrong if you think this practice has any positive influence in hygiene, sexual feelings, or any protection against any virusis time to change your mind people, once again I just suggest you to take a look at medical notoriou sites, serious ones. All this is beyond any religion. That's all , simple as I'm exposing it. [/QUOTE] Ans.

If you are not a biased Medical Doctor and consider ALL aspects of benefits and harms o Circumcision, you will find that it has very little ill or side effects and its benefits far outweigh its losses. That is why in Most Hospitals, doctors perform this operation on newly born babies, irrespective of their religion. It is only Jews who raise this to the level of Dogma..Christians on the other hand (following logics of St Paul) are also against it as a Dogma. Muslims do it, because it was recommended by our prophet, but we do not raise it to the level of Dogma.

I hope you also look at Medical sites that show the benefits of Circumcision.you shoul broaden your outlook and be not so narrow minded to call it Mutilation of human body

105 of 277

Questions and Answers with Non Muslims Vol-8

Q# 102

Circumcis ion

[QUOTE who="Drone"] 01. You are just trying to justify want is written in Hadith one way or the other. 02. Regarding your question who wrote guru Granth guru Granth is compiled by our 5th master . 03. Now about circumcision .it was never ordained by allah please for sake of Allah(god

If Allah wanted he would have removed it permanently the body is created by him and he can modify it if he wants .it was for hygienic purpose in desert area but later people attached it to religion . 04. That tells me your Quran is not a revealed by Allah .it is human composition and he hides his name too .

05. When Mohammad said prophets were sent to all nations before , where are the prophe of India ,china ,.don't you think he had just knowledge of few surrounding nations only .

06. If he was capable of splitting the moon then he should have knowledge about the wor why did all the prophets were born in middle east area only . [/QUOTE] Ans. I thank you for your post. Here are short answers for your points.

01. I am trying to justify what is written in Hadith, because it is sayings and actions of ou prophet and a prophet of God can never tell a lie or do a wrong thing.

People object on Hadith without thinking and without using their minds and that is where they are wrong. 02. If Guru Granth Sahib was written by the Fifth Guru, then the first four Gurus were unaware of it and did not believe in it, so they were not Sikhs? Is it true.

I know there is lot of things which Sikhs have taken from Islam, and really there should b no bad blood between Muslims and Sikhs.

But because of those political fights with Moguls, Sikhs have become enemies of Islam and they did maximum killing and carnage during the partition and when people wer moving from India to Pakistan.

106 of 277

Questions and Answers with Non Muslims Vol-8

Q# 103

Conversio n

[QUOTE who=Allen"] Zakat ( charity )comes in two forms - one, only to muslims. two. to the infidel with the intention to convert [/QUOTE] Ans.

There are eight heads in which Zakat (Compulsory Charity , taken for Muslims to be distributed amongst the Poor of Muslims) is to be spent on following classes of peole a. To poor b. To Needy people c. Those who work in collection of Zakat. d. To help those Muslims who have recently converted to Islam and who need financial help so that their faith be strengthened. e. To help those who are in debts. f. To free slaves. g. To help the wayfarers h. In Preparation for Jihad These are eight heads under which Islamic Zakat is to be distributed. But A muslim is exhorted to spend more than the Compulsory Zakat.this is called voluntary charity and this is spend on both Muslims and Non Muslims.

If there is no one left in Muslim land to receive Zakat (it did happen for many years durin height of Muslim rule over the world)..then even Zakat money can be spent on No Muslims.

107 of 277

Questions and Answers with Non Muslims Vol-8

Q# 104

Conversio n

[QUOTE who=" "] [/QUOTE] Ans. 10 Reason to become Muslim: I have this challenge being thrown at me to name 10 reasons why people should become Muslim, when I reflected on this, I found that 10 is a very big figure, in fact there is only Main (primary) Reason for people to become Muslim, all other reasons are secondary! Primary Reason for People to Become Muslim: If someone wants to avoid being thrown into the hell fire, he or she must not die, till he she is in the state of Islam! This is the reason and proof for this statement comes from following Quranic verses: a. Islam is the only way of life before Allah:

The (only) DEEN (way of life) before Allah is Islam (i.e. submission to His will) (3:19) b. Allah will accept no other DEEN (way of life) in hereafter:

If any one desires a DEEN (way of life) other than Islam (i.e. submission to Allah will never will be accepted of him, and in the hereafter he will be in the ranks of those who have lost (All spiritual good) (3:82)

c. The advice given by Prophet Abraham and Jacob to their sons (do not die, except in the sate of Islam) :

This is the legacy that Abraham left to his sons, and so did Jacob: O my sons! Allah ha chosen the DEEN (way of life) for you: then die not except in the faith of Islam (2:132) d. And the warning given by prophet Jesus:

But said the Christ O children of Israel worship Allah, who in my Lord and your Lord Whosever joins another partner with Allah, Allah with forbid him Paradise and the Hell Fire shall be his abode. There will be for the wrong doers there be no helpers ( 5:75) e. The Real success is success of hereafter:

Every soul shall have the taste of death: and only on the Day of Judgment shall you be paid your full recompense. Only he who is saved from Hell Fire and admitted to Paradise will have attained the object (of life). For the life of this world is but goods and chattels o
108 of 277

Questions and Answers with Non Muslims Vol-8

Q# 105

Conversio n

[QUOTE who="mac"] Mr. new logic , Nizam offered money to Ambedkar for converting islam but he refused. Because he wants to accept an "Indian Religion"(from Indian soil), not any "Non Indian religion".

Second reason was he was atheist, hindu can be atheist, Buddhist are complete atheis But islamic can not be an atheist. [/QUOTE] Ans. 1. If Nizam offered Ambedkar Money to accept Islam, it was not as a bribe, but only to ease the suffering for his people. In Islam , you cannot bribe a man to accept Islam. 2. Ambedkar might have wanted an Indian religion, then he should have gone for Sikhism which have a separate identity from normal Hindus. And it is a Live religion in India and not something Dead as Buddhism (I am talking from Indias view point).

3. If he was an Atheist, he could not say the same thing for bulk of his followers, who did believe in God (and still do). 4. Ambedkar (for all his genius and qualities) was not of that caliber to revive the Buddhism in India on his own. 5. That is why he left his followers without any new place of worship, no scholars, no scriptures, not even change of name.

6. And the beauty of all that was that he and his followers were linked to the devil of cas system for eternity even after leaving Hinduism and accepting Buddhism!! I hope there is no caste system in Buddhism 7. So from Hind sight it was a very foolish decision and did not uplift Dalits in any way. 8. And then KANSHI RAM and MAYAWATI have to revive the Dalit Movement, and with their caliber and uncouth behavior and taste for corruption, one could see what would be the end result. 9. MAYAWATI is no different from any of the corrupt and uncouth politician and the way she is ruling UP, every one could go and see. 10. She had to INVENT gods and idols for Dalits, some of whom are: a. AMBEDKAR
109 of 277

Questions and Answers with Non Muslims Vol-8

Q# 106

Conversio n

[QUOTE who="TTL"] Mug: have u managed to convert even one person here? U have been at it for months.

Or do you feel that perhaps you have stirred up more anti-Islamic sentiment than there wa already?

It's pretty simple hey, you have your messed up violent, full of lies and untruths, and mos of all un-accepting of others and their beliefs.

In the contrary, the western world is relatively peaceful, most of the war in the world is in the middle east. The west is accepting of others. The democratic ways of the west have le them to be a) the world leader b) better places to live. Its pretty simple. And all of your trickery and lies can't hide the truth. [/QUOTE] Ans. You have raised a very serious point and it made me think a bit. My aim was never to increase hate or hatred for any society or civilization. You can see the posts of mine and compare it with the tones of those who criticize Islam and Muslims. If you think that my posts are harsher and more abusive than theirs, then I do beg your pardon and pardons from those whom I offended. I have always tried to use civil words and have been polite in my responses to my fellow correspondents.

Only times when I have been harsh where the rhetoric replies which I posted in some cas and even there my tone was much milder than the ones whom I was replying. And my aim on these threads is to clarify doubts and misconceptions people have about Islam. It is my experience that most people have no knowledge about teachings of Islam, their opinion is based on Media reports and what they see black sheep of Muslims doing. I do not defend wrong actions of any one, be him a Muslim or Non Muslim.

As regards actual conversion of a person, I have yet to see any one who has accepted isla or Christianity or any religion on these threads.

110 of 277

Questions and Answers with Non Muslims Vol-8

Chapter-6 Islam-2

111 of 277

Questions and Answers with Non Muslims Vol-8

112 of 277

Questions and Answers with Non Muslims Vol-8

Q# 107

Creation

[QUOTE who=" "] [/QUOTE] Ans. Creation of this Universe and God:

Many people have some basic misunderstanding in the concept of God' Creation process. They are programmed to think (mainly due to Biblical story of Creation) that this whole Universe was created in 6 days of 24 hours period.

Due to this faulty concept, they think that if any process takes say 1000 years to complete it is not done by God, but it happened by its own by slow permutation and combination process.

They fail to understand that time is immaterial for God, who has eternal life, if any proce takes thousands of years (according to our calculation) in God' eyes it does not take any time. He is the Originator and Planner and Executioner of this whole Universe. It is He who made all these laws of Physics, Thermodynamics and Chemistry and every other science. These laws did not come by themselves and none of the scientists who discovered them created them. They were there since the beginning of time. Due to this faulty Biblical concept of God creating this Universe in six days , we see so much infighting on these threads.

Quranic concept of Creation is free from this Biblical defect.... it does not talk about 6 da or 24 hours periods.... it talks about six stages of indefinite time and there is nothing wrong in that from scientific point.

But people are so programmed with Bible and so much misinformed about Quran that the do not ponder and consider Quranic statement with care , otherwise there will be no point of contention between religion and science.

Quran' basic logic is that nothing comes with out its Creator and any one can see that sam laws are applicable in this whole vast Universe, there are no different laws in differen parts of this Universe, that proves that there is "only One Will" working in this whole Universe. The basic fact that this Universe exists and is working like a clock, proves that it has a Creator and Planner at its back. (Nothing can create itself or can be created just by
113 of 277

Questions and Answers with Non Muslims Vol-8

Q# 108

Creation

[QUOTE who="THINK"]<quoted text>MUQ, why don't you explain Islam's teaching ho it is possible that humans were living in heaven before God created the world? Explain how humans got to be in heaven before they were born here on earth?[/QUOTE] Ans.

Why did you put your "scholarly" post before asking Muslims? Why did you not "Think before you posted? The point of issue is "when the human souls were created"? Are they created when a human is born or they were created before birth of humans on this earth?

The evidence in Quran is that Allah created all the souls of human beings before they cam on this earth for earthly life. There is a verse in Quran which says "When your Lord drew forth from the Children of Adam from their loins- their descendents and made them testify concerning themselves (saying): "Am I not your Lord (who cherishes and sustains you)"? They said "Yea! we do testify" (This was done) lest you should say on the day of judgment "Of this we were never mindful" (7:172)

This verse of Quran says that Sprits and souls of all humans were created before they cam into the world and God did take a covenant from them. And the memory of this covenant is preserved in subconscious self of every human, and that is why every human has a concept and notion about God. How much he tries to erase it by artificial means, it sticks to them in one form or other. In the same way, we have sayings of prophet meaning souls of all humans were created many thousands of years before Adam the first human was created. Now the question comes where these souls are stored and how they look like without human bodies? No one really knows the answer and no one can answer it with confidence.

Even if these souls are kept in heavens, there would be a designated place for them.which might be separate from what Righteous servants of God will get in the hereafter. Because that place is when both Body and Soul join together and enjoy bo spiritual as well as physical nourishment. Before they come into earth, every soul is innocent and it is not in the nature of Merciful God to put them into a place of discomfort .so they are kept in a safe and comfortable place, worthy of Mercy and Grace of our Lord and Cherisher.

114 of 277

Questions and Answers with Non Muslims Vol-8

Q# 109

Crime

[QUOTE who="CHTT"]

These punishments should never be allowed and are indeed, not allowed or ever will be in the UK. You seem to have an hysterical outlook on the world according to the lies your religion teaches you about everything non-Muslim. No crime can justify stoning to death, if there is to be a capital punishment it should be humane and not satisfy the blood lust of religious lunatics, your justice systems are not capable of making a fair and just decision because your laws are based on racism, sexism and a wholly outdated set of values. [/QUOTE] Ans. It is your idea of crime and justice. There are crimes that call for maximum penality and unless you provide them, the criminals would multiply in the society. That is why Quran says In the law of retribution there is life for you, O you men of thinking!! But what is needed is quick and swift punishment, which this Western System does not hasa crime is committed today and the punishment comes after 10 years or so. Islamic criminal justice system is quick and swift and provides for easy and swift dispensing of justice and that is why it keeps the criminal rates to a minimum and makes the society safe for law abiding citizens, who are in overwhelming majority.

But in your society the criminals and pampered and it is the common law abiding citizens who are in overwhelming majority that have to suffer and keep on suffering. And that is why rate of crimes in general and violent crimes and crimes and violence against women is rising and rising and rising in your society and you can do nothing about it. Simmer in your own juices is the short comment which I can make on the criminal status of your society.

115 of 277

Questions and Answers with Non Muslims Vol-8

Q# 110

Criminals

[QUOTE who="LWsciencejunkie"]<quoted text> What does Islam say about truly cruel and evil criminals? They don't get to say "OOOOOH, I AM A MUSLIM AND BELIEVE IN ALLAH SO I'M GOING TO PARADISE", do they? .. And the Jews were certainly not wholly removed from the Levant. You know that as well as I do. Some became Muslims, some didn't. Some left because of the Romans, some stayed. Some became Xtians, and they're still there too. Too bad they all can't get along.[/QUOTE]

I asked you to get away from religious topics because they do not suit you. You are very easily ignited and try to change the track. Long back, I had stopped having any discussion with you directly, but somehow you do not leave me and continue to post me direct replies.

I do not wish to continue this series, because the moment, you have no answer, you go in by-lanes and start talking about Muslims and what they are doing.

I have told you many times, I am not discussing Muslims and their action, I am discussin islam and its teachings, talk on these and prove to me that they are wrong and they are unpractical. But you avoid that, because you have no knowledge about religion and scripture. Now let me touch only two points from your latest post: 01. Religion and Criminals: A. Hypothetical Example: a. Let us first take one example, I am made to understand that you have a granddaughter, and obviously you love her very much.

b. You take care of her every need and try to instill into her points of good upbringing an points of etiquette and civilized behaviour. c. You tell her about the ill effects of drug abuse, alcoholism, getting too close to bad boys, shoplifting and all such criminal activities d. But when she grows up, you come to know that she has broken every rule which you told her and has fallen into bad days.

e. Now one night she comes to your house dead in the night, with tears in her eyes and sh says, I am sorry for all what I did in past, and now want to turn back and be a good child.
116 of 277

Questions and Answers with Non Muslims Vol-8

Q# 111

Divorce

[QUOTE who="HS"]

The divorce business is an industry with legs of its own. Fairness to all parties is importan when a marriage dissolves. Yet divorce is more common than we would like. It is becoming more common in Egypt and elsewhere. Societal ills and family breakdowns are perhaps related... [/QUOTE] Ans.

It is immaterial if Divorce is getting more common in Egypt, Divorce is not any disease o sickness per see

If Husband and wife think they cannot live together, they can divorce each other and find other partners. It should be a simple process and essentially a family matter. Not something in which courts should be interested in. (unless there is some cruelty or violence). Islamic system of marriage and divorce is therefore simple and a family matter.

But Western Govt. have per see decided that divorce is the fault of males and in order t protect the rights of women have made it very complicated.

That is why most men and women think it is easier to live-in and do not get involved in the complicated business of marriage and divorce.

And that is why more than half children in US and other European countries are born out of wedlock and are really bastards What will be the future of any country if majority of its population consists of Bastard People?

117 of 277

Questions and Answers with Non Muslims Vol-8

Q# 112

Evil

[QUOTE who="Dragnet"] I think enough has been said on the issue of gender equality. Maybe we can revisit the topic later on if necessary.

I want to know MUQ' s views on the problem of evil from an Islamic standpoint. The problem of evil is perhaps the single greatest malady plaguing mankind and my viewpoint is that religions have dealt with the problem of evil in a less than satisfactory way.

Now here is the Epicurean riddle which captures the enigma that evil is. "Either God wants to abolish evil, and cannot; or he can, but does not want to. If he wants to, but cannot, he is impotent. If he can, but does not want to, he is wicked. If God can abolish evil, and God really wants to do it, why is there evil in the world? [/QUOTE] Ans. Now our learned friend has raised the issue of evil, when he had the leisure of having direct communication link with me, he was obsessed with going on in circles and repeating and re-repeating same arguments. A. What is Evil: In itself, there is nothing which we can call as evil. It is a relative term. In absolute term there is nothing which is evil by itself.

a. Let me take an example, killing an innocent man is perceived as evil by every one including every religion and civil laws of every society, but so many innocent person are killed during natural disasters and earthquakes and lightning strikes, which are ac of God. Can we say that God did evil by killing these innocent people? No, Never. Because Life is a Gift from God and He decides when to give and when to take back that Gift.

b. Another Example, taking of another mans property without his permission or by force is called stealing and robbing and is an evil recognised by every one.

But when the same thing is done by God during natural disasters and earthquakes etc and properties of thousands of people perish it is not an evil act at all. Because every thin in the Universe belongs to God and He decides when to give and when to take back that gift.

c. Third example, to inject any human body with bacteria, or virus or other disease causin thing is an evil thing beyond measure and no one will argue with that.

118 of 277

Questions and Answers with Non Muslims Vol-8

Q# 113

Ezra

[QUOTE who="YPD"] MUQ why do Muslims believe that Yahood think that Ezra was ibn Allah? [/QUOTE] Ans. Because there were Jews during the time of our Prophet (their might still be some Jews) who said that UZAIR was Son of God, the same way Christians say that Jesus is the Son of God. Since the Jews of Medina did not object to this Quranic verse, it is sufficient proof that they did not find any thing wrong with it. And Quran did not say Ezra, it said UZAIR, which could be Ezra or some other person. It need not be the belief of most Jews or Jews of today, but we cannot deny that no Jew ever said that UZAIR or Ezra was Son of God. That when we see so many persons in Bible are called as Son of God.

The epithet Son of God was a very harmless in the Jewish language meaning a godly person, some one closer to God or a Prophet of God, all of these could very well apply to Ezra. In Christian Terminology the Epithet Son of God was reserved only for Jesus and in a totally different sense, making Jesus the Only Begotten Son of God. Allah knows best.

119 of 277

Questions and Answers with Non Muslims Vol-8

Q# 114

Faith

[QUOTE who="Someone"]

MUQ How is that the shahada in the Quran is no where written in one full sentence .why d somebody add Mohammads name to the shahada .shahada is one the pillar in Islam Muslim scholars are trying there best to prove that Quran is scientifically right .but how come all Muslims are divided on the shahada . [/QUOTE] Ans. It is wrong to say that someone added the Mohammad (PBUH) s name in the Shahadah. Both parts of Shahada are found in Quran and if any one reads Quran, it will be clear to him, that Shhadah says nothing else, which is repeated many times in Quran. I.e. LA LILAHA ILL AL ALLAH MOHAMMD AL RASOOL ALLAH. And this Shahadah was the article of faith of every New Muslim who accpeted Islam on the hands of Prophet.

There are hundreds of thousands of such example, so the answer to your question as to w added the name of Mohammad (PBUH) in Shahada, the answer is Mohammad (PBUH) himself!! Allah Knows Best.

120 of 277

Questions and Answers with Non Muslims Vol-8

Q# 115

Faith wman uk wrote: <quoted text> Really you now claim to see through Gods eyes. How is that possible? So you believe every thing the iman tells you ? You revise history to suit yourself why not religion. ie Churchill started the war and was a tall lanky man with a cigar only one of those things was true you posted it as fact. Ans. No I do not see thru God' eyes. This is a very common sense observation. This is the most simple explanation of continuation of same religion and its revival thru prophets and messengers of God thru the ages and thru the different status of its growth in civilization.

people got "stuck up" at some level an refused to acknowledge the prophets that came aft their own prophets, that is how they "created" their own religions and their own names.

All these names of religions and religious groups are self concocted and have no authority from God.

Instead of answering me reasonably and logically, you people try to insult and make fun a me....and this is a clear sign that you people have no logical answers to my comment

121 of 277

Questions and Answers with Non Muslims Vol-8

Q# 116

Family

[QUOTE who="Skeptic"]

They are always one and in marriage they are one in union with each other with the same goal and out come in life. Does your children obey you and disobey your wife? or vice versa that disobey you and obey your wife? Or do the obey and respect you and your wife as one authority? If you are the only one with authority then you don;t have a wife , you have a slave. Which is it? [/QUOTE] Ans. They were never ONE before marriage and they never become ONE after marriage. They retain their individual identities before and after marriage. Their goals and outcome might be different from each other before and after marriage. A man in politics , for example, would have different goals and objectives than his wife who is not in politics. A business tycoon can have totally different goals and objectives than his wife. What unity Islam is thinking is the of the family as a basic unit to build up society and civilization and different roles played by husband and wife. Islam does not consider that they dissolve and become ONE after marriage (and what would you say about those millions of men and women who are living-in without getting married, have they also become ONE?) Most of children now a days obey NONE of their parents and this is common cry from most of parents, just ask for vote on this topic and you will know the reality.

In Islam both husband and wife have authority in their own sphere and they are responsib for their responsibilities.

Our prophet said each one of you is a trustee and every one is responsible for his trust A husband is responsible for his household and he shall be questioned about it.

A wife is responsible for her children and guarding her chastity and wealth of her husban and she will be questioned about it.. Each one of you is a Trustee and each one of you shall be questioned about his trust
122 of 277

Questions and Answers with Non Muslims Vol-8

Q# 117

Followers

[QUOTE who="Jan"] 1. Reality is Christians have more conversion, Islam has higher birth rates. Still more Christians join the world each year than Muslims, mainly due to is already higher numbers and success in conversions. 2. Christianity is penetrated in ALL part of the world, Islam not yet. 3. Bible prophecy tells us the false prophets religion will raise in the final days, they will kill us for following true God and surround Israel and want to wage war on her. 4. I would say your correct according to prophecy Islam will continue to rise and control by force the world for 3.5 years, then God will come and bring peace. Islam will them be no more! 5. By the way a Muslims terrorist tried to murder Christmas shoppers in Stockholm today, he failed and only killed himself! [/QUOTE] Ans. 1. You are not correct in your assumption, that rise of Muslim populations is mostly because of their higher birth rate and Christians due to they getting more converts. While I agree that Christian missionaries are more professional and they are more organized and they have more monetary and other support from wealthy Christians..yet their return on investment is very less as compared to limited Islamic missionary activities. a. Christian Missionary gets converts mostly from poor and ignorant people in remote corners of the world, while Muslims get top notch converts from the hear of Christendom.

b. The quality of Muslim convert is much higher than the quality of Christian converts. I most cases they are content if the person changes his or her name and dangles a cross across their necks. In Islam, he becomes a practicing Muslim almost immediately after conversion. c. Christian missionaries are roaming around remote corners of the world and have no concern about their home bases where Christianity has lost most of its power and appeal. Muslims are saving their Home Base and penetrating into the Christian territories. 2. I answered that already
123 of 277

Questions and Answers with Non Muslims Vol-8

Q# 118

Force

[QUOTE who="Arbitrageur"]

Back to your "ideology", now, after avoiding my earlier line of questioning on the matter I must continue, although you will certainly avoid any specifics, and employ your ambiguous sweeping generalizations.

I see, on your one hand, that "ideology" supposedly cannot be fought with force, and on t other that those who have invaded Afghanistan now have no ideology of their own. You have now upped the ante. ..

Has this ideology versus ideology inherent in Afghanistan's civil wars ever been peaceful or has it always employed your "naked force"?

You know, forceful ideologies fighting forceful ideologies without the influence of foreig lack of ideology. [/QUOTE] Ans. You have confused the issue knowingly or unknowingly. Before starting Afghan operation, your media painted the picture that Taliban have terrorized the whole Afghan population and the moment they were removed from power, Afghan people would jump into the western bandwagon of Freedom from religion (The same faulty logic they assumed for post Saddam Iraq). But even after removing Taliban from power (with the real help from Northern Alliance, other wise US and NATO forces had to suffer heavy casualties) your forces are yet to win the hearts of Afghan people..they see in your daily bombardments and drone attacks and killingsyour scant regards for their culture and their religion. And that s why they are turning against you and Taliban are getting stronger and stronger. I want to know why? What did US and NATO did for the development of Afghanistan (which they promised in bold letters before start of Operation)..the same story of Iraqi development after fall of Saddam is repeated here.

What wrong I did is assuming that US and NATO have no ideology to fight back Taliban and their ideologythey only ASSUME that once Taliban are removed, Afghan
124 of 277

Questions and Answers with Non Muslims Vol-8

Q# 119

God

[QUOTE who="CS"] I suggested you to if you lack something, that does not mean you should show your stupidity on public forum.

All your statement are with some consideration, that the God is always demanding ( Wan a people should worship Him ), God is filled with hatred ( create discrimination among people by calling them Hindu, muslim, christian etc ), God is arrogant ( Asking people to worship only Him otherwise prepare for sever punishment ).

But to let you know this is the Only imaginary God which Muslims are worship. and from that point of you , you raised those foolish question and irony is, you expect logical answ for those silly question. that is stupidity. Chatursingh is just a name, I m not claiming anything. [/QUOTE] Ans. I think you are not a CHATUR person to take a hint and your name is misnomer.

It is not God which divided people into Hindu, Buddhists, Judaists and Christians..thes names are not given by God..but these are the names which followers of these religions picked for themselves.

As per God, every prophet was a Muslim and every religion which these prophets preach was Islam (that is submit your will to will of your creator).

God as Creator, and Sustainer of this Universe is always exalted above His creations. Th is statement of fact, and the praise which Humans bestow on God, does not increase His stature in the least.

And humans have been sent to earth for a test and not to just eat, drink and procreate like an animal and then die and mix in the dust. Humans gave a greater purpose in life than these animals. And now from every path leads to same goal.you have slipped to the position that Muslims only worship imaginary god.and what about the god of Hindus, Jews and Christiansdo they also worship imaginary gods

That is why it is correct in a sense, that all religions worship imaginary gods and therefor they all lead to same goal, nowhere? Is that the pearl of wisdom which you want to distribute on this thread?
125 of 277

Questions and Answers with Non Muslims Vol-8

Q# 120

Gods Will

[QUOTE who="123 N"]

Muslims does not believe in work they just believe that everything will automatically happen as per Allahs will but Allah also want muslims to do hardwork in order to achieve something but muslims r sleeping over this. By saying that allah will do it as per his will is wrong as allah also wants hard work from his people.

[/QUOTE] Ans.

You are not correct, Islam does not want Muslims to sit Idle and hope that every thing wi happen as per Allahs wish. That concept is totally against the teachings of Quran and sayings and action of prophet and his companions. We should try our best and THEN leave the result to Allah, not that we just sit idle.

And the belief that every thing happens as per Allah wish is to console the Muslims, whe they do not see the positive results of their efforts quickly in front of their eyes. And that they should not gloat over any success or riches that they have got. That is the true meaning of Faith in the QADR. Allah Knows Best.

126 of 277

Questions and Answers with Non Muslims Vol-8

Q# 121

hadith

[QUOTE who="COR"]

01. You say that Qur'an is the Prime and Foremost source. That means it overrides the other three sources in matters that it addresses. Well, the Qur'an has strictly disallowe the imposition of the death penalty except in two specific cases. One of them is wher the person is guilty of murdering another person and the other is where a person is guilty of creating unrest in the land -- like being involved in activities that create unrest in a community, for example activities like terrorism etc. The Qur'an says:

02. "Whoever kills a person without his being guilty of murder or of creating unrest i the land, is as though he kills the whole of mankind." (Al-Maaidah 5: 32) Obviously, apostasy can neither be termed as "murder" nor "creating unrest in the land", except under some odd interpretation of "unrest".

03. Ah, so now we get to the root of your issue. You're worried that a person who logical concludes that Islam is false may lead other people of "weak faith" astray. Well, one could reasonably argue that "weak faith" is the result of a "weak religion" and not the fault of the apostate. The verse does NOT say "no one should be compelled to accept Islam". The verse says "There is absolutely no compulsion in religion". It does NOT distinguish between entering a religion or leaving it. You have added your own words to that verse to accommodate your barbaric practice.

05. Two or three wrongs do not make a right. Even if the Jews and Christians engaged in such barbaric, bronze-aged practices, they certainly do not do it today, or else they face the same scrutiny and criticism that Islam does. Let history be the judge of how they behaved in the past.

06. In conclusion, MUQ, I have shown that your religion's practice of murdering apostate is not only barbaric, but has no defensible basis in the Qu'ran and has no place in this world. This is yet another example of a corrupt, bronze-age religious practice dictate by your backward-thinking "scholars" that no amount of rationalization can justify. [/QUOTE] Ans. A. Against Quran or Additional to Quran:

01. Quran is the First and Foremost source of Islamic Shariah. If Quran had allowed or permitted apostasy and then Prophet had banned it, the you could have an argument 02. From Quranic verses it is clear that God does not like Apostasy and threatens these people with dire consequences in the hereafter.
127 of 277

Questions and Answers with Non Muslims Vol-8

Q# 122

Hate

[QUOTE who="CS"] Of course these names are not Given by God BUT THE FALSE GOD OF ISLAM. and that is the main cause of hatred towards Islam. No religion creates discrimination among people but Islam.

"As per God" ??? who told you this. Muhammad told this is in the quran. and what is the credibility of Muhammad, it is mentioned in the Quran that Muhammad is prophet. Is this your logic? Everybody is muslim??, who has given this name??? Your imaginary God ( created by muhammad ). Thats why i said, All those foolish question raised by you had these consideration. that was the shit of wisdom which Muhammad transformed into you people. [/QUOTE] Ans. No religion creates discrimination among people but Islam.

I never thought that I would hear this from a Hindu!! A religion which is based on casteis and permanently divided human being into four castes with graded inequality calling Isla as the Only religion that discriminates between people!

Hinduism discriminates between people on things about which they have No Control. I is not in any one power to be born into a Brahmin family or a Dalit family. Islam at least discriminates based on things which are in human own control and not beyond control. Islam discrimination is issue based and discrimination in Hinduism is arbitrary. Regarding your comments about our prophet, I need not have to answer them. By saying these you are in fact showing your own ignorance and proving by your own words that your name should indeed would have been MOORKH SINGH AND NOT CHATUR SINGH!! PS: And I think that it is time to stop answering your posts. I do not correspond with people who use such type of foul language against our prophet.

128 of 277

Questions and Answers with Non Muslims Vol-8

Q# 123

Hate

[QUOTE who="Redoran"] 01. People are created the same all over this rolling dirt ball.

02. I still don't understand the terrible hatred of muslims against all non-muslim people al over this rolling dirt ball. Why are muslims so hateful against anyone who is nonmuslim? Can you answer this?

03. Your Koran directs you to kill all non-muslims so you can rule this rolling dirt ball. d you agree with this premise? [/QUOTE] Ans.

01. You are right, all humans on this planet are the children of same pair of Adam and Ev That is why Islam wants a Universal brotherhood.

02. You are wrong, it is not Muslims who hate all Non Muslims, it is the other way round

All Non Muslims whether they be Jews, Christians, Hindus and pagans, they ALL unite when there is any issue with Muslims and they try their best to inflict as much damag to Muslims as possible. So you have to look for the cause of this hate in others attitude towards Muslims that otherwise.

03. Quran NEVER directs Muslims to kill ALL Non Muslims, and it was never ever done You cannot find a single case in which Muslims killed every Non Muslim in any city or village (However I can show you cases from Bible, where Jews have done this crime, not one time but many times). So I think your post is really upside down and you are asking victim, why every one hits you. PS: As a Hint as to why EVERY ONE hates Muslims, let me say that as of now, Islam is the ONLY religion for mankind which is approved by God Almighty. That is why all the forces of falsehood join to extinguish this flame of truth. This has not happened for the first time in human history, but has happened repeatedly.

Whenever any prophet of God came with truth, every one (barring a small minority) became his enemy and tried to kill and harm and exterminate him and his followers b any and by every means fair or foul.
129 of 277

Questions and Answers with Non Muslims Vol-8

Q# 124

Heaven

[QUOTE who="Dragnet"]

A. Okay, now here is my next question. In what way does the concept of Islamic paradise impact the behavior of a person in a meaningful way?

B. In contrast hindu philosophy says that you must do good karma or good deeds wit an aim to evolve spiritually. [/QUOTE] Ans.

I think that before you ask me, what is Islamic concept of Paradise, you should have aske what is the purpose of human life on this earth?. When we understand the purpose of our life, then it will become very logical how the concept of Swarg and Nark fits beautifully into it. A. Islamic Concept of purpose of human life:

(I can quote Quranic Verses as references for each point, but omit it to save space- MUQ

1. According to Islam, every thing Created in this Universe has a purpose, and nothing ha been created without a purpose.

2. The Creations of God, consist of things which seem lifeless to us (like Sun, Moon Star mountains), or have limited life symptoms (like trees and vegetation kingdom) or show more signs of life (like Animals), each of them have a well defined role in the Divine Plan of Creation.

3. Everything in this Universe speaks to its Creator, in a manner we may know or we may not know about it. All of these Creations have no free will and live their lives as per plans and guidelines provided by their Creator.

4. It is only humans and Jinn, who has been given free will and have has been sent to eart on a test. Man has been provided the faculties of haring, seeing, tasting, touching an given the gift of reason to analyze truth from the false hood. 5. Humans have been given dominion and power over every other creature on this earth and are superior to every one of them.

6. The test of human beings is that whether they will live their life in accordance with the guidance provided by God. They have their life times as their Capital and they have to pass this test of life using that capital.

130 of 277

Questions and Answers with Non Muslims Vol-8

Q# 125

Heavens

[QUOTE who="Dragnet"]

01. Okay, you have presented a perfect argument in support of Islams idea about heaven and hell. .. 02. The only thing I would like to say is that there are other standpoints which have their own take on the mystery of human life. .. 03. Looking from a purely scientific standpoint, science has been able to argue that man came to his present form through a slow graded evolution ..

04. I would like you to look at another piece of evidence that has been recorded in ou modern times. These are the account of near death experiences. .

A long period of time in the astral world will actually correspond to a very short period in the physical world. .. 05. Now let me tell you what Hindus have to say about heaven and hell. [/QUOTE] Ans. I have summarized your reply under these five heads A. Is Life a Test or Not?

Who can decide whether human life is a test or not? The best one to decide is our Creato and He has categorically stated that our life is indeed a test. Now it is left to us to take it or leave it, but do not complain if you made wrong choice. B. There are other view points: Of course there are other viewpoints, but what is the source of these other view points? Are they based on human thinking or human philosophy or a result of meditation?

Whatever might be the source, it cannot override the knowledge provided by our Creator. C. Theory of Evolution:

131 of 277

Questions and Answers with Non Muslims Vol-8

Q# 126

Hereafter

[QUOTE who="AB"]

Actually, Allah is the maker, ruler and sustainer of the AFTER world...once you realize that you may decide to change your outlook in this world...until then, continue the hatred and I'm certain this will expedite your trip to that afterworld...as for me, I'm not sure if there are baby back ribs in the afterworld so I think I'll have a slab smothered in old number 7 bbq sauce for lunch, sure hope your nation of islam forgives me... [/QUOTE] Ans.

You are wrong, Allah is also the Creator of this world. Who made sin, moon, planets, sea and mountains and rivers? Who made all these animals and plants and fruits? Who placed all these minerals in the earth? If you think humans had any say in any on these, then please bring your proof.

And Allah also created the Afterworld, to Reward and Punish those who follow in His pa and those who rebel against it.

You can enjoy your BBQ and Lunch (Gifts from God). And be thankful to your Creato and prepare for the Afterworld, (where you have to go, you like it or not)

132 of 277

Questions and Answers with Non Muslims Vol-8

Q# 127

Hereafter

[QUOTE who="Frijoles"] 01 Its also the dominant view of the west.

02. But I would take it further and posit that most religions that aggressively proselytize tribal peoples to create universal ecumenical movements - Christianity an Islam as examples - are inherently imperialistic as they roll over and dominate the local religious cultures - many of which were valid spiritual paths in their own right before they got squashed. That was the reason I followed up on the definitions - so you would see this more clearly. 03. Muq, you do understand that Judaism is not a religion that is based on the hereafter?

The stress in Judaism (whether you agree or value this) is in the present - it is similar to Bhuddism in this respect. Our concepts of heaven and hell are not religious canon more like folk belief - we believe that right action will help you in the hereafter but w do not live for the hereafter. Nor do we believe we as humans are born in sin. So salvation is not release from sin (in other words we do not believe that there is any reason for us to be "saved", as we havent done anything bad to begin with). 04. Salvation is more a sense of closeness to God. I think Islam sees this differently. [/QUOTE] Ans. 01. As I said earlier, truth and falsehood are not to be decided on the basis of Majority or Minority view. Truth shall always remain truth, even if ALL humans in the world reject it. So I am not impressed by what is dominant view in the west and in my view that view the result of most of their problems. They are a society which is rudderless, they do not know what are their goals and where they are giving. And let me say that Jews and their Philosophers have a very big part is making that a Dominant view in the west!

02. I do not know what you mean by aggressive proselytization . We should preach our message to every one and then leave them free choice to accept or reject it. This is the most normal and practical approach to this issue.
133 of 277

Questions and Answers with Non Muslims Vol-8

Q# 128

Hijab

[QUOTE who="CTB"] 01. You are one fcking sick psychopath!

02. You know I didn't support the ban on the burqa. But no matter, it doesn't hide the fact that most of your Islamic countries ban practically everything that goes against your statu quo of Islam. 03. It isn't as if women can't practice their faith freely or not wear their little headscarves a little country called France. Don't like it don't move there. Plenty of Western countries that do accept the burqas including the US. Can't say the same for Saudi Arabia.

04. You people and your ridiculous excuses for executing "apostates" and making women cover themselves like tents. You know at first I thought you were not as bad as people made you out to be. In fact you're even worse than some of the people who shout death to this and death to that. You disguise your repugnant arguments by making them as if they weren't your opinions while sneaking them in here and there.

05. Men and women have oppressed each other for centuries so it isn't just a "woman" thing. Elizabeth the first and Catherine the Great imo were better leaders than some of the male folk. You like your wife are just scared little insecure people who can't handle not having control over everyone else's life. [/QUOTE] Ans.

01. Swearing at me and using abusive words only shows that it is you who are hateful and spiteful and not me. We could have differences but to going "off your handle" every time I say some thing which goes against your belief is a sign of weak minds. I do not go "off my handle" for all what you write about me and against Islam. I use sarcastic and rhetoric words some time and that is my limit. 02. How do I know you did not support ban on Burqa, I commented on post of another lady and not you.

Banning anything is not the choice, how come a Muslim minority in Europe "force" their women to wear Hijab and Burqa against their wishes.

This is basic flaw in your arguments, you people can never think that there are systems an laws better and different from your systems and people do accept them from their own free will. You people think that Islam is all full of threats and swearing and every man and woman
134 of 277

Questions and Answers with Non Muslims Vol-8

Q# 129

Hijab

[QUOTE who="Some One"]

Muq it is Canada where the women can sit topless and bottomless on your lap but still yo are allowed to touch certain parts of body .and still man can is limited to do certain things .I have seen belly dances in Dubai where the women is in cloth things get out Control .if someone wants to walk topless it is there choice .btw it is good for eyes o a hot summer day . [/QUOTE] Ans.

I know it is Canada where people can go topless and bottom less..then why should you complain about women who choose to wear Islamic Hijab? Why all this hate propaganda against them? They should also get their choice to wear whatever they like.

If other women have choice to display for public eyes, which ever portion of their bodies they want to show, why not the same right to women to coverup which ever part of their body they want to cover? Why this double standard?

135 of 277

Questions and Answers with Non Muslims Vol-8

Q# 130

Hijab

[QUOTE who="Someone"] There is equal rights in Canada who said she is not allowed to wear a Hijab .who cares what bunch of racist say .sorry it is not France .they are having some problems in Montreal with full face cover . But other than that Canada is welcoming people from all cultures . We share border with USA there might be chance that some people might have American side effect disease . Any way did Mubarak to you and your family .take it easy go and focus on your prayer .we will wait for you we are not going anywhere . [/QUOTE] Ans.

If I am not mistaken, you be the same Sikh person who was discussing about Sikhism wit me. The fight which is going on in Europe against Islamic Hijab is soon to reach US and Canada also.

How can they be remain unaffected of the Virus which has inflicted their cousins on th other side of the Atlantic

After all US and Canada are equal partners with NATO and European nations to Civilize those Barbarians in Afghanistan, Iraq, Libya, Syria, Yemen, Iran, Pakista etc. They also have to catch the virus sooner or later.

That is why I am posting these views to forewarn people in US and Canada that when the take so neutral stand against topless, bottomless and nudity.why should they put restriction on Islamic Hijab. What is needed is that no one should be FORCED to wear any dress.

In the case of Hijab it is pre-decided that no women shall wear it untill forced to wear i And that is the wrong concept and should be removed from people mind. What Mubarak has to do with this debate? He will be punished like any dictator who
136 of 277

Questions and Answers with Non Muslims Vol-8

Q# 131

Hijab

[QUOTE who="Liam-R"]

There is NOTHING inherently good or moral about wearing a burqa, nor is there anythin inherently evil or immoral about wearing a bikini or miniskirt - or even wearing nothing a all. The morality comes not from what the woman is wearing, but from how YOU choose to react to what she is wearing. If you see a woman in a bikini and your thoughts turn to rape, that is YOU being immoral. [/QUOTE] Ans.

You are right, wearing Burqa or dressing modestly are not the ultimate good in themselve What is really needed is the piety and the feelings inside the heart.

But when you make rules for the society, you make them in such a way, so as to cause lea temptation for the average member of the society. That is the basis on which rules are made. Islam keeps a balance between the two, is asks male and females to dress modestly and lower their gazes and at the same time says that the real piety is the piety of heart.

And since no man can know what is inside any ones heart (Only God knows about that) that is why the rules are made for outside appearances only.

It is only God who knows the secrets of heart and that is the basis on which He shall judg every one of us. But we cannot disregard the outward dress so that weaker members of society are not tempted beyond their capacity. That is the basis of Islamic laws, to remove temptations as much as possible.

137 of 277

Questions and Answers with Non Muslims Vol-8

Q# 132

Hijab

[QUOTE who="Frijoles"] Its not about you, its not about me, its about the woman's own right to choose.

What happens in Saudi Arabia or Iran if a woman decides to not wear her Burka in public Thought so. [/QUOTE] Ans.

Why you worry about Saudi Arabia or Iran, why do you not consider looking in your own backyard and ask following questions: a. What would happen if a Women comes to a Public place in Burqa, in France and Denmark and Holland and other European countries? b. What would happen if a women comes to a public place in these countries in Bikini or Micro skirt? How many women and civil right groups would come to the aid of Burqa clad woman versus the Bikini Clad woman? The problem is in your society, where you are free to do any type of evil things and immoral things in the guise of freedom of choice but where the avenues to do good and lead a moral life are blocked.

138 of 277

Questions and Answers with Non Muslims Vol-8

Q# 133

Hijab

[QUOTE who="CTB"] 01. You are one fcking sick psychopath!

02. You know I didn't support the ban on the burqa. But no matter, it doesn't hide the fact that most of your Islamic countries ban practically everything that goes against your status quo of Islam. 03. It isn't as if women can't practice their faith freely or not wear their little headscarves a little country called France. Don't like it don't move there. Plenty of Western countries that do accept the burqas including the US. Can't say the same for Saudi Arabia.

04. You people and your ridiculous excuses for executing "apostates" and making women cover themselves like tents. You know at first I thought you were not as bad as peopl made you out to be. In fact you're even worse than some of the people who shout dea to this and death to that. You disguise your repugnant arguments by making them as they weren't your opinions while sneaking them in here and there.

05. Men and women have oppressed each other for centuries so it isn't just a "woman" thing. Elizabeth the first and Catherine the Great imo were better leaders than some o their male folk. You like your wife are just scared little insecure people who can't handle not having control over everyone else's life. [/QUOTE] Ans.

01. Swearing at me and using abusive words only shows that it is you who are hateful and spiteful and not me. We could have differences but to going "off your handle" every time I say some thing which goes against your belief is a sign of weak minds. I do not go "off my handle" for all what you write about me and against Islam. I use sarcastic and rhetoric words some time and that is my limit. 02. How do I know you did not support ban on Burqa, I commented on post of another lady and not you.

Banning anything is not the choice, how come a Muslim minority in Europe "force" their women to wear Hijab and Burqa against their wishes.

This is basic flaw in your arguments, you people can never think that there are systems an laws better and different from your systems and people do accept them from their own free will.

139 of 277

Questions and Answers with Non Muslims Vol-8

Q# 134

Human Service

[QUOTE who="Bill"]

If Islam is so good answer me this, What good does Islam do? Do they help with disaste like Haiti or Japan? Is there an Islamic Red Cross? Do they help to feed the starving? D they have orphanages? I would really like to know.

[/QUOTE] Ans. Muslims are supposed to help every one in disasters. There is no sayings in Quran or Sayings of prophet which goes against it. There is Islamic "Red Crescent" society in almost every Muslim country. Their job is similar to that of "Christian" Red Cross. There are Orphanages in almost every Muslim city.I think your knowldge and assumptions about Muslim world is very limited.

140 of 277

Questions and Answers with Non Muslims Vol-8

Q# 135

Hygiene

[QUOTE who="FEN"]

MUQ, is too late for you now, u can't come back, u're mutilated, I'm sorry.. but stop doing it on new born babies now. Who gives a sh!t about what Paul thinks about it, I think with my head. So you keep encouraging this practice? C'mon, Allah looks at your good deeds not at your d!ck I can't believe some time I'm on topix debating on how you must treat your d!ck and on how you have to clean after a good sh!t... c'mon. [/QUOTE] Ans.

You can look at it what ever way you want to look. Bodily cleanliness and purification is also important for soul purification and cleanliness. A purified and clean soul lives i a purified and clean body. Human is composed on both physical bodies and spirit and you have to take care of both of them.

Islam is a complete religion, so it insists on purification and cleanliness at all levels. You should have clean body, you should eat clean food and you should read clean books and you should have clean thoughts. That is why bathing and WUDU (washing of hands and feet before prayers) and many personal hygiene instructions were given by our prophet and they all are good.

Circumcision is also one such practice which helps in keeping your body parts clean and helps in control of certain disease. However in Islam, this is only a recommended practic not any Dogma. It is not an article of faith and not the one who does not do it should be cutoff from the society (As jewish law says).

Islam is always the middle path between the two extreme. For Jews, according to them Life is not worth living if you are not circumcised and according to people like you wh call it a Mutilation of human body. Islam way is in between, according to our prophet It is the practice of prophets which came before me and it is liked by God. You are right, Allah does not look at our bodies and looksHe looks at our souls, but a Clean Soul resides in a Clean body!! So we are back at square one!!

141 of 277

Questions and Answers with Non Muslims Vol-8

Q# 136

Hygiene

[QUOTE who="TheJazuma"]<quoted text> Ok, i'm not wiggling out of anything. Now, consider this After eating KFC, would you use cold water to wash your mouth? Not only does the oiliness not go away, it spreads around your whole face. Obviously that isn't nice. The same can be said about your deuce dropping, when you have finished dropping deuc you use toilet paper to remove the remnants, then you wash your hands. However, using water to wipe your azz is pretty silly since your hands are in contact with the sh t...now is that hygienic to you? [/QUOTE] 01. The private parts are not "as cleaned" with toilet paper as they could be with water....this is common sense. Any one washing his parts with water after toilet is more hygienic and less smelling than who uses only toilet paper.

01A And THEN, we wash our hands with soap.....so every thing is fine... and why YOU people wash your hands with soap when your hands did not came in contact with the sh*t?

01 B It is like some one said "they use buckets of waters to clean the toilet bowl, but not a single drop on the body". And yet, they continue to argue that they are more hygienic!!

02. The Islamic requirement to wash your hands and feet and face five times a day (befor prayers) is a very hygienic practice that no one can question.

03. Islamic requirement to have a bath at least on Fridays (at least once in a week) and af sexual intercourse is a requirement which no one can find fault with (And there is no a single word written in any of NT books about personal hygiene). 04. Islamic requirement for women to have bath after their monthly period is over is a hygienic practice that no one can find fault with. 05. Islamic guideline to clean your teeth with MISWAK (or tooth brush) before each prayer is a hygienic practice that no one can find fault with. 06. Islamic guidelines for people to remove their public hairs every week (40 days maximum period) is a hygienic practice that no one can find fault with. 07. Islamic (and Jewish too) guideline to have circumcision of male child is a very hygienic practice, no one can find fault with. And yet, people like you want to teach us as how to take care of personal hygiene.!!

142 of 277

Questions and Answers with Non Muslims Vol-8

Q# 137

Infidel

[QUOTE who="Stephmar"] 01. Islam helps itself ;at the expense of the free world. They think they are fooling everyone. Sorry, no one fools this old fool anymore.

02. Always hear people use this term 'infidel .'Do you know who the real infidels are? Th are the very people who consider other people infidels. These http://www.topix.net/forum/world/TUNQFNMODCAE49UNJ/p23#lastPost [/QUOTE] Ans.

01. I do not know what you mean by this term "That Islam helps itself at the expense of free world". The reality is Islam is suffering maximum damage at the hands of the so called free world.

The free world has attacked and conquered Iraq, Afghanistan, now they are attacking Lib and Syria, Yemen, Iran are next targets. You are certainly a fool if you think that Western People are really helping Islam and Muslims.

02. The term Infidel is used for the one who does not believe in one' religion. It may also mean, one who does not believe in God.

We are not encouraged to call any individual person as "infidel". Because our prophet sa "If one Muslim addresses another Muslim as "Infidel" then this world proves true on one them. If that Muslim was really infidel, then it stays, otherwise if he was not infidel, then the caller becomes infidel".

So we should be careful in our posts and talks to label people as infidels without thinking

143 of 277

Questions and Answers with Non Muslims Vol-8

Q# 138

Intelligenc [QUOTE who="Frijoles"] e 1. A most interesting argument. Deficiency by default. Thats like saying Islam does not address the question of personal intelligence therefore it does not prepare its practitioners for being intelligent in this world.... [/QUOTE] Islam has very high regards for human intelligence. At many places Quran asks us to reflect and Ponder and analyze the working of Universe and every thing in it.

But there are issues where human intelligence cannot reach, or where humans are not abl to judge between good and harmful results unless after lot of hit and trials. That is why, our Creator in His infinite mercy told us the best possible course Himself. These are the directions we find mentioned in Quran and Prophetic sayings. None of them are random and not based on any reason, logic or proven benefits. It would be childish, if we just disregard these instructions and try to find our own way from scratch and fall into so many potholes on the way. It would be making our life and our task difficult, which Allah and His messenger have made easy. As a puzzle:

What are your opinion about effect of Alcohol on human society? What our Super Governments and Our Super Philosophers have done to remove this evil? And what have been their success rate?

Now compare that with a Single verse of Quran banning Alcohol, Gambling, Fortune telling and Sacrificing to idols haw effective that single verse of Quran has been against so much efforts and propaganda of your Super Governments? Muslims constitute the world biggest group who do not imbibe alcohol!! Was Allah dictatoria or a Wise Doctor and Well Wisher of Humans?

144 of 277

Questions and Answers with Non Muslims Vol-8

Q# 139

Is it from God?

[QUOTE who="JAN"] First 10 reasons Islam is not from God. [/QUOTE] Ans.

-JAN Reason #1:

Islam cannot be from God because the prophet of Islam, Muhammad, did not pass God's first test for a prophet. Deuteronomy 18:21-22 tells us that God will authentica those He sends as a prophet by having them perform signs. Muhammad gave no sign as required by God. MUQ:

a. Miracles are like Toys, you give it to little children when they are growing up. But onc they are grown up, they do not need those toys. b. The so called Miracles of OT and NT prophets are for the period when the Human civilization was in its initial stages and people needed some corroborative proofs that the so called prophets are from God.

c. There are many prophets in OT and NT books, who showed no miracles, still they are genuine prophets. For Example John the Baptist, he showed no Miracle, can we say h was a false prophet? d. And islam Provided the Ultimate Miracle, the Quran! A Miracle to beat all other Miracles!! All miracles of OT and NT prophets are only to be found in the pages of books, while Quran is a living miracle. A book which no one can match in its eloquence, style and truth!!

- JAN Reason #2:

Islam cannot be from God because Muhammad's teaching does not pass the second test God gave for a prophet. Deuteronomy 13:1-4 tells us that no prophet will bring a revelation that is in conflict with previous revelation. His revelation must agree with the previous revelation of God. Deuteronomy 13:1-4
145 of 277

Questions and Answers with Non Muslims Vol-8

Q# 140

Its Death

[QUOTE who=" "] Predicting the death of Islam The Crisis of Islamic Civilization by Ali A Allaw [/QUOTE] Ans. Predicting Death of Islam is a favorite game and pastime of people who do not like it to survive . And this game is more than 1400 years old!!

1. The first people to predict death of Islam were the pagans of Makkah who predicted th it will die when the religious intoxication of its followers was finished.. well, they were wrong and it was they who vanished from the scene. 2. Then it were Jews and Hypocrites of Madina who predicted that Islam will be dead, if tribes of Arabia attacked Madina jointly.. well, they were wrong and it were they who vanished from the scene.

3. Then it were the Two Superpowers of that time, Mighty Persian Empire and Mighty Roman Empire who predicted death of Islam., if its followers ever dreamt to pick up fight with any of them.Well, they were proven wrong and in a short span of 30 odd years , Muslims took on both of them and it was they who vanished from the scene. 4. Then it were the Crusaders who predicted the death of Islam, when Mighty Christian armies of Europe (supported by God, Son and Holy Ghost) landed on the shores of holy land.Well, it were the Crusaders who were Gone With the Wind, leaving no trace behind.

5. Then most people predicted death of Islam once the Cruel and Brutal Tatars attacked Muslim nation, it appeared so in the beginning.. but within a century it was Tatars who became Muslims and were defenders of islam for next few centuries.

6. Then many people predicted death of Islam, once most of Muslim countries came unde colonial rule of European nations. Well Islam survived and it was these European nations who had to vacate those Muslim countries willingly or unwillingly.

So I have no doubt that the current prediction about death of islam would meet with the same end as those previous before it (just like those 500 dates for Jesus return to ear and End of World predictions). One may ask me, why I am so sure. I am sure because of One Verse of Quran (actually repeated three times).

It is He (i.e. Allah) who sent His Messenger (i.e. Mohammad) with Guidance (i.e. Quran
146 of 277

Questions and Answers with Non Muslims Vol-8

Chapter-7 Islam-3

147 of 277

Questions and Answers with Non Muslims Vol-8

148 of 277

Questions and Answers with Non Muslims Vol-8

Q# 141

Jesus

[QUOTE who="Zeal"] my brother, i respect your point of view on jesus based in the holy book. but what matters me there are two verses in the quran seemed very hard to reconcile the two events of where jesus story has been originated. we are inside the Qurans jurisdiction of truth. so we must base our conclusions based in the Qurans point of view and nothing more. in Al Nisaa 4:157-158 states that jesus was not the one who was crucified on the cross.

so it could be that the man on the cross has been a god of the RCC instead of the right on based in the Qurans revelations, right?

so from the quoted passages who was this man talking to Allah in Surah Al Maidah on th basis of worshipping him(jesus)? do the two verses have a unified scene on Christ? [/QUOTE] Ans. I am thankful for presenting your observations in most civil manner. A. Quranic claim: Quran in Nisaa 157-158 is in answer to the Jews Boast that We killed the Christ Jesus, Son of Mary, a messenger of God Before that verse Quran speaks of other crimes committed by Jews thru the ages.. But in answer to this charge, Quran says They killed him not neither crucified him, but it appeared to them so And Quran further adds

Those who dispute regarding this (crucifixion or killing of Jesus) have no knowledge an they only speak of conjecture And to complete the statement

But they never killed him, and God took him to Himself , for Allah is All Powerful Wise Quran does not engage in long discussions and arguments, it uses limited words,
149 of 277

Questions and Answers with Non Muslims Vol-8

Q# 142

Jesus

[QUOTE who="zeal"]<quoted text> yes BMZ, im happy that you have finally come to my term. but here we are discussing jesus based in the quran's point of view that jesus wasn't crucified on the cross. so we can only draw the conclusions as long as we use the truth from the quran and not from the bible's revelation, right? now based in the quran we found out that the RCC had worshipped the wrong jesus on th cross, will you agree that the Moslem people accused wrongfully the RCC of Christ being worshipped? [/QUOTE] Ans. You are right in a sense, we have two conflicting sources for information on Crucifixion Jesus, Quranic and Biblical.

Quran is revelation from God Almighty, Who was Witnessing the Scene as it happened and passed on His Information to His Chosen Messenger 600 years after "As it happened

Biblical version is based on hearsay report of four persons, who did not witness the scene with their own eyes and compiled from what they "heard" others speaking in the street. N two versions are exactly like each other and they have also been "touched up and modified" by some "unknown hands and pens" over the years. I do not know, As a legal person, as a logical person, or as a reasonable person, which of the two sources you will take as "more authentic"?

One coming from God Almighty who Saw and Knew every thing....coming after 600 yea of the event...and the other "confused and conflicting" records of four people who took down what they learnt from hearsay.

You make the decision and stick to it, It is OK for us, we do not want to FORCE our view on any one!!

150 of 277

Questions and Answers with Non Muslims Vol-8

Q# 143

Jesus

[QUOTE who="SS"] 01. Poor Muq. Doesnt even believe in a second coming of Christ.

02. But a man DID bring forth the Quran in the first place, did he not, Muq? Unless you d not believe that Quran was dictated to Muhammad word-for-word for him to record. Which is Muq?

You either DO believe that the Quran was dictated to Muhammad to record which includ the return of both Jesus and Muhammad or you do NOT believe that the Quran was dictated word-for-word from God's mouth to Muhammads ear. You either DO belie that Mohammed was revealed the Quran and then distributed it or you do NOT belie that. Which is it?

03. (by the way, excellent work on following Harold Camping. If more Christians follow such chicanery as closely, a lot less damage would be done to individual lives.) But which is it?

04. Either Muhammad was a liar, or he was telling the truth. I can't believe you espouse t Hadith, by the way. If there is anything anywhere that has done more damage to the reputation of Islam in the world, it has been the Hadith which are completely contradictory to almost every word that was ever printed in the Quran. [/QUOTE] Ans. 01. Who said Muslims do not believe in second coming of Jesus Christ.

The truth is Islam is the ONLY Non Christian faith of the face of earth which as its artic of faith believes in Jesus as a prophet of God and in his second coming. However Muslims reject this buffoonery which is played every few years by predicting that Jesus will return on such and such date.which is a favorite past time of many Christians!! 02. Are you not aware how Quran was revealed and composed?

Quran was revealed directly to prophet by the Archangel Gabriel, and he memorized the wordings. When Gabriel left the prophet would call one of his many scribes (becaus prophet himself could not read or write) to write down the latest revelation.

He would ensure that the writing was correct by asking the scribe to read it back to him, s that to ensure that he made no mistake in writing it. Such was the care and precaution by which whole Quran was preserved in written form
151 of 277

Questions and Answers with Non Muslims Vol-8

Q# 144

Jesus

[QUOTE who="Karzai"] 01. Brother muq, did muhammad ever meet jesus? 02. With regard to planners of 9-11, have you asked the Taliban if it was wise to aid and support the 9-11 planners UBL and Al Qaeda?

03. With regard to your comment 2, followers changed the message and divided into sect and denominations is a well written clarification of where Islam is today don't you think? 2

04. Brother muq, since you posted this story I was wondering your opinion on this part, d you view the afterworld as physical sensual earthly pleasure place? [/QUOTE] Ans.

01. First of all, it was not essential for our prophet to meet Jesus physically. He got all the info that was needed thru the Revelation from God Almighty, who is source of all knowledge.

But according to Islamic traditions, our prophet did meet Jesus physically on that blessed Night of Ascent, when our prophet was taken up to heavens and met many prophets of God, including Adam, Abraham and Moses and Jesus 02. Taliban were no part of 9/11 or any of so called plot. They were implicated into this affair by the press and media. As to why they did not break up with Al Qaeda and Osama after 9/11. They asked for US to provide information and proof that Osama and Al Qaeda were indeed involved in 9/11 plot. But by that time US was riding its highest horse and considered it an insult to talk and show evidence to minnows like Taliban when GWB the Great and Dick Cheney have believed in it like they believed in Gospel. And the rest is history!!

03. Yes it is true for Islam also that Muslims got divided into sects. But the position in islam is different from all other religions, because things in them are so muddled up that i is no longer feasible to know what is the real path. In case of islam, true teachings (in the form of Quran and Sunnah) are still preserved in their original and pristine conditions.

152 of 277

Questions and Answers with Non Muslims Vol-8

Q# 145

Jesus

[QUOTE who="Sophie"]

01. wow, wow, wow, slow your role here friend. I am not trying to attack your post. I AM interested in knowing the old tribes, I am currently studying by my own initiative. I am just saying that sadly OTHERS are probably not going to take this seriously if they see no relevance to it.

02. And as far as the whole Jesus, son of God thing.. I think its important to look at storie like that metaphorically. Jesus was son of God because he was treating people fairly, just fully, he carried out the world of the Lord through example much like a son would fulfill his fathers wishes.

03. I think what the Christians were trying to say is exactly this, but since have shaped in a literal translation because how it has been translated. I mean, if your trying to look at it logically then thats what I would assume you would arrive to? Like, Jesus I would say IS the son of god, but not his physical seed. Jesus was just seen a the only son of god because the time period he was in very few were using religion like it was intended by God /Allah / Hashem, much like today. [/QUOTE] Ans. 01. I did not criticise you for your comment. I just told the fact that it is a comprehensive biography of our prophet, that is all.

I know that most people do not like to read these things, but if some one is interested, the info is available on hand.

02. Regarding Jesus being Son of God, I heard your explanation and that is why I said, I have no problem if Jesus be called a son of God in the sense in which it is used in Jewish language. I.e. some one who is closer to God , a godly person, a Prophet of God etc. But Christians have crossed the limit, they turned Jesus into a Begotten son of God, A God in human form, one in a Trinity with God etc.

They worship him, they pray to him, they make his idols and statues and hang them on wall and worship it, they even worship the cross This is totally against the Monotheisti teachings of Bible and Jesus himself.

On the other hand, Jews have gone to another extreme and they deny jesus and his messa and speak bad words about him.

153 of 277

Questions and Answers with Non Muslims Vol-8

Q# 146

Jesus

[QUOTE who="Walker"] . Muslims Insult Jesus. [/QUOTE] Ans. I would only like to comment on that part of Mr. Walkers post, where he said that Muslims insult Jesus. This is far from truth, in no way Muslims insult Jesus. It is other way around, it is Christians who insult Muslims and do not feel any shame. 1. Muslims give great regards to Jesus and consider him a Mighty Prophet of God.

2. They believe in his miraculous birth, his miracles and that he shall return to the world a the end of world. 3. No Muslim ever takes the name of Jesus in his vernacular language without saying May peace of Allah be on himthe same epithet that they use for prophet Mohammad and each and every Jewish prophet whose names are mentioned in Quran.

4. Name of Jesus in Arabic (ESSA) is an honored name amongst Muslims and we find many Muslims named ESSA thru out Islamic history. And we do not find even a single Arab Jew or Christian taking the name Mohammad, which is not a bad name, going by its meanings!!

5. To my knowledge, No Muslim draws any cartoon or caricature of Jesus and makes fun of him and lampoons him in public (It is Christians themselves who do all these things). They do not even feel that by doing this they are insulting Jesus. 6. To have difference of opinion about theological issues is not insulting:

a. If Muslims deny that Jesus is God or Begotten Son of God or Died for Sins of Humans and deny him as part of Trinity, that is not insulting him.

b. Muslims have theological proofs that all of these are not what jesus taught or preached these are additions into his teachings by other people. c. By engaging in these discussions is not called insulting, it is a part of religious dialog, which even Jesus had with Jews of his time.

7. On the other hand, Christians and Jews not only do not believe in the message of our prophet (which is their choice, by the way), but during any discussions hurl abuses a
154 of 277

Questions and Answers with Non Muslims Vol-8

Q# 147

Jesus

[QUOTE who="Skeptic"] Muslims keep Jesus below their own prophet [/QUOTE] Ans. We degrade no prophet. Every prophet is as honored by us as the other. But as regards following the teachings, you have to follow the one's whose teachings are current.

Let us take an example; do you dishonor George Washington, the First President of USA

Then will you follow the laws passed by George Washington or the Current US president Obama?

Does by following laws enacted by Obama which are against those of George Washingto you degrade Washington? That is exactly the difference between Jesus and our prophet. The time period of mission Jesus ended when the Last and Final prophet was sent by God. So we Muslims revere and respect all prophets of God, and yet we follow the laws and teachings of Last and Final prophet of God.

The people who still insist on following teachings of past prophets, are in fact complainin against God as to why He sent the Last Prophet.

Or , Do they mean to say that Jesus will be happy at their stand? By God, Jesus will be th first to disengage himself from such people....because He did tell his people to watch for that "Another Comforter" whom God will send and who will guide them into all truth and who shall abide with them for ever.

So do not try to put your mistakes and false zeal into the account of Jesus or think that yo are doing a great service to Jesus or Jesus will be happy and pat you on the back for sticking with him....

155 of 277

Questions and Answers with Non Muslims Vol-8

Q# 148

Jews [QUOTE who="Rick Moss "]<quoted text> Your jealousy of the Jews consumes you.[/QUOTE] Ans. Why would we be jealous of Jews, our YOUNGER cousins by EVERY standards? - We outnumber them by many orders of magnitudes

- We control many countries in the world and they only one (that too supported by the sti of their "arch enemies" and now "bosom friends" Christian). - Arabic Language is a major live world language and Hebrew, well its speakers can be counted on fingers like our Younger cousins!!

Muslims permeate every country and every race and every language of the world and Jew only live in limited countries. There is nothing by which we could be jealous of our Younger cousins in any way.

However, there are things in which we are REALLY inferior to our cousins...and these ar -.Racism, - False propaganda, - Usury and controlling other people and nation thru usury, - Mischief, - Intrigue - Promoting Pornography, homosexuality...Loose Morals etc. If you think we are jealous of our younger cousins because of our inabilities to compete with them on these items....we confess

156 of 277

Questions and Answers with Non Muslims Vol-8

Q# 149

Jews

[QUOTE who="THINK"]<quoted text>Muhammad lied and Millions died. own .. The Apostle of Allah, may Allah bless him, returned on Thursday 7 Dhu al-Hijja Then He commanded them to be brought into al-Madinah wheE ditches were dug in the market. The Apostle of Allah, may Allah bless him, sat with his Companions and they were brought in small groups. Their heads were struck off. They were between six hundre and seven hundred in number. [Ibn sa'd, Kitab Al-Tahaqat al-Kabir, vol. 2, p, 93]

On this occasion Muhammad revealed the lack of honesty and moral courage which is an unattractive trait in his character unbecoming of a prophet. Muhammad sat there while each and every male was beheaded unemotionally disturbed. [/QUOTE] Ans.

So according to you, because our prophet sat and watched the beheading of 600 to 700 Jews (The actual figure was between 400 & 500, but that does not matter, a sentence which was imposed on them, according to their own Torah, for their treachery and rebellion and endangering the very survival of Islamic state, when all Muslim were busy in defending Madina from all tribes of Arabia.and those treacherous Jews of BANI QURAIDHA broke their treaty unilaterally and threatened women and childre and old people of Muslim community).disqualifies from him being a true prophet God

Then let us see How Moses watched killing of 3000 Jews in one day, for their crime of worshipping Golden Calf:

Then Moses stood up at the gate of the camp and said All those for Yahweh come to m And all the sons of Levi rallied around him,. Then he said to them This is what Yahweh, the God of Israel commands: Let each one carry a sword at his side. Go back and forth from door to door and do not hesitate to kill even your brothers, your companions and your relatives The Levites did what Moses had ordered and that day about 3000 men fell

Moses then said From now on your hands are consecrated to Yahweh for each of you h been able to turn against his very sons and brothers, because of this Yahweh gives yo today His blessings.. And so Yahweh punished the people with a plague because of calf Aaron had made for them (Ex. 32: 26-35).

So Moses killing 3000 Jews in one day, still remains a Prophet of God and these people who took part in killing are blessed by God and God kills many more Jews by plague .for false worship..all this is acceptable.but our prophet punishment for Jews for treachery and rebellion and endangering security of Nation for ever remove his name from being a true prophet.
157 of 277

Questions and Answers with Non Muslims Vol-8

Q# 150

Jihad

[QUOTE who="Buford"] Offensive and defensive Jihad [/QUOTE] Ans.

Now some people have created a new controversy that Islamic Jihad is of defensive natur (i.e. protect your self when some one attacks you) and not of offensive nature (strike first when you see the enemies are about to strike you). The reality is, there is no such distinction in the Islamic concept of Jihad and there are situations where both defensive and pre-emptive actions are needed. And this comes because many people (both Muslims as well as non Muslims) do not properly understand the meaning and scope of Islamic Jihad. So let us throw some light on the subject of Islamic Jihad. A. What the word Jihad means:

In Arabic the word Jihad means to struggle or to strive. Every one in the word is engaged in some type of Jihad one way or another. Some people are doing Jihad to earn their living, some for getting the education, some to construct their houses and some to establish their business. B. What is Islamic Jihad:

In Islamic Jihad the word Jihad is followed by the Arabic word FI SABEEL AL ALLAH i.e. striving in the path of Allah. In Quranic words to Keep up the name of Allah and to keep down the name of falsehood. Muslims are therefore not doing Islam Jihad for: a. Just expanding their kingdoms. b. Just looting their neighbours and exploiting their material and economic wealth. c. Not to show off their bravery and boast of their physical power. d. Not to create oppression, mischief, tyranny and injustice in the world. e. To settle personal score against particular person as a personal vendetta.

158 of 277

Questions and Answers with Non Muslims Vol-8

Q# 151

Jinn

[QUOTE who="Frijoles"]

I am not well versed in Jinns as described in the Islamic Literature. I do not believe in Santa Clause, however I am convinced that the Tooth Fairy exists. I do prefer my jin extra dry, and my Jen with legs spread wide. [/QUOTE] Ans. [QUOTE who="Liam-R"]

Well, while we have no clue as to the nature of life on other planets, I'd say that it is a VERY safe bet to state that there is no such thing as the jinns as described in islamic myth living on this Earth. [/QUOTE] Ans. To: Liam-R and Frijoles I think both of you have Jewish backgrounds and are well up in your studies of both OT and NT books.

If such is the case, how can you people say that the word Jinn is totally non heard by you The basic concept of Quran is same as in OT and NT books. In this world there are creations whom the science has not yet discovered and those who are beyond the reach and scope of our modern science. As per Quran there are three categories of intelligent creatures in this Universe: A. Man whose origin is from dirt (his whole body parts come from the earth except his soul, which is not made from any physical material, so no scientific instrument can detect it)

B. Angels who are created from Light, they also have a body and soul. But different from humans.

C. Jinns, who are created from Fire, they also have a body and a soul. But their body is no made of any material, so they cannot be seen by our physical eyes.

Jinns share the planet earth with humans, but they normally do not interfere with humans
159 of 277

Questions and Answers with Non Muslims Vol-8

Q# 152

Jihad

[QUOTE who="Palwa"]

Name one time or place where the enactment of jihad hasn't killed people. It's very easily understood when you look at the facts compared to your insane claims. All Muslims are primitive morons and along with their Jewish, Christian and Hindu /Buddhist ilk, can never fit in a sane society; all they do is cause trouble and war. [/QUOTE] Ans. What you said is based on your poor knowledge of Islamic Jihad and you just believe in whatever you read in media. Islamic Jihad has many stages. Jihad literally means to struggle and strive in the cause of Allah.

The first and most difficult is the Jihad, which a Muslim does against his or her own self. They controls their passions and thoughts and actions against every thing which tries to go against their beliefs and commands of Allah.

The next stage is Jihad with tongue, where Muslims criticize and try to stop wrong things and evil things present in the world.

The next stage is Jihad with pen, in which Muslims use their pen to stop the evil thoughts and evil philosophies and evil systems that are prevalent in the world. When all these stages are passed, then comes the matter of Jihad with Sword. When every thing else fails and there is no recourse but to take arms and fight the evil, the Muslims do not shy from it. But all these are different shades of same struggle and striving against evil.

But it is media which presents any fight by Muslims as Jihad and they try to malign Islam

When Western people wage war to protect Democracy, Human Rights and American Dreams.. this is also Jihad, but it is Jihad for these ideals and not Jihad in the cause of Allah.

160 of 277

Questions and Answers with Non Muslims Vol-8

Q# 153

Kaaba

[QUOTE who="wadget"]

1. are you mistaking me for hindu? atheists believe i'm christian, some christians mistake me for Buddhist and i giggle to myself because all these people believe that the ONE Uncreated Creator is an exclusively practising <place your religion of choice here>.

2. what is idolatry, if not the circling of the ka'aba? what is idolatry, if not praying toward mecca? i would think those who venerated the ONE would pray towards Alla or is Allah entrapped in Mecca? in the south we have an old black spiritual song that goes .. lost in a snow filled sky who do you love when you come undone [/QUOTE] Ans.

1. I think I did not comment on which religion you belong. My answer was for the points you mentioned. You may giggle or laugh yourself to death seeing the stupidity of religious people it is neither here nor there.

But let me tell you, all smiles and laughs and giggles would be removed fro your face, the moment you die and you face the REALITY. So giggle away, I do not mind.

2. To say that Going around Kaaba or praying towards it idolatry is the summit of ignorance. I could hardly believe that a giggling person like you made this origin discovery!!

Have you ever thought what Muslims say when they go around Kaaba? To whom they address their supplications? To Kaaba/ To Black stone? Or to the Lord of Kaaba an Lord of Black Stone and the Lord of Universe?

When Muslims pray towards Kaaba from every where in the world, they show a purpose unity. Otherwise if you give people choice, every one will face in different direction and there will be total chaos. Our Lord wants us to be united in purpose and goal. We turn our face towards Kaaba , as it was the First House that was dedicated to the worship of True Lord and God of Universe. Which south you are talking about? South of India or South of USA? See I do not even know which nation you belong to, how could I have guessed your religion!!
161 of 277

Questions and Answers with Non Muslims Vol-8

Q# 154

Kafir

[QUOTE who="Real Evil"] 01. hey muq can i get the status of kafir??? 02. will u send me to islamic hell for being a kafir??? 03. will i get any (w)hores in hell???? [/QUOTE] Ans.

01. I do not make or give any one status of KAFIR. If you think that message of Islam ha reached you and you have understood it, and then you want to reject it with your hea and with your tongue..then you have made YOURSELF a Kafir. A. Fundamentals of Islam: In case the basic message of Islam has not reached you, I would like to rehearse it to you: Islam consist of Articles of Faiths and Basic do s and dont s. let is see them one by one: a. Unity of Deity:

This is the most important fundamental of Islam. To believe that there is only One Creato in this Universe. His name is Allah in Arabic, but He is known by many names in differe languages.

He is absolutely unique and there is no partner in any of His Divine attributes. He creates He cherishes and He males people to die.

To worship a false god or to associate any one in the Divine Worship is the Biggest of the Big sins and it makes anyone doing that worthy of Hell Fire, unless he repents and correc his ways b. Angels of God:

One has also to believe in angels of God. These are special creatures of Allah, who do various duties in this Universe. They have no will of their own, but they only do what the are told. They are made from Light (NOOR) and have no male or female sex and they should not be worshipped. c. Prophets of God:

To provide guidance to human kind, Allah chooses certain human from amongst the peop to deliver His message and present himself as a working model.

162 of 277

Questions and Answers with Non Muslims Vol-8

Q# 155

kafir

[QUOTE who="dabang"] Muq, 01. Understood the definition of NON- MUSLIM & KAFIR. 02.What you call those Muslims who kill and terrorize innocent ? Muq, You said: 03. So Kafir is a Non Muslim. Non-Muslim is not Kafir.(What you call non-muslim in Urdu or Arabic) [/QUOTE] Ans. 01. That is very nice if you understood the definition and difference between a Kafir and Non Muslim.

02. If any Muslim kills has killed any innocent man women or child, he has committed on of the Great Sin in Islam, and his punishment is very severe in the hereafter.

If anyone kills a soul for other reason than killing some one or for creating mischief in the land, it is as if he has killed the whole humanity.. (5:35) And punishment for killing a Muslim is even higher, as Quran says: If a (Muslim) man kills a believer intentionally, his recompense is Hell Fire, to abide therein (for ever); and the Wrath and the Curse of Allah shall be upon him and a Great punishment is prepared for him (4:93) 04. Every Kafir is also a Non Muslim , but every Non Muslim is not a Kafir.

In Urdu and Arabic the word for Non Muslim is GHAIR AL MUSLIMEEN i.e. One wh is Not a Muslim. In Sanskrit the world for some one who is Not Hindu or not Arya is I think MLECSHA.

163 of 277

Questions and Answers with Non Muslims Vol-8

Q# 156

Killing

[QUOTE who="water_nymph"]<quoted text>One has to research no further than knowin your religion promotes the stoning of women and children to know it's no more than man-made religion that a moral person would not follow. No omnipotent God would have to have humans kill other humans for him. [/QUOTE] Ans. Any one can do any research to satisfy one self. In Islam nothing is hidden every thing is open.

As for stoning married men and women for committing adultery, the Islamic punishment same as what is mentioned in Bible.

I do not know by whose authority they abolished it, in Islam no one has authority to modify or change what Allah and His messenger has decreed specifically. This is th strongest point in favor of Islam and not against it.

If you claim to be "More merciful" than Allah, the one who Creates, and Cherishes all lif on this earth.....then it could be your own position. What "mercy mission" these US and NATO forces are doing in Afghanistan, Iraq and Libya? I think killing women and children and men by Missiles, Bombs, Drones, Chemical weapons, Biological weapons...and every other means is OK. They should not be killed by sword and by stones for committing the crime....that seems be your basic objection. Is it OK.

And what is this "Modern religion" which allows killing on innocent people by this way?

164 of 277

Questions and Answers with Non Muslims Vol-8

Q# 157

laws

[QUOTE who="Scarab"]

01. Most intelligent and informed people are in full agreement that past religions had atrocious and cruel practices. Few religions today continue those practices and Islam is no and was not the great liberator.

02. If Allah doesn't believe in bring discomfort to the body why they does Islam inflict th veil and burka's on women? there s nothing more annoying then having a scarf wrapped around your head.

03. The Muslim banning of eating pork is equally an archaic carry over from the past. Pig were considered destructive to other plant life because they dig to and eat the roots of plants thus destructive to agriculture in areas with limited agricultural areas.

04. Of course you don't want to enter into this type of argument with Brother Mahmood.

05. I note that every Muslim participating in Hajj is required to "buy" an animal to sacrifi to Allah. This is certainly a lucrative financial boon to the butchers approved by the Saud at their yearly (guilt enforced) Bazaar at Mecca. [/QUOTE] Ans.

You have become your own prosecutor, judge and Jury in your case against me and Islam You frame charges, judge it and gives your decision without waiting for response . 01. Most religions had atrocious and cruel practices:

This could be true for actions of the followers of these religions. To my knowledge NON of the revealed religions preached hate, hatred and violence against each other. These religions preached the same messages to their fellow country folks. 02. Islamic Hijab for women.

You are wrong in drawing the worst conclusions for Islamic Hijab. And you are wrong th Islam only provides Hijab for women only. In Quran Hijab and dress code for both men and women is mentioned.
165 of 277

Questions and Answers with Non Muslims Vol-8

Q# 158

Love

[QUOTE who="Chuy"] Well you got me "MUQ" I'm not sure how to answer ...The Greeks had several definitions ...Saint Paul writes his understanding of Love ..I think you may have read it already (Love is kind ,love is gentle ,love is not jealous ..etc.etc.) God loves his children and still he will chastise us . I have to concede to your argument . [/QUOTE] Ans. There is nothing to feel bad about, actually love is an emotion and it is not possible to define any emotion. That is why there is no fixed definition of love and how to measure it. One test of love is see how much one is ready to sacrifice or suffer for whom he loves.

That is why coming to the neighbours, islam has fixed the criterion to be nice and be kind to ones neighbors and do not say or do any thing to annoy or hurt him. Our prophet said he is not a Muslim who has his stomach full while his neighbors has nothing to eat And prophet said he is not a Muslim whose neighbors are not safe from his evil And prophet said If someone beliefs in Allah and the last day, he should nor hurt his neighbor and should honor his guests

And prophet said Gabriel kept on reminding me about neighbors till I thought he will be included in the list of heirs.

There are many such sayings about neighbours in the Books of Hadith . And they all ar in line with the command Love thy neighbor!! PS:

Gods Love is not blind (like most of us humans have). Since He knows what is inside our hearts, His love and His Justice act according to the state of our hearts.

His love and mercy is for those who believe in His message and try their best to follow th path of His messengers and prophets .

While His Justice is for those who willfully reject His Prophets and Messengers and try to
166 of 277

Questions and Answers with Non Muslims Vol-8

Q# 159

Love

[QUOTE who="Chuy"]

Hi.. "MUQ" I'm very impressed with you and your responses to personal attacks ...you st polite.! I don't think it is easy to see ...how others view Islam.

This post reads as though muslims are bogged down with practices and minutiae..(very much like Judaism). The reason we do "not" study every aspect of our Lords life.. He made it simple.. "Love t Lord God with all your heart and with all your soul and with all your mind and with all your strength ,The second is this 'love your neighbor as yourself .There is no commandment greater than these . [/QUOTE] Ans. I thank you for your kind words. People do not see real Islam mainly because Muslims themselves do not present themselves as good examples of real Islam.

Islam is not only saying and preaching , it is a practical religion that has to be practiced a used on the ground. And Muslims are just not doing that. See your own perception that Islam is bogged down in rituals like Judaism, which is not correct. In Islam rituals have their peace and so have the other sections of human life. The two commandments you have Quoted from Jesus 'To love your Lord and be good to your neighbor" are also to be found in Islam. But on a Much Better Scale!! Listen to this Single Quranic Verse: "Your Lord has declared, that you shall worship none but Him and (you) shall be kind to your parents, and your kinfolks, and your neighbors (those who are related to you and those who are not) and companions of your travels and way fearers and those (slaves) whom your right hand possess."

See how deep and comprehensive is this Quranic verse and compare it with that comman of Jesus.

If Muslims act only on this single verse of Quran, the whole perceptions of Non Muslims about Islam will change.

Now they feel alarmed when any Muslim moves into their neighborhood, then they woul
167 of 277

Questions and Answers with Non Muslims Vol-8

Q# 160

Makkah

The Reason I was absent from this thread for last few days was that I had gone for UMRAH (minor Pilgrimage) to Makkah and ZIYARAH (Visit) to Prophet's City Madina, what our "Ignorant friends" might call PUJA and DARSHAN of Kaaba and Black stone!

I tried to find out, very objectively, how much this PUJA resembles and is different from what we normally see amongst Hindus in India and other parts of the world. I. UMRAH at Makkah: 1. What I did not see:

a. I did not see any Coconut, Date, or any fruit of any type being offered or broken, so tha whole area is filled with water and smell.

b. I did not see any flowers like GENDA, Tulsi, Rose, PIPAL, Lotus etc. being offered an littering the floor and area. c. I did not see any cow, ox, goat, buffalo, camel or any animal around there.

d. I did not see any GULAL,AMBAR, SINDUR, KUMKUM, or any sort of colors being thrown away or smearing the walls of Kaaba or near or on Black Stone. e. I did not see any priest or person with BHABUT, YELLOW, RED, WHITE, or any other marks on their foreheads.

f. I did not see any one with DHOL, TAASHA, BANSURI, SITAR, or any , other musica instrument. No one singing BHAJANS, QAWWALI, etc. there. g. I did not see any ladies with THALS of AARTI, or dancing GARBA, KATHAK, , KATHAKALI, BHANGRA, or our Filmi style dances or "items" in the arena for every one to enjoy. h. did not see any separate lines for VVIP and for common people to do DARSHAN. J. And there was no DAAN PETIKA (Contribution Box) which is so essential part of Every temple, every JHANKI, JAGRAN, GANESH PANDAL or any PUJA. There was no scope for anyone to make any money out of this entire process. 2. What the Deity was looking for:

a. From what I could see, every one wearing two white non-sewn sheets wrapped around him (women fully covered in black BURQA with only faces and hands visible). No one knew the person next to him is a beggar or millionaire.

168 of 277

Questions and Answers with Non Muslims Vol-8

Q# 161

Madina

II. Visit to Madina:

After I finished UMRAH at Makah, Allah blessed me with the visit to Madina, the City o Prophet, which is the second holiest place for Muslims after Makkah.

The purpose of Visit to Madina is to pray at the prophet's mosque and to offer Salutations and thanks to Prophet and his companions who are buried there. Here is a brief account of these places, to quell the doubt that Muslims Worship their prophet and do any such things: 1. The Prophet's Mosque:

a. After many expansions, this is one of the most beautiful buildings in the world. There are no words to praise its construction and its minarets and the overall effects it has. b. Suffice is to say that Governments. And Rulers all over Muslims history have left no stone unturned to beautify and expand this mosque. c. From its humble beginning with date stalks as pillars and date leaves at its roof (when Prophet constructed it himself) it has grown to its present shape. d. It was in this mosque that whole history of Islam' first 40 years or so is written down, The events which changed the world were taken here and around this mosque. e. Now the Power of this mosque is gone, but its importance is still in the hearts of every Muslim. We are supposed to go and pray there and ask Allah for His blessing and guidance and Help. d. There are no special prayers prescribed here, except for voluntary prayers at spots of historical importance. 2. Visit to the Prophet's Grave: a. As every scholar of Islam knows, the prophet died and was buried in the House of his Wife Ayesha in 11 AH.

b. The prophet , who was the whole sole ruler of entire Arabia had no "Palace or Mansion or Big House" of his own. What he had was nine small rooms, one for each of his wives. c. It was in these rooms that he would shift, one night in each of his wives. And it was in the house of Ayesha that he died and was buried in the same house.

d. This house and all the other houses of the prophet's wives are now part of the Prophet's mosque. The grave is invisible to public, who can see it from the grills outside.

169 of 277

Questions and Answers with Non Muslims Vol-8

Q# 162

Modesty

[QUOTE who="CTB"] 1. Such as going to a synagogue, a church or freely expressing their own faith without threat. .. 2. I have a friend who is Muslim and does not wear the scarf all the time. .. As for modesty, I am modest. ..

3. OK. That is Islam's belief and with that I respect. I am learning about Buddhism and its zen on modesty and its philosophy of the middle path. 4. Some god. That's a lot of people going to hell. I guess it makes it sense as fear tactics often work in order to control masses. [/QUOTE] Ans.

01. Same way as you follow laws of your country and your parliament, Muslims follow t laws of Shariah. They are only applicable for Muslims.

If you do not like it, it is your own problem. But we can show that NONE of the Shariah laws are harmful to human society and human civilization and they are moderate and hav sufficient flexibility to take care of changing circumstances.

And I do not OWN any country, I have Indian citizenship and only work in Saudi Arabia 02. Once you are Muslim, you have to follow the guidelines and instructions of your Creator and follow the teachings of your prophet. Islam is not just a label which you put on your shirt sleeve. It is a state of belief and Muslims men and women follow Islamic laws by their own free will. No law can be enforced against the will of people and if majority is against it. We see Muslim women in west wearing Hijab of their own, there is no force on them (while there is ALL the pressure from Media and society to remove it). They do it, because they like it!!

If you are modest, I welcome you, but every one cannot have his or her own definition of modesty, that is why Islam sets the limit and within that limit you have much flexibility.

Islam does not give women the status of property. In Islam a Muslim women is a lega person and entitled to rights and freedoms.

But Islam does not go to the Other extreme by making women as Exact Equal to man
170 of 277

Questions and Answers with Non Muslims Vol-8

Q# 163

Names of God

A. Quran and beautiful names of Allah I have seen with interest, what our learned readers have contributed towards beautiful names of Allah, I think it is high time to give some inputs as to what are Quranic guidelines for these beautiful names Quran on the one hand says To Allah belongs beautiful names, remember Him by these names

And on the other hand it also says And part company with those who profane the names of Allah (or make it an object of fun. Therefore finding beautiful names of Allah is a very serious matter and should never be attempted as a matter of jest and merrymaking. B. Examples of beautiful names of Allah mentioned in Quran: Quran gives a list of around 99 beautiful names for Allah in Arabic language and says that there could be many more in other languages of the world.

Let us just scan thru some of these beautiful names and see, how much childish and ill educated is the approach of those who have suggested some names from their own minds: A. AL RAHMAN = MOST GRACIOUS B. AL RAHIM = MOST MERCIFUL C. AL MALIK = THE SOVERIEGIN D. AL QUDOOS = THE HOLY ONE E. AL SALAAM = THE SOURCE OF ALL PEACE F. AL MUHAIMIN = THE ONE WHO PROTECTS G. AL AZIZ = THE IRRESISTAIBLE H. AL KAHLIQ = THE CREATOR J. AL BARI = THE PLANNER, THE ORIIGNATOR H. AL RAZIQ= THE ONE WHO FEEDS The Quranic list compromises of many words which have deep rooted meanings. It is strange that people find time to make fun of such a serious subject.
171 of 277

Questions and Answers with Non Muslims Vol-8

Q# 164

Old religions

[QUOTE who="Eric"]

01.There was no need for G-d to name the way of life because it is the only way of life th G-d ever approved. And for that reason, there is no need for those who believe in it t name it either.

02. Oh, one more thing. At the time in question, people were referred to not by their religion but by the name of the country they resided.[Boy that sounds like today!] Up to thru the time of Solomon, the country was Israel (Yisrael)and the people Israelites (shel b'nai Yisrael). When Israel split in two following the reign of Solomon, the nam of the southern country was Yehuda in Hebrew and later Iudaea in Latin. People who resided in there were referred to as Yehudi or Iudaeorum (of Judea) However, not all Yehudi or Iudeorum were of the Tribe of Judah. The priests were of the Tribe of Lev but they lived in Judea. Historically, Paul was of the Tribe of Benjamin (probably killed off the tribe single handedly.)

[/QUOTE] Ans.

01. It is your assumption, that there was no NEED for God to name His religion. There is no proof from the scripture. And you cannot say that ALL people around the prophets followed their religion or ALL people were one group.

There were people then (as we have today) who were following different paths and faiths and religions, so there was EVERY need for God and Prophets to call people to their way by a specific name. 02. I thank you for the explanations that you provide regard the origin of name Judaism. That may be OK. But it does not provide any answer to the original question.

What you have written shows that Religion of Noah, Abraham, Isaac, Ishmael, Jacob, Joseph, Moses, Aaron, David and Solomon was not Judaism. But what religion they followed? Your books do not give you answers (because they were written After Solomon, when Jews were already divided!!). It is Quran, which has preserved that name. And it is Islam, you can call that in Hebrew SHALOUM!! The religion of Peace!!

172 of 277

Questions and Answers with Non Muslims Vol-8

Q# 165

Old Scriptures

[QUOTE who="Eric"] 1. Would you please give citation to where the Qur'an says this. Which of the 5 books of the Torah are not the revelation of G-d. Which of the 5 books of the Torah were not transcribed originally by Moses. 2. First, thank you for finally admitting that "every group has his own 'Original Manuscripts'". That has been my contention from the beginning. .. 3. Again, thank you for admitting that there are many examples of changes. [/QUOTE] Ans. To: Mr. Eric: A. Quran and Torah, Injeel and Zabur:

There is some common misunderstanding than when Quran speaks about Torah, Injeel and Zabur, the Jews and Christians think that it means Pentateuch, (i.e. First five books o OT), Gospels (First four books of NT) and the Book of Psalms. So let us first clarify this misunderstanding: 1. Quran and Torah:

a. According to Quran, Torah was the revelation which God gave to prophet Moses (peac be on him). Torah literally means the Law and (part of) it was given in written form to Moses when he went to Mt. Sinai to talk with God.

b. That was the basis of Torah (I think it refers to the Ten commandments), and after that God send other instructions to Moses as he managed the affairs of Jews during those forty years of wandering in the desert of Sinai.

c. The revelation which Moses revealed and which was preach ed by Moses was Torah an which is the book which Quran praises in many places.

d. Torah (according to Quran) is not the first five books of OT or Tanakh which are ascribed to Moses. No one really knows that who wrote those five books and that all its contents were seen my Moses. There are inside informations which deny that it could ha been written by Moses (for example ending of Book of Deuteronomy and so many stories of incest recorded there).
173 of 277

Questions and Answers with Non Muslims Vol-8

Q# 166

Old Scriptures

[QUOTE who=" "] [/QUOTE] Ans.

I have no intention of entering into a protracted debate with Mr. Mahmood, as I have said on many occasions. All other religions in the world have their "books" but have lost their "Prophets"!!

They have no authentic records of lives, sayings and actions of their prophets, so they jus go by the book and every one interprets it its own way. In Islam it is the sayings and actions of prophet which are recorded in the most authentic way humanly possible, so Muslims know how to interpret their "Book".

And since the very early stages, enemies of Islam are trying to wean away Muslims from their prophet and let only the "Book" remain, so it can be interpreted in any way they like. All deviant groups in Islam have tried this route to attack Islam and undermine its teachings.

174 of 277

Questions and Answers with Non Muslims Vol-8

Q# 167

Paran

[QUOTE who="G Rider"] Mt Sinai is in the Egyptian Sinai.

The wilderness of Paran is the Negev desert of Israel Sinai area not Mecca or Medina are like Muslims believe. The Wilderness of Paran is mentioned in the Bible as a place where the Israelites have spent much of their wanderings following the Exodus. King David spent some time in the wilderness of Paran after Samuel died I Samuel 25:1. Today the name relates to the watershed of Wadi Paran. [/QUOTE] Ans. Do not get confused with modern day political and national boundaries. 1. If Mt. Sinai is in Egypt, then Jews after Coming out of Egypt were still inside Egypt? !! Red sea separates Egypt and Arabia. After getting out of Egypt, Jews wandered in deserts of Arabia for 40 years and in that desert there is Mt. Sinai 2. Paran is not a specific place, it is spoken for the entire land of Arabia. When we speak times of Abraham and Ishmael, it refers to place where Ishmael was settled and that was Makkah, which is also a part of Paran. David, no doubt fled to desert of Arabia when he fled from Saul, it was very near to the Jewish areas, they were joined at that place.

I know Jews and Christians want to expunge the name of Ishmael and every thing related to him from their books, but they cannot, why? Because God Promised to Make Ishmael also a Great Nation!!

175 of 277

Questions and Answers with Non Muslims Vol-8

Q# 168

Past Scriptures

[QUOTE who="I Trip"] The Quran tells Christians and Jews to consult the scriptures and Gospels. So according to the Quran they must be uncorrupted, or else they make no sense. 002.041 ... 002.044 002.089 007.157 005.047 005.068 053.036

This next verse tells people to check the Quran s revelations against the scriptures so as to confirm them. 010.094 If the scriptures are corrupted then the Quran can't be true either. [/QUOTE] Ans. First of all I appreciate with your research work and many quotations you have provided from Quran in support of your claims. Your case is totally different from these Non Thinking Thinkers who issue blank and generalized statements, without doing least amount of thinking and analysis. The subject you have raised is a very serious one and needs careful analysis, does Quran certifies that Every Book and Every Word of Bible is the word of God or that Quranic accuracy is dependent on the accuracy of these books: A. Quran position on Past books: a. Quran in general confirms that God did sent prophets and messengers to every nation
176 of 277

Questions and Answers with Non Muslims Vol-8

Q# 169

Politics

[QUOTE who="Arbitrageur"] How many parties does Islam allow? [/QUOTE] Ans. Islam does not prescribe any specific type of models as to how to run the government, except saying Their issues are settled by mutual discussions (SHOURAH).

The rulers should be selected by the consent of people. Not just by counting people but b the consent of those who run the matters of state.

There is nothing wrong if any leaders continue till his death, if he is doing good and peop are happy and those who run the matters of state are satisfied.

There is no pressing need for him to get the re-consent of his people every five years. If a any time during his rule people and those who run the matter of state, think he is incapable, he can be deposed.

But the model where there has to be groups and each one trying to do down the others. And when one group gets power, the other groups are hell bent on criticizing it on each and every issue and the one in power hell bent on supporting his each and ever action. And that this drama of election repeated every four or five years and then the parties changing roles. Is not what is very much appreciated by Islamic model.

But if Most people of any country want this model, they could try it and waste their mone and energy and time in doing these things.

That is why in our time Politics has become a Business and not a service and trust, whic it was supposed to be earlier.

177 of 277

Questions and Answers with Non Muslims Vol-8

Q# 170

Prayer

[QUOTE who="Redoran"]

01. You spend a goodly part of your day praying to your Allah. Therefore, you will never become a productive society.

02. I spent a minimum 12 hours a day for many decades, working, no prayer time at all. N Temple, no Synagogue, no church, no nothing.

03. Why should I pray to some god that created me in the first place? I should thank it for creating me? HA! I think I was a productive member of society, not of a supernatura vapor. [/QUOTE] Ans.

01. Considering that Allah is our Creator, Lord and Has given us everything, it is only fai that we spend a lot of time in praising Him and singing His glory.

But the same Merciful God has asked us to go and earn our living and go out and do hone works. That is why a devout Muslim is no hermit, he goes out and does his work.but wherever he may be, he never forgets his Creator.

02. And on practical level, the time Non Muslims waste on tea and coffee breaks, lunch breaks and gossiping and watching TV, listening to music etc. Muslims spend it on remembering their Allah. If Muslim workers were less productive, no one would hire them at all. 03. Why should you pray to God that created you? This is a good one. You would be really ungrateful child if you are not thankful to the One who has done so much for you. Allah says That is you count His mercy, you will loose count!! Why one should not find time to thank this Merciful God? And by thanking Allah, you are not adding anything to His stature or His Power and His dominion, it is for your own good and for keeping you on right path.

So a Muslims gets the best part of this world and the Hereafter a Non Muslim (or on who has no time to spend on the remembrance of God), might get something of thi world, but nothing in the Hereafter. So it is a choice that everyone has to make.

178 of 277

Questions and Answers with Non Muslims Vol-8

Q# 171

Propagatio [QUOTE who="Lilya"] n 1. Muq stated: "Islam is not the fight for any piece of land. It stands for principles." You mean like going into other countries (America included) and proclaiming Islam principles as being the only way, and initiating movements to overlay those of the host country, to herald in your evil concepts of principles. It also shows your desire to assimilate all cultures and lands for the evils of Islam to rule.

2. Are you saying that Islam with its Sharia Law, and all those other obscene practices of stoning, beating, hanging, sexual mutilations of females, enslaving females and forcing them to have no human rights, keeping males in a drugged state where they think they are supreme and worthy of being a human, taking your young boys and teaching them how to cut heads off and steal their futures with evil hate of all, are not God given? That Allah didn't advocate these barbaric actions?

3. is this Allah's concept of ordering justice, equity and peace to be maintained in the land [/QUOTE] Ans.

01. The Muslims in America are its citizens and they have right to believe and propagate their religion. If they argue that they should be judged according to Shariah law, the are not doing any crime. And if they argue and point out the beauty of Islamic Shariah to people of America, they are not doing any crime either. Are not US and all Western people trying to impose their culture and their version of Democracy on Muslim countries? 02. The "Judgment" which you have pronounced on Islamic Shriah is very biased and highly objectionable. Islamic Shariah and its rules are based on Nature of males and females and divide rights and duties of which are as per their nature. Islamic Criminal justice system, brings down the rate of crime in any society to its minimum leaven and most of people live in peace and tranquility.

But I know Western Justice System only worries about Criminals and Their Rights.tha is why the rate of crime never comes down in Western Countries and is always on the ris

03. Yes This is Allah' ways of maintaining peace and Justice and equity in the society. You have tried to put your own wit against the wit of God and every one can see the resu
179 of 277

Questions and Answers with Non Muslims Vol-8

Q# 172

propagatio [QUOTE who="Dabang"] n Muq, 01. Just like you say Islam is good but most followers (Muslims) are bad. .. 02. I have no intention of persuading you to convert you into any other religion. .. 03. It means just reading loads of books is no use unless you have LOVE in your heart. ..

04. Action speaks louder than the words. Set an example first and let people ask you which faith you follow rather than trying to sell your religion. [/QUOTE] Ans. 01. We do not invite people to follow the ways of Muslims but call them to Islam and follow its teachings and be a better Muslims than the present lot.

Because, according to Quran, dieing in the state of Islam is essential for them being save from Hell Fire in the Hereafter and it is not voluntary for them to choose any other path and still be saved from the hell fire.

02. If you feel that your religion or faith is better and best on the earth and is essential for salvation of mankind, you must try to convince other people and call them to that pat

Otherwise you are only a hypocrite and do not have firm belief in your own faith.or yo think that divergent and conflicting and contradicting ways and views will all lead yo to the path of salvation. This is wishful thinking. 03. You do not have to read lots and lots of books to understand the basic simple philosophy of Islam and then compare it with your own beliefs and find which is closer to truth and logic and reason.

04. Yes action speaks louder than words and muslims should set an example for others . But is it not unfair to these Billions of Non Muslims to go to hell because Muslims were busy in improving themselves and lacked the drive or courage to tell Non Muslims to follow teachings of Islam. No Non Muslim will be held responsible for wrong actions of Muslims.. . but I think almost every Non Muslim (including you) will blame Muslims (in the hereafter) that why did they not preached the message of Islam to us?
180 of 277

Questions and Answers with Non Muslims Vol-8

Chapter-8 On Islam-4

181 of 277

Questions and Answers with Non Muslims Vol-8

Q# 173

Prophet [QUOTE who="agree"]<quoted text> more inspirational stories, - Noah was sexually abused by his own son (Gen 9:22 interpreted in Sanh. 70a) - Lot committed incest with his own daughters - Aaron created the golden calf for idol worship when a holy scripture attributes these heinous acts to righteous men of God (anashim elohim), and not to the bad guys, there must be something wrong somewhere in the transmission of the ancient source - the lying pen of the copying scribes to be exact (Jer 8:8)[/QUOTE] Ans. Thank you for additional inputs....(in fact we did mention them in earlier posts). There is basic difference in the concept of prophethood between Muslims and Jews and Christians. As per Quran, the prophets are chosen people of God, who are selected at birth and they deal a very pious and exemplary life. They are free from any moral defect and never commit any sin or transgression which requires moral turpitude. They are set as an example for their followers to copy from. And the people will be as good as their models are. Jews and Christians on the other hand believe that a job of prophet is just to pass on the message received from God, after that he is as a human as any one else, they cheat, they lie, they massacre people and they commit incest and whatever else crime you can think of. But they are still prophets of God and these acts do not lower their status. And that is why Jews and Christians do not raise an eyebrow when these crimes done by their prophets are mentioned. They say "what a big deal? After all they were also humans".

And it pains every Muslim to see such crimes and sins attributed to these innocent prophe of God, because as per Quran they were all Muslims and they were free from all thes allegations. Now the choice is before people, select which ever model they like better.

182 of 277

Questions and Answers with Non Muslims Vol-8

Q# 174

Prophet

[QUOTE who="Buford"] bmz, MUQ vamoosed, so I'll ask you, "Why would Gabriel order Mohammad to READ when he couldn't READ?" [/QUOTE] Ans. BMZ and Frijoles have provided their answers. Gabriel said the word IQRA to the prophet, which has many shades of meaning. In its simplest sense, it is used to read from a book or parchment in front of one. And in another sense it is used for repeating what the other person is saying.

So when Gabriel said only the word IQRA (which was part of the verse which was going to be revealed), the prophet said I cannot read (thinking in its common sense).

However after saying the word two or three time, Gabriel read the full verse IQRA BISM RABBIK AL LADHI KHALAQ (Recite in the name of Thy Lord who Created). Then the prophet got the meaning and he repeated after what the Angel has said. Allah Knows best. PS:

1. There are some sayings in Islamic literature which say that Gabriel brought these verse on a parchment and asked prophet to read. When he said he could not read, then Gabriel read and the prophet followed after him (Allah Knows best).

2. And this Read and I do not know how to read sequence was to fulfill a prophesy in th Book of Isaiah where it is said The book is given to one who does not know how to read! (Allah Knows Best)

183 of 277

Questions and Answers with Non Muslims Vol-8

Q# 175

Prophet

[QUOTE who=" "] [/QUOTE] Ans. Wailukum Assalam Respected Brother BMZ For that reason NO Prophet comes with any NEW message (according to Quran). They all come to REVIVE the same OLD message of Islam which was preached by all prophets that came into the world.

Remember Quranic verse that the religion in the eyes of God has always been one (and it very logical too).

I am seeing that you are getting very hard on Jesus, what Christians have done to him and destroyed his Prophetic image, does not allow us as Muslims to cast any doubt on the ability or credibility of Jesus Christ who was a Mighty Messenger (ULUL AZM) according to Quran. I know it is very difficult to keep the balance course. Arguing with false notions of Christians and Gospel writers while preserving the dignity of original Jesus.

That is why I follow a very cautious course and try my best to never pass a loose commen on Jesus as a prophet.

The job which God gave to him was to reform the Jewish religion by introducing moralit into a religion which had become too ritualistic (during his time) and give the world glad tidings of The Last and Final Prophet which will come into the world.

And he did both the jobs admirably well and God took him up, when his enemies tried to kill him and he will come back to earth near the End of Times, to bear witness about his actual status.

Allah save me from saying or believing or doing any thing which lowers the position and respect of Jesus in any way. I seek forgiveness from Allah if I have done any such thing knowingly or unknowingly. This is my way and I do not think that I am deviating from the Quranic line.

Once again I thank you for post and prey to Allah to keep us on the middle path, saving u from going to any extreme. MUQ

184 of 277

Questions and Answers with Non Muslims Vol-8

Q# 176

Prophet Muhammad was shit scared of the eclipses. he didn't understand this phenomenon and believed it to be a sign of Allah.

Now who will tell the idiot Mohammedans that of all the billions of galaxies and trillions of billions of planets of this universe, every moment there would be an eclip happening as we speak. would MUQ remain in prayer mode with his raised posterior perpetually on that account? Ans.

Considering what were the superstitions prevalent in the world 1400 years ago, our proph showed an attitude which was centuries ahead of its time. As per Hindu mythology:

When Lord Vishnu churned the oceans using Himalaya mountain as post and Giant snake SESHNAG as a rope and Angels pulling from one side and Devils on the other side till one by one gems started coming out of Great ocean.

The last gift was a pitcher of AMRIT a potion that makes conquer death. The AMRIT w to be shared by all, but VISHNU wanted it to give it only to angels, to he asked them to sit in separate rows and giving it first to angels (with a trick that by the times it reaches devil, it will be finished).

But two devils, RAHU and KETU became wise and they sat in the row of angels and dra that AMRIT. Now Sun and Moon spotted them and pointed it out to VISHNU who instantly removed their head from their bodies. Now these heads of RAHU and KET are searching the sky for Sun and Moon and when they catch them they swallow them That is solar and lunar eclipse. But since they have no body , so sun and moon come out and so period of this eclipse is short.

I do not know what Christians, Jews and Romans and Greeks and other people had to say about these Solar and Lunar eclipses. Our prophet attitude:

The day his son Ibrahim died, there was a solar eclipse in Madina and people started sayi Because son or our prophet died today, even the sun is mourning

When prophet heard this, he climbed the pulpit and said Sun and Moon are two signs fro Allah they do not eclipse on any one birth or death, so when you see these signs, pra
185 of 277

Questions and Answers with Non Muslims Vol-8

Q# 177

Prophet

[QUOTE who="Skeptic"] You are right there are no proofs.

That includes that the Quran can not prove its scripture comes from Allah and there i no proof that Mohammed is a prophet. But what can be proven my Mohammeds own life on earth is that he inspired Muslims to murder hundreds of millions innocent non-muslims for rape, booty, and slavery. Those records MUQ of Eye witness's against Mohammed are still in the history archives. [/QUOTE] Ans. I am glad that you have FINALLY accepted that there is no PRROF that ANY of your Books of OT and NT is from God. And it was only your false claim and false assumptions that you were saying so far. But you have again made a false claim that our Quran is not from God, you should have asked us for proofs before making such false claim, as I had asked from you. And our proof is very simple and very straight forward; It is Quran itself which gives the PROOF and falsifying test!! Listen to these verses from Quran: 01. And if you are in (any doubt) as to what We have revealed from time to time to Our servant (is not from God), then produce a Surah like it: and call your witnesses or helpers (if there are any) besides God, if you are truthful (2:23) And

02. Or do they say he has forged it say Then bring a Surah (chapter) like it, and call ( your aid ) anyone you can, besides God, if you are truthful (10:38) And

03. Or they may say He forged it say then you bring ten Surah s (Chapters) forged lik it, and call to your aid whomsoever you cam, other than God, if you speak the truth (11:13) And
186 of 277

Questions and Answers with Non Muslims Vol-8

Q# 178

Prophet

[QUOTE who="I trip"] After Muhammad had received his first revelation he ran home to his wife, Khadija who took him to see her cousin Waraka Ibn Nawfal who was a Christian convert. So in walking distance of his house lived a cousin of his wife's who knew the bible's stories.

Whilst we can never know just how much they talked about the bible there's no reaso to believe Muhammad hadn't heard all the stories we find in the Quran. [/QUOTE] Ans.

If that is the sort of proof that it is needed to prove that Our Prophet heard the stories of Bible and all these special books which were not part of the Canonised Bible. then there is much more proof that Osama did 9/11 single handedly and Saddam had links with Al Qaeda and had WMD and Nuclear capabilities!! I think it is this attitude of you people, which allows you to believe in ANYTHING and EVERYTHING you hear or imagine about your adversaries.

This incident of prophet running to his wives and she taking him to a person who had som knowledge about these things, proves the sincerity and innocence of both prophet an his wife.

As some people claim that prophet had heard all those stories while he traveled to Syria during his trade routes and was secretly rehearsing them in the cave to pass it later on as revelation. His reaction would have been totally different than what we observe And then this once meeting with Waraqa does not mean that prophet became a regular visitor to him afterwards. The records show that Waraqa died soon afterwards. And what type of Christian would tell prophet that Jesus was not God and neither Son of God and there is no Trinity and Jesus did not die on the cross. And why each Quranic story differs in detail and morals from what is recorded in Bible?

So after analyzing all this evidence, we can say that Yes Saddam did had link with Al Qaeda and He did have WMD s and Nuclear Capabilities and well. Osama did 9/1 single handedly and three Towers fell on 9/11 with a single jet strike!! Right, Mr. Sherlock Homes and Dr Watson!!

187 of 277

Questions and Answers with Non Muslims Vol-8

Q# 179

Prophet

[QUOTE who="Mahmood"] ... That is what Mohammad did in the case of Kaab Ashraf & Asma bin Marwan. [/QUOTE] Ans. Another problem with you is that you have a very short memory. Even if we have discussed a topic one hundred time, you always come back with the same comments as if it was the first time we ever discussed it. That is why I told that we have discussed enough and there is nothing left to discuss. So before I put before you again the crime list of Kaab and Asma.let me put a passing comment, you said killing by any Army General in defense of his country is not murder. So killings by Army Generals who attack another country comes under Murder? So all the army generals who attacked Iraq and Afghanistan and conducting air raids on Libya, Syria, Yemen etc are murderers? Just pleas say so in simple worlds so every one could know how unbiased and how straightforward you are!! A. Crime List of Kaab Bin Ashraf:

01. His tribe had made a pact with the prophet that they will not help enemies of Muslims But this person after the battle of Badr went to Makkah and sided with Makkans and incited them to take revenge from Muslims. This comes under high treason and his punishment is death in any civilized society.

2. He used his poetry to criticise the personality of prophet and lampoon him. This is agai a crime under the head of state and state. In the society of Arabia the poets were the source of what we call mass media of our time.

So this was like causing mischief in the land and for which any penality could be provide

3. Then this Kaab started using his poetry to malign the characters of Muslim women, tha showed the meanness of his character and the immoral nature of his personality.

We can liken this person like a Mad Dog which should be silenced quickly and surely. That is why Prophet said that such people themselves make shedding of their blood lawful.

188 of 277

Questions and Answers with Non Muslims Vol-8

Q# 180

prophet

[QUOTE who="Buford"]

Sad, isn't it, that Mohammad WASN'T the "Word of God," WASN'T the Christ, WASN'T empowered to perform miracles, WASN'T born miraculously, and DIDN'T give life the dead by God's permission or heal those born blind or with leprosy by God's permission! But never fear! Mohammad is still the SUPERIOR prophet!!! [/QUOTE] Ans. You are absolutely right!! Our prophet was Not the Word of God He was Not born Miraculously He Did not perform all those Miracles -He did not gave life to the dead by Gods permission, - He Did not cure those born blind, or with leprosy, or born blind or deaf Yet and. Yet, he achieved things which the one who did all these things could not achieve!!

When our prophet Died, he saw that whole country of Arabia was following his faith AN he trained and left a band of more than 124,000 Dedicated Companions who carried out his mission to all parts of the world. And the prophet who had ALL the qualities you mentioned in your post, when he died, bulk of his people had rejected his message and he died as a petty criminal on the cross and all his trusted disciples left him and were scared to do any thing or even open their mind.

You become Great by what is your achievements and not by what favours and gifts you g from God.

And By the way, we do not rate prophets and put one prophet against another, it is not ou job. For us every prophet is equally honored. It is God and God alone who ranks His prophets and not us; PS:
189 of 277

Questions and Answers with Non Muslims Vol-8

Q# 181

Prophets

[QUOTE who="DHG"] 1. Someone forgot to tell Joseph Smith.

2. Hey, if there were 224,000 prophets (or even 1000), how come Islam only remembers the ones whose names were preserved by other religions (25) or who had lasting impact on their culture (like Socrates) and whose names would be widespread even among the ignorant? 2. If Islam is perfect, why is it's knowledge of the names of the prophets so utterly incomplete? Doesn't this indicate that the Quran is 'incomplete'? [/QUOTE] Ans.

I think your answer is based on some misunderstanding.. usually your posts are very balanced and to the point. Here I will give brief replies to the points you mentione

1. You should ask this question with Joseph Smithas per us, our prophet was the Last and Final prophet in that glorious chain of prophets.and one claiming prophet hood after him, becomes a liar by default.So I do not worry about Joseph Smith or ANYONE else who claims Prophethood (believe me, there have been hundreds of such false prophets, since the death of our prophet).

2. Quran is the book of Guidance for humankind and not an encyclopedia or a list of nam of ALL Prophets and ALL books that God sent to various nations and at different times. It only mentions name of those prophets which were familiar to Arabs (the original recipient of the Message) and Jews and Christians living near by.

The message would have got diluted, if it mentioned the list of 124,000 prophets (most of them obscure and no one knowing them).

3. Islam is perfect, because its teachings are perfect and sufficient for humans till the day of judgment.

You are putting your own standards of perfection and you want that Quran should follow your criterion than the other way round.

190 of 277

Questions and Answers with Non Muslims Vol-8

Q# 182

Quran

[QUOTE who="Rick Moss"]

Technically speaking, the Koran isn't an ancient book. The term Ancient typically refers t the time from the fall of classical civilisation (4th century CE and before). The Koran dates from the well after that so is no more an "ancient text" than a John Grisham novel. [/QUOTE] Ans. Ancient and Modern are relative terms and there can be no technical limits on it. When Moses came out from Mt. Sinai, he had a Brand New book in his hand, now we call it an ancient story. When Jesus was preaching , his sermons were the newest news on the block.but we call that as an ancient story. When Quran was being revealed, it was also the newest thing on the planet.but after 1400 years it also comes under the title of ancient. The time is fleeing and there is no definite line which can separate modern from ancient. Time itself is a relative thing as per Theory of Relativity.

Some times our own period 2000 CE would also come under the category of Ancient, s do not mind about such things. PS:

How many complete and intact manuscripts of OT books we have in our hand, that date t 7th Century CE?

191 of 277

Questions and Answers with Non Muslims Vol-8

Q# 183

Quran [QUOTE who="Bill"]<quoted text> Hi Muq, interesting set of posts you've made on the old testament. And I agree with this biblical criticism. But why is the Quran any different? Doesnt Muhammad use the Old Testament as his source material for his book? [/QUOTE] Ans. Thank you for liking my post, as you see, I am not the author, I am just posting it. A. Quran is different from Bible in many ways, some of them are: a. It is Direct Word of God as revealed by Archangel Gabriel to our prophet. No other word of any one (even our Prophets) is mixed with it. b. It was immediately written down and memorized after the revelation was completed.

c. It is preserved and protected in its original language and its textual purity is beyond any reasonable doubt in the world scriptures. d. Since its day of revelation, Quran is being memorized by hundreds of thousands of Muslims thru out the ages, such a memorization is absolutely unique in the other world scripture. You would hardly see any Muslim Preacher carrying Quran while speaking and you will notice hardly any priest without Bible under his arm when preaching.!!

B. And it was wrong for you to say that our Prophet used Bible as source material for Quran, because: a. Our prophet did not know how to read or how to write.

b. Bible (its Arabic translation) was not that easily available in Makkah of his time and he did not have any known teacher or guide to help him compose Quran. c. There is only outside similarity between Quran and Bible telling the same story. In Bible the thrust is on its historical side, while Quran deals with the lesson and morals of each story. d. It was not possible for an unlettered man to have so much command over Bible that catch the source of its deviation and comment on it in authoritative way.

e. Even a plain reading of Quran would convince any unbiased and fair minded person th Quran could not have been composed by our prophet. The wordings are so majestic and the comments so authoritative that no Human could have made it.
192 of 277

Questions and Answers with Non Muslims Vol-8

Q# 184

Quran

[QUOTE who="I trip"] Considering the many differences between the Bible and Quranic stories my impression has always been that your prophet had heard the stories but couldn't remember the details. [/QUOTE] Ans. This can be said by only that person who has not studied Quran seriously. Quran provides the Moral value of each story and not Historical details. Scriptures are the books of Guidance and not the books of History.

There is separate section for the books of History in every nation and every library and they should not be merged with the books that deal with Guidance for the human kin

That is why Quran deals with the core matter and not the obiter dicta which Historians lik so much.

Give me a single sentence of what Noah or Abraham preached to their people from Bible Why their prophetic side and preaching should be blocked out from the Book of God?

Why should God be interested in telling how many cows, sheep and camels Abraham or Noah had and do not tell about what they preached to their people and what was thei message to their people?

And then Quran pin points the errors and provides answers to most of the controversies which Jews and Christians and different Christian sects have between them. Quran says This Quran does clarify most of things on which People of Books (i.e. Jews and Christians) dispute.. Comments like yours are from those who have only seen the Translations of Quran but never took serious study of these verses.

193 of 277

Questions and Answers with Non Muslims Vol-8

Q# 185

Quran

[QUOTE who="Trip"]

01. Your arguments are also based on assumptions rather than proof. How do you know what Muhammad did or did not know or who he spoke too or wha was common knowledge in Mecca? 03. So your God couldn't protect the important part of his message? 04. I think Osama had a lot of help. A jet, the resulting fire and the unique design of the towers. 05. .151: Say: "Come, I will rehearse what Allah hath (really) prohibited you from": Join not anything as equal with Him; be good to your parents; kill not your children on a plea of want... It's prohibited to be good to your parents?

6.151: Say: "Come, I will rehearse what Allah hath (really) prohibited you from":...b good to your parents... I presume this is bad grammar. I think I could do better. How about never disrespect your parents? [/QUOTE] Ans. A. Prophet and Copying from Bible:

01. When you accuse some one, it is for YOU to bring the proofs not HIM to prove his innocence. A mere denial from his is enough to refute your claim. This is what Natur justice says. 02. Our prophets saying that I did not copy, carries all the weight, because he was a person who never told a lie in his life, he was unable to read and write and never studied under any teacher and well the stories are so different from what we see in Bible. They all prove his innocence. 03. While all your charges of copying are based on assumptions and hypothesis with out having any proofs whatsoever. B. God Could not protect earlier books:
194 of 277

Questions and Answers with Non Muslims Vol-8

Q# 186

Quran

[QUOTE who="Mahmood"]

have said to you before, that as long as the idea and theme is the same, it can be construed as plagiarism. Sure, the story of Noah & Adam too has been plagiarized!! ..\ Anyway, let the readers of this debate decide. KORAN & PLAGIARIZM: EXAMPLE # 3:

In Sura Al Kaaf - 18:09 - 26, is a narrative of a group of Christian youths who hid inside a cave to escape persecution by the Romans. They fell asleep for 309 years an later woke up.

The Koran says 18:10 - " This story is a rehashment of the Seven Sleepers of Ephesus borrowed from a legend that pre-dates Islam. Mohammad (peace be upon him) took this story, mixed it with some of his own ideas, and pasted it in his Koran.

SEVEN SLEEPERS LEGEND: Seven Sleepers of Ephesus, the heroes of a famous legend that, because it affirmed th resurrection of the dead, had a lasting popularity in all Christendom and in Islam during the Middle Ages. .. I look forward to your response - refute it. [/QUOTE] Ans.

If you say that readers shall decide who is speaking the truth, then let me tell you that Tru cannot be decided by popular votes. At best let the readers hear both sides of arguments and let each one decide based on his own reasons and thinking. A. Strangest case of Plagiarism in the World!

1. This would be the strangest case of plagiarism that the world has ever seen, because th accuse him of the things which he never claimed.

2. He said that I did not write the book, I even do not know how to read and how to write they say, no you are lying, you MUST have written the book. 3. He says the info is not from me, it comes to me thru revelation, they say, no you are lying, you are concocting it yourself.

195 of 277

Questions and Answers with Non Muslims Vol-8

Q# 187

Quran

[QUOTE who="Skeptic]

If any one has a copy of the Quran, save it for it is about to be revised and the jinn-devils are going to be abrogate out of it. So save all old copies of the Quran. And any old website for Islam is now in a new phase a new look but it remains the same old ancient belief in jinn-devils. [/QUOTE] Ans. The idiot thinks quran is like so many versions of his Bible. RC Bible has 73 books and Protestant Bible has 66 books, RSV has so many portions removed from KJV and so on.

Quran since its first revelation has always remained one.and there is a chapter in it title Jinn. I dare see any one playing with Quran. And the Quran is imprinted on the hearts of many millions of Muslims who have memorized the whole Quran. This idiot does not even know what is he talking about.

196 of 277

Questions and Answers with Non Muslims Vol-8

Q# 188

Revelation [QUOTE who="DAKO"]

All religions make claims of access or sanction of GOD via divine messages. If GOD wished GOD can send message directly to each individual. However, religion is importan to psyche of humans because the fear and dependence on GOD developed while human brain was evolving. However, then to argue "mine is best" or "then only one" or "truth" is stretching the matters a bit too much! Oh! MUQ! [/QUOTE] Ans.

There is no restriction on Gods power. If God wished, He would have every human a fir believer. If God wished, He would have made angels visible to humans and telling every one what is the true path. So I am not arguing about the power of God, the question is did it really happen.

What we are seeing from the dawn of human civilization is that God selects prophets and messengers from amongst humans and sends His guidance and revelation thru these persons. That is why people accept whatever teachings they receive from their prophets.

To claim anything about Jesus, the proof should come from the lips of Jesus and not from some one else.

That is what our basic point of discussions. People claim in the name of Jesus things whic he never even thought of. In Islam we do not say Ours is best we say that All Prophets preached the same message, our prophet was only the Last prophet in that long chain. Our arguments with people of our faiths is only to ask for proofs as to how authentic and preserved their scriptures are. If the books have been tempered with, they loose their authenticity and one cannot be dogmatic about what they contain.

There is a lot of difference in the way we question people of other faiths and the way they criticize islam and Muslims. PS: And your argument that God was needed when human brain was developing holds little water, because human minds is still evolving and shall never reach the end of that evolution cycle.

197 of 277

Questions and Answers with Non Muslims Vol-8

Q# 189

Saudi Arabia

Why there are no Temples or Churches in Saudi Arabia? Part-3 D. About Christian Church:

Saudi Scholar said that we wanted to construct a Church here, but again we faced similar problems. Christians from all countries of world come here for work and we wanted to have something which is agreed by all. We faced similar problems (not as confusing as Hindus, but on a smaller scale). 1. Which Church to build? Saudi Scholar said that this was the first of many problems. There are so many Churches and we do not know which one to built as there are so many and we have adherent of so many churches in Saudi Arabia. Some of these Churches are: a. Roman Catholic Church. b. Anglican Church. c. Greek Orthodox Church. d. Coptic Church of Egypt. e. Syrian Church. f. Russian Orthodox Church. g. Jehovah witness. h. Unitarian Church.

Then some one suggested that you just make a Church of Jesus Christ and put a statute or Picture of jesus there and you forget about these smaller differences between Christians This did not solve the problem but complicated it. 2. Which Authentic picture or statue of Jesus should we keep in Church of Jesus Christ?: We were not sure which or where the authentic picture or statue of Jesus is?.

Western people said that he was like a White man, with blue eyes and golden hairs. They referred us to see films like King of Kings and Passion of Christ to see the Real life and authentic pictures of Jesus. We did not want to see a movie to know how Jesus reall looked like.

198 of 277

Questions and Answers with Non Muslims Vol-8

Q# 190

Saudi Arabia

Why there are no Temples or Churches in Saudi Arabia? Part-1

This question has been asked so many times on these and other threads, that I think some special answers is required. I posed this question to a Saudi Scholar, and this is what he replied: A. All such structures have to be ultimately destroyed: He said that Saudi Arabia is a 100 % Muslim country and there is not a single Hindu or Christian who is its citizen. What Hindus and Christians we see here are only temporary workers. They are here as long as oil is here. When the oil supply is over, they will leave Saudi Arabia and would return to their native countries.

And then we will have to demolish these structures, because we cannot leave them unattended. Then the same people will blame us (and quote destruction of Bamian statue as example) that these Muslims demolished our places of worship. B. We do not want confusion:

He said that we Saudi people are simple people, and we do not want confusion. We inde decided to build one temple for Hindus and one Church for Christians in every city.

But we wanted them to unite and give us a single answer to these questions, but we never got a single reply. I present below the results of our findings. C. For Hindu Temple: 1. Which idol to install? We wanted to install a single idol in Hindu Temple, we asked Hindus questions about it and this is the choices we got (presented in percentage) a. Ram: (8.3285 %). b. Krishna (8.2358 %). c. Brahma (6.2356 %). d. Shiva (6.3265 %). e. Vishnu (7.2589 %). f. Lakshmi (4.5656%).

199 of 277

Questions and Answers with Non Muslims Vol-8

Q# 191

Science

[QUOTE who="Really"]

01. Science merely provides a framework for obtaining answers which are verifiable a repeatable. If someone is mislead, it's because they haven't been following the proce correctly, or they misunderstood it. You can't blame science for that. You can only blam ignorant people.

02. It's more like Islam makes up stories to explain the unknown. I call that intellectual lazy. It's also known as "Argument from Ignorance" and it is a logical fallacy whi brainwashed people like you like to always fall back on.

03. Evidence that you are brainwashed. You have been convinced to believe that without your god, your life is ultimately meaningless. Your religion promotes reliance upon another being. Atheism promotes skillfulness from independence. 04. As far as what happens when you die, perhaps Shakespeare summed it up best... Out, out brief candle! Life's but a walking shadow, a poor player That struts and frets his hour upon the stage And then is heard no more. [/QUOTE] Ans.

1. I am never against science, it is only a tool and every tool has its limitations. If you do not use a tool properly, you are wrong. And if you use wrong tool, then also you are wrong. So if some scientists are drawing wrong conclusions, they are wrong and those who use science in areas which does not belong to science, then they are also wrong. Science basically is the study of forces that are found in nature and which affects the material life of this Universe. If you use science for other purpose, you will get wrong result and you cannot blame it on science at all. 02. Islam does not make up any stories. It is the Creator of Universe helping you find answers which are beyond the scope of science. I created the Universe and every thing in it what is logical and scientific fallacy in this statement? I created this Universe for a purpose and Human life is a test what is logical and
200 of 277

Questions and Answers with Non Muslims Vol-8

Q# 192

Science

[QUOTE who="CHTT"] Muq can you elaborate on your claim that science is .01% fact, with your claim in mind could you explain how we managed to land men on the moon and bring them back? would be really interested in how you apply your claim :) [/QUOTE] Ans. Either you did not understood my reply or you are trying to be dumb.

I never said that science is 0.01 % fact, I said that even in the age of science, any individu personally verifies less than 0.01 % of the info and believes in rest 99.99 % in good faith. For example, if we find written on a bottle,

Danger! Poison !! We just do not consume it, without investigating. How do we KNOW that it is Poison or How do we KNOW it will kill us? In the same way, when we find written: Danger, 25000 Volts! We do not touch the wire, how do we KNOW that 25000 Volts will kill us?

In the same way, we continue to believe each and every claim we find written in scientifi books, in good faith that it must be true. If we would start verifying, each and every thing ourselves, our lives are too short to be able to do even 0.01 % of all the facts. Tell me, I am speaking the truth or not? And then see, what charge you were putting at me? I was speaking about Personal Verification and you pasted it on that science is 0.01 % fact!!

201 of 277

Questions and Answers with Non Muslims Vol-8

Q# 193

Scriptures

[QUOTE who="Think"] MUQ, is there any scripture in the Quran equal to this teaching of the Lord? LEVITICUS 19:16- You shall not go about spreading slander among your kinsmen; 17. You shall not bear hatred for your brother in your heart. 18. Take no revenge and cherish no grudge against your fellow countrymen, [/QUOTE] Ans.

This is the first post where you have really thought!! And you must be congratulated on that.

Quran confirms the books that came before it and acknowledges that they contained Divi guidance. I have no qualm in accepting that these are very good teachings and every one should practice it.

So I will not give a tit for tat quotation from Quran to prove that Quranic teachings are Superior to these Biblical verses, but I will quote a few verses from Quran on the same subject: 01. O you who believe! Let not some men amongst you laugh at others, it may be that (latter) are better than the (former): Nor let some women laugh at others: It may be that (latter) are better than the (former), Nor Defame, nor be Sarcastic to each other, nor call each other by offensive nick names (49:11)

02. O you who believe! Avoid suspicion as much (as possible): for suspicion is some cases is a sin: and spy not on each other, nor speak ill of each other behind their back would any of you like to eat meat of his dead brother?..) (49:2)

03. Serve Allah, and join not any partner with Him, and do good to parents, to kinfolk, Orphans, those in need, neighbors who are of kin, neighbors who are strangers, the companions by your side, the way fearer you meet, and what your Right Hand posse (i.e. Slaves), for Allah does not love who are arrogant, the vain glorious (4:36).

04. O you who believe! stand out firmly for Allah, as witnesses to fair dealing, and let n hatred of others to you make you swerve to wrong and depart from justice, Be Just, that is next to piety and fear Allah, for Allah is well acquainted with all that you do. 05. O you who believe! Stand out firmly for justice, as witnesses for Allah, even as against yourselves, or your parents, or your kin, and whether it be (against) rich or poor.follow not the lust of your hearts that you may swerve and if you decline justice, or decline to do justice, for Allah is well acquainted with all that you do (4:135)
202 of 277

Questions and Answers with Non Muslims Vol-8

Q# 194

Shariah

[QUOTE who="Chuy"] 1. MUQ I find "sharia" repulsive...One of the ways you know that you serve GOD is if you LOVE "justice" and "mercy"..Sharia ...for the non-muslim is neither of those two !! I would like to ask : MUQ , how do you see sharia .You must see the beauty and structure ?? Is it fair to the Non-believer ? Your opinion , please . Sorry for asking ....but ..please ..don't make ...too...too long .thank-you 2.

MUQ...I asked this to " Thinker" he said " Atheism is a million times better than Islam ! " So let me ask you the same question ...?? What is (your opinion) better.?? Atheism "VS" Ahmadiyya !

[/QUOTE] Ans. A. Islamic Shariah:

I can say that you have been misled about Islamic Shariah, or you have been only fed wit selective portions of it. Islamic Shariah is a complete code of life that is derived from Quran and sayings and actions of our prophet. It includes every portion of humans life.

People generally think that Criminal Justice system is the ONLY thing that Islamic Shari is all about. Well they are wrong and even in Criminal justice system, Islamic Sharia is much more moderate than laws of Torah / Bible. But if you find it repulsive, then it is only applicable for Muslims if they also find it repulsive they can leave Islam and go to any other system that is less repulsive. Islam allows Non Muslims living in its society to adopt their own system, if they do not
203 of 277

Questions and Answers with Non Muslims Vol-8

Q# 195

Shia

[QUOTE who="Red Flag"]

MUQ, you are posting ostensibly from Saudi Arabia, which country is one of the most conservative Islamic nations, and which country implements a very strict constructio of Islamic law. Saudi Arabia is also the protector of the holiest site in Islam, which is the Kaaba structure in Mecca.

Westerners are interested in the difference between Shia and Sunni, for a very important reason. Islam predicts that eventually everyone in the world will convert to Islam, or be destroyed. This prediction spells doom for Judaism, thus the obvious contempt for Jews by Muslims.

Christians, of course, can convert to Islam. Islam is a religion of conversion, that is, conversion TO Islam and not conversion FROM Islam, which of course is punishabl by death according to the writings of the Prophet Mohammed. So those of us who ar not yet Muslim are curious about whether to become Sunni or Shia Muslims.

Which one does God or Allah favor, Shia or Sunni? If we should choose the wrong one, i could be troublesome for us later. Give us a break, MUQ. We don't know which Muslim tradition best suits us, or which one is "correct" or "more correct" than the other one.

So students of religion like me always wonder how God or Allah makes mistakes, fo example, why did God or Allah allow the "schism" or division of the Umma? You yourself state that the division of Muslims into Shia and Sunni has caused more trouble for Islam than any non-Islamic forces.

Errors by Allah or God are troublesome. There are two explanations for the schism o Islam. One is that Allah desired and chose to divide Islam, for His purpose known only to Him. Another explanation, problematic for the Umma, is that Mohammad created Islam from "whole cloth" and thus was not inspired of God or Allah. If Islam is a "false religion," then the schism of Islam becomes very easy to understand. Mancreated religions, being created by flawed humans, are rife with errors of design. Mohammad, we all know, was not clear in choosing his successor, thus the split or schism between followers of Ali (the Shia) and others who became the Sunni group.

MUQ, it is kind of like the problem the Catholics have had with child-molesting priests. If the Catholic religion is "correct" and "of God," then why didn't God tell th Pope back in the 1960s about this brewing problem, so the Pope could take action earlier and avoid the worldwide embarrassment and shame as a result of the festering priest child-abuse scandal?

Religions which have difficulties like the Schism or the priest child-abuse scandal have a tough time claiming to be inspired of God or Allah, unless we are prepared to claim or admit that Allah or God makes mistakes or is riven with error. Since virtual all religions posit an omniscient God, a God who can see the future, then in that case error on the part of God or Allah is impossible.
204 of 277

Questions and Answers with Non Muslims Vol-8

Q# 196

State

[QUOTE who="Arbitrageur"] (1) I'm glad you are firm in your statement, but you must mean in any country or government except Islamic ones.

(2) Here is your exception, Islam needs no separation, it gives religious AND secular direction. Interesting. This is exactly one of the important reasons why most of the world is concerned, MUQ. Did you ever wonder why there is separation of church and state in the first place? I would wager that you think Islam is the only religion that doesn't need this separation. How about Catholicism, can that exist, giving both religious and secular direction? [/QUOTE] Ans.

01. What I said about separation between state and Church was primarily about Christian Because their religious scriptures have no guidance regarding state matters. Since Christians are now leading the world, so they try to force their own methods and cultures and way of life on all other people of the world. 2. Islam is not a religion, but a way of life and we are not part time Muslims. We are Muslims in whichever situation we may be, a trader, a worker, an officer or a ruler.

03. And Islamic directions are not against humanity or against human civilizations, they are from our Creator and not from any one else. Who can know better about Humans and their nature, except their Creator?. The world is made to worry about Islam by media and people with vested interest, because they fear of their own style and way of living would go when they act according to Islamic principles. 04. Catholicism is also a man made system, because it gives all powers to Popes and priests and since they have nothing in their scriptures, so they make their own laws and that results in injustice and corruption in the land.

And most of western people, are frightened from their own past and they fear that Islamic rule also must be the same, but it is not the same, because in Islam, all basic rules are
205 of 277

Questions and Answers with Non Muslims Vol-8

Q# 197

Sufism

[QUOTE who="Karzai"]

1. Once again, my apologies if you feel insulted brother muq. You are correct, I should have known better, you being from India and working in KSA, one thing is for sure, that rules out you being a true dervish, but your stated feelings of a one true islam should have clued me in that there was a lot of sufi influence in your upbringing.

I am quite intrigued by the caste role in India, can you share your position? I mean, it is a given that prominent Muslim scholars like Maulvi Ahmad Raza Khan Barelvi and Maulv Ashraf Ali Faruqui Thanvi championed the notion of birth based caste superiority. What your stance on this notion? I find it interesting that many of the Patels I have known over the years were such strict adherance to walking in certain formations. Is this an example o caste roles?

2. Why would you not care? Where is it written that it Allah's will that only the past 2000 years are relevant for history? 3. In the eyes of religious circles they are like "Outlaws" who have usurped the authority authentic law making bodies. .. 4. Peace in palestine the past 1300 years? How about before that time? And what about during the times of the Ottomans, .. 5. Isn't it apparent to you? GWB the Great respected Pakistani sovereign lands, you can respect that can't you? 6. Now brother muq, allah would not be pleased with your last sentence about the manhattan skyline...don't you think you should apologize? That was very un-islamic... [/QUOTE] Ans.

1. A True Dervish is one who follows Islam in all its completeness. I have very good opinion about Real Sufis of India, whose Sufism is derived from Quran and sayings o prophet. 2. Caste system in India originated by living along side Hindus for so many centuries. Muslims also inherited that virus in some forms.

They divided Muslims into four major groups Syed, Sheikh , Mughal and Pathans we can refer to them as tribes in Arabia. Because they are based on the origins o their forefathers. But caste based discrimination amongst Indian Muslims is not as severe as amongst
206 of 277

Questions and Answers with Non Muslims Vol-8

Q# 198

Tanakh

[QUOTE who="Frijoles"]

1, As I feared the picture example went over your head. However, you still got the right conclusion at your final statement. Now apply that to Christianity from the perspective of Jew and you will get my point. 2. Koran is "pure" and Tanakh is not? There is no quality control on Tanakh? Hebrew is not a continuously used language? You have ignored whole series of posts on this issue. In short, you are wrong at best, and overly arrogant at worst. You are deeply in need of a course in comparative religion. I guess Saudia does not offer such a course.

3. I dont know about hallelluyah. I will check around. My guess is that it is only in few of the Psalms. Psalms in Hebrew is Tehellim - root is hallel 4. Dont forget the other meaning of Halleljah - more beans - this is found in Exodus. 2.

It is a more than a couple of words. It is also the value placed within the Christian religio system on the interpretation.

Christians reread the Jewish scriptures and reinterpret the passages to fit their own dogma and canon. This is not a small deal, a separate world religion has been formed because of this.

LOOK, I get that you do not want to acknowledge this. So, please stop pretending you do not understand This is basic knowledge that anyone living and breathing knows, plus ther are at least a half a dozen posters who have been telling you this for many weeks. So please stop the charade. [/QUOTE] Ans. To: Mr. Frijoles: A. Quran and Ancient Scriptures: a. I read what you said about your scriptures and Jews are not alone in making such claims. I have similar statements from Hindus and Buddhists and other people of ancient religions.
207 of 277

Questions and Answers with Non Muslims Vol-8

Q# 199

Tanakh

[QUOTE who="Frijoles"] Halleluyah can have at least two meanings in Hebrew 1. More beans 2. Praise God "How can they be, when they use the Same Hebrew manuscripts as their basis?" I believe Eric, I, Ex/Seaside, and others have opined to how this happened. Either you (want to) get it or you don't. .. Even better example, if I draw a PICTURE of Allah and put it on my wall as an icon to focus my prayers to, call myself a "new" Muslim sect, (and YES I get that muslims can not create pictures - that is exactly why I choose this as example). [/QUOTE] Ans.

1. So halleluyah is a Hebrew word, is it classic Hebrew and used at other places in Tanak or only in the book of Psalms?

2. I see your problem and your dilemma. Any scripture is really a scripture in the langua it is written. You have taken the example of Quran. A. Textual Purity of Quran: Quran is what it is in original Arabic. Every Muslim recites it in Arabic and no one is permitted to change any word or even any punctuation mark during recitation..

Even people who are not Arabic speaking have to recite Quran in Arabic language. Yo go to any mosque in any part of the world and you will find Muslims reciting Quran in th same way and in the same Arabic language during prayers.

Then all copies of Quran, from earliest to our present times, are exact copies of each othe

Recently I happened to visit some Historical monuments in Agra , India, like Taj Mahal and Akbar mausoleum there. These are 400-450 years old buildings; they have Quranic verses and whole chapters inscribed on their walls. Any one can go and compare, as to ho exactly they look like Quran s that we have with us today.

208 of 277

Questions and Answers with Non Muslims Vol-8

Q# 200

Tanakh

[QUOTE who="Eric"]

I'm sorry but I can't find it. I went all the way back to the first time I referred you to it on Page 932 post 20232, and I don't see it. That means I waded thru 73 pages of barbs being traded by the CT contingent and you, and I came up empty. Be a nice guy and point it out, or repost. [/QUOTE] Ans. Mr. Eric

I am sorry, I just remember posting my answer on this thread. May be I posted it on wron thread, or it was erased, I saved no copy of my original answer , so I am doing it again as a fresh reply. [QUOTE who="Eric"]<quoted text> Start with the one I already gave you. A Hebrew word has been changed. Tehilim 22:17 Chabad version

17. For dogs have surrounded me; a band of evildoers has encompassed me, like a lion, m hands and feet. Psalm 22:16 RSV version

16 Yea, dogs are round about me; a company of evildoers encircle me; they have pierced my hands and feet [/QUOTE] Ans. Now you have asked me to comment on which of the two translations is correct.

I am not a Hebrew Scholar, so I cannot say which of these two translations are according Hebrew words. The key word is Lion in Jewish Translation and Pierced in Christian translations. Obviously the Christians wants to re-enact the scene of Crucifixion by these verses. But according to Quran there was no crucifixion, so their whole efforts goes waste.

209 of 277

Questions and Answers with Non Muslims Vol-8

Q# 201

Time

[QUOTE who="Petesake"]<quoted text> For once I find myself agreeing with aspects of this poster's offering but restricted mainly to his talk about Time in relation to the Lord God, Great Master of Creation. [/QUOTE] Ans. Thank you for agreeing, but why only in relation to time. Can you tell me how these basic laws of Thermodynamics and Physics originated? By themselves? When you look deep down, you cannot fail to realize that this Universe could not have existed had it not been for precise calculations and fine tuning that was provided by the Creator. Calling it just a matter of chance is too much to say about chance.

But somehow people are programmed to think that "any thing done by God" has to be instantaneous and must violate all laws of physics and nature..... but if we can explai any thing by the laws of physics (like rotation of planets along the sun and moon circling along the earth etc...) then it cannot be attributed to God. (It happened due to science?) This basic mistake has made so many atheists of level-1 and level-2 scientist. But all Great scientists I will call them Level-3 and above and those who know the limitations of science, all believed in existence of God.

But their concept of God was different from what is mentioned in Bible....it is sad that because of bias and prejudice they did not look at the concept of God as mentioned i Quran, otherwise they would have been the first to recognize it as Universe's Creator And sadly the world is filled with level-1 and Level-2 scientists, who are heading this atheists movement and providing it with "scientific proofs"

210 of 277

Questions and Answers with Non Muslims Vol-8

Q# 202

Tolerance

[QUOTE who="Tall Man"]

01.You say that you are convinced that Islam is the only religion which is acceptable in th eyes of our creator.

02. You certainly have a right to believe this but what will you do with your fellow huma beings who share the earth that believe something else? 03. What about the Christian, Jew, Buddhist, Hindu or whatever who thinks that his religion is the only acceptable religion? 04. Does mankind fight and kill each other over it or do we strive to live in peace and tolerate our differences? [/QUOTE] Ans.

01. And I am not saying it on MY authority. I am saying it on Authority of the Creator of this Universe!! It is recorded in Quran:

The only DEEN (way of life) acceptable in the sight of God is Islam (i.e. submit yourse to your Creator) (3:9) And If anyone desires a DEEN (way if life) other than Islam (i.e. submission to Creator), never will it be accepted of him: and in the hereafter he will be in the ranks of those who have lost (3:85)

02. Yes we have a right to believe in this and we care least what other people of the earth may think of our belief. We are entitled to our own belief, and as I told you, it is based on clear unambiguous statement from our Holy Book Quran.

03. Certainly Jews, Hindus, Buddhists and Christians are entitled to have their opinion th THEIR religion is the ONLY one acceptable in the sight of God, but they have to bring their proves!!

In the hereafter God will certainly ask them, why you rejected the religion of Islam and where in your Holy Books it was written that they were final and only religion acceptable to Me.
211 of 277

Questions and Answers with Non Muslims Vol-8

Q# 203

Tolerance

[QUOTE who="Wondering"] Surah 5:54 The Abdullah Yusuf Ali translation states: Oh ye who believe! Take not the Jews and the Christians for your friends and protectors: ... and taken from the Muhammad Asad translation, the verse states O you who have attained to faith! Do not take the Jews and the Christians for your allies: ... However, in his commentary Sheikh Imran Hosein details an error in these translations: "The original Arabic text of the verse does not state: they are but friends and protectors each other(Yusuf Ali), or Jews (not all Christians and all Jews)." http://www.imranhosein.org/articles/understan... [/QUOTE] Ans.

You are new to this debate and you do not understand what the background is, let me brie you about it and THEN I will comment about Translation: A. Background to the present debate:

1. It started when some one mentioned that Jews believe in Tanakh and not Bible. I knew that Bible consists of OT and NT and OT is supposed to be the Jewish Scriptures. 2. The word Tanakh was new to me, so I asked what it is, some one told me that it is an Acronym and is spoken about a group of books.

3. I then went to Jewish site and checked the list of the books; they seemed same as OT o Protestant Christians. So I said that Tanakh is nothing but OT of protestants.

4. This was hotly contested by some Jewish Scholars on this thread and they said that I do not know what I was talking about. So I rechecked each book of Tanakh and OT and said that they are almost same as OT books.

5. Then they said that the problem is not in the form but in the translation, so I started checking verse by verse translations of Jewish Tanakh and Christian OT books and I find them to be same thing, with different choice of words.
212 of 277

Questions and Answers with Non Muslims Vol-8

Q# 204

Tolerance

[QUOTE who=" "] [/QUOTE] Ans.

[QUOTE who="abbey"]<quoted text> 100% = 10 out of 10 to you, bringing such bright radiant truths to this forum, of course we know that it is not from you but He , Himse the one who Guides Man Kind unto All Truth, The Creator Of all us. Also a fact uttered by the noble JESUS pbuh. showing to the world that he is not the last of revelations , there are much more facts still to come , brought to the world by anothe form , and that can only be by the Miracle of the Quran. : "I have many things to say to you , but you cannot bear it now." the time obviously was not conducive . Muq again thank you , thank you again . All strength to you and sabr katheer .[/QUOTE] Ans. Thank you brother for your kind words, I am touched and impressed. These are posts which give me encouragement to continue, otherwise it is generally the abuses and insults which people write back to me. I am not on these threads to get any personal gain or influence people. I write which I believe to be truth and I mention my reasons also. It is for the people to accept it or reject it. I at least have done my part.

If you really think about it, Muslims are the ONLY people on earth who really respect an Honor Jesus the Christ. If you raise someone's status to God, you are in fact insulting him in front of God and he will be questioned by God, as to did he ever made such claim or even gave a hint of making that claim.

These people by raising Jesus to level of God and using such phrases as God of God, Lor of Lord, One thru whom every thing was made etc..... is going to create problem for jesus in the Hereafter.

And Quran clearly mentions that God will Ask Jesus "O Jesus, did you ask your follower to take you or your mother as gods?" Can any one imagine the condition of Jesus, when God asks him this Question? Like a True Prophet of God, he will say:

"Glory be to you! How could I have said such a thing for which I had no right, I only said
213 of 277

Questions and Answers with Non Muslims Vol-8

Q# 205

Tolerance

[QUOTE who="Sentinel"] You asked earlier why Christians and muzzies can't get along. Your post above answers some of that question. Islam is the worlds biggest false devilish cult. [/QUOTE] Ans. I told you the shortcoming of devilish cult title being pasted on Islam. 1. No devilish cult would start with opening statement cursing Devil. 2. No Devil cult would ask its followers to worship only One True God of the Universe. 3. No devilish cult would ask its followers to respect all past prophets and their books. 4. And no devilish cult would ban alcohol, drugs, gambling , dancing and porn shows.

5. And no devilish cult would recommend strong punishment against theft, murder, rapes and high way robbery

6. And no devilish cult would ask its followers to be respectful to their parents and kinfol and fulfill their trusts and treaties and judge with justice and equity between people o different faithetc..etc. Sir, may I say, your slip is showing!!

214 of 277

Questions and Answers with Non Muslims Vol-8

Q# 206

Tolerance

Someone had challenged me to show me one example when Muslims have dealt lenient with weaker forces. I gave one such example from the life history of our prophet to how he pardoned most people of Makkah when he conquered it by Sword in 8 AH. Now I will give some more examples from the life of our prophet himself, how he dealt with these Weaker forces.

Because it is prophet whom these people accuse of maximum cruelty and bias and that Muslims have taken this trait from the life style of their prophet, who was Extremel hateful towards all Non Muslims and would offer people no choice, Islam or death. A. Prophets dealings with Jews of Madina and Arabia:

I think every one is convinced that it is towards Jews that prophet and Muslims have mos hatred. Let us see how our prophet dealt with them. Please remember that Jews wer always a weaker force in Arabia so no one should be under impression that prophet dealt with. The following Jewish tribes were living in Madina and Arabia during the time period of our prophet. 1. BANI QAINUQAH 2. BANI NADHEER 3. BANI QURAIZAH 4. KHAIBAR 5. FADAK 6. WADI TEIMAH

The first three lived in Madinah and others were scattered in the Northern part of Arabia. When prophet came to Madinah, one of the first act he did was to make a written pac of cooperation and non aggression and mutual defense with these three Jewish tribes But they broke their pacts at different times and the prophet had to take action agains them. Let us see what he did to each of them. 1. BANI QAINUQAH : Prophet asked to vacate Madina and go elsewhere when they broke their pact with Muslims. No one was killed and they took all their belongings and arms and every thing. This happened in 2 AH. 2. BANI NADHEER :
215 of 277

Questions and Answers with Non Muslims Vol-8

Q# 207

Violence

[QUOTE who=" "]

"What do Australians think of Palestinians boys, girls, women and old men being killed b the bullets of Jewish army and police and civilian Jews every day?"

We don't like it, and we are not use to it as we have a very peaceful land, that our grandfathers fought and died for. We are also concerned about very young girls being sol then married to people old enough to be there grandfathers in addition to that we also don like young boys being picked up off the street taught to dance for old Muslim men the engaging in sexual acts against God. Think you should voice your concern about this. If your muslin people were serious about peace, and some silly preacher in the USA decided to burn your holy book, you would say Ok so what, water off a ducks back, just like the peaceful people of Australia would. This is one act by one radical extreme religious christian pastor.

But this is not what happened, the riots and the killings of human beings in supposed retaliation for the burning of the Koran. This is also heinous behavior and is not in any way, shape or form justified by the burning of the Koran. In civilized society, one does no kill people for disrespecting or destroying the symbols or instruments of ones faith.

MUQ do you think the USA's pastors burning of your holy book justified the cutting off o the United Nations womens head in Afghanistan ? [/QUOTE] Ans.

The problem with western people is that they do not try to understand the Muslim psyche and Islam and Muslim society.

They think that their is the only society and their are the only rules and every one shoul be governed by their rules and their laws.

They are extremely insensitive and callous to the feelings of other people and other socie Islam is a major world religion and by this time every one on the earth should know how much Muslims love their religion and their prophet and their Quran.

And by this time, every one in the earth should know that in Islam it is prohibited to mak photo or image of Prophet Mohammad (peace be on him) and to make any cartoon or thru abuses and insults on him. Now in the most civilized nation and Only Super Power of the world, a lunatic Christian declares openly his plan to burn the Holy Book of Islam , the Notice is given in advance.

216 of 277

Questions and Answers with Non Muslims Vol-8

Q# 208

Violence

[QUOTE who="SS"] 1. Could it have something to do with Islamic suicide bombers, and Muslim-on-Muslim murders in Darfur, and Egypt, Libya, Yemen, Syria, Jordan, Lebanan, Gaza, Saudi Arabia, Bahrain, Iran and Iraq? 2.

You haven't yet figured out that Islam hasn't evolved at all from the 1400s?? Whereas Judaism has, and Christianity is trying? How can Tutu make any other decision base on the most recent history of Islam. If Islam wants to claim the same god, perhaps it' time for the imams to get into this century and quit calling for murderous jihad. [/QUOTE] Ans. 01. There are no Islamic suicide bombers.there could be Muslim suicide bombers and there is a lot of difference between the two. Islam is against killing of Innocent human beings, be they Muslims or Non Muslims. In Quran the harshest and biggest punishment is for Muslims who knowingly kill any other Muslims. So these Muslim suicide bombers who kill other innocent Muslim knowingly should decide their own place in front of Allah. 02. It is not a bad point but a good point that Islam has not at all evolved in past 1400 years. Because when a thing reach perfection, the Only evolution for it is to go down!!

Jews have evolved because they had no choice for them. Their religion became unpractic and could not be practiced in the changing times.

Christianity is trying to change because bulk of its followers have renounced it and they a using it only as a label. It is trying its best to walk on two horses.be close to God as well as justify Homosexuality, free Sex, Alcoholism and all the ills that ail Christi society.

Islam was declared to become perfect, on the day of Last Pilgrimage of Prophet, when th verse was revealed:

This day I have perfected your religion for you, completed My favors on you, and hav chosen Islam as your religion (5:3) No such verse is available in any World scriptures. And prophet said Any one who adds (or subtracts) any thing in Islam, which I have not
217 of 277

Questions and Answers with Non Muslims Vol-8

Q# 209

Why Islam

[QUOTE who="Sameer"]

Mr. MUQ, Please answer these kafirs. They are creating doubt in the greatness of Islam. You are now the only savior of Islam. Please tell them the 10 good points about Islam If it was asked to me, I would have answered it. But please answer them. Quran

O Prophet! Strive hard against the unbelievers and the Hypocrites, and be firm again them. Their abode is Hell,- an evil refuge indeed.(9:73, 66:9) Therefore listen not to the Unbelievers, but strive against them with the utmost strenuousness.(25:52)

And who does more wrong than he who invents a lie against Allah or rejects the Tru when it reaches him? Is there not a home in Hell for those who reject Faith? And tho who strive in Our (cause),- We will certainly guide them to our Paths: For verily Alla is with those who do right.(29:68-69) [/QUOTE] Ans. The purpose of KAFIRS is to create doubt about Islam, Quran, Prophet, his life, his conduct , his character and why should we be alarmed by that. If that was not the case why they would be KAFIRS? They will become Muslims. A. Kafir and Non Muslim:

By the way there is difference between a Non Muslim and a KAFIR. Non Muslim is a general term for people who is not a Muslim.KAFIR on the other hand is a person who received the message , understood it and then rejected it.

So every KAFIR is a Non Muslim, but every Non Muslim is not a KAFIR. Most of Hind living in India are Non Muslims, but very few are KAFIRS, because we Muslims hav not delivered the message of Islam to them. B. Quranic verses in their proper context:

Now coming to these Quranic verses, these verses have come in certain context and unles you see the Quranic verses in their proper context, you will derive wrong meanings and wrong teachings from them.

I take you one example, in GITA Lord Krishna asks ARJUN to stand up and fight agains
218 of 277

Questions and Answers with Non Muslims Vol-8

Q# 210

Women

[QUOTE who="Sara"]

What happens MUQ if a Muslim woman is not able to have to children? The man marries another and divorces her. At least the Hindus realise it is NOT always the women who is unable to have childr and at least the woman is not encouraged to leave her sterile husband. I don't mean to sound like i am digging at your religion but can't stand it when peopl poke at others beliefs, implying that theirs is perfect. [/QUOTE] Ans. I am glad you asked me this question. 1. First of all, in Islam Children are deemed as gifts from God and His mercy and blessings to the human couples. It is God who decided which pair has to have how many children.

2. Nevertheless, every couple should try to have as many children as they can get thru the normal sexual life. Usually most of couple would get children in the normal way, as is normal law of God for human kind.

3. However, if due to any reason, the couple do not get any children, they could go to doctors and find out, if there is any medical problem. If there is any medical problem with either husband or wife, they should get the treatment.

4. If it is found that there is some problem with either of the two spouses, the couples hav a choice. If they want to live as husband and wife, without any children, it is OK.

5. If man wants to have children and the woman has some medical com placations, he ma get a second wife and get children thru her. He may keep his first wife also, but if sh wants divorce, she will be granted her request.

6. If the problem is with husband, then women can ask for divorce and it will be granted t her, thru Islamic courts. 7. Islam therefore has a very practical and reasonable approach for this tricky problem. PS:

My mention of Hindu system of NIYOGA was not to criticise Hinduism. But on the face of it, it is immoral and in introduces a unknown element between sexual relation of husband and wife.

||And then this system can be misused, and is being misused, because of religious sanctio
219 of 277

Questions and Answers with Non Muslims Vol-8

Q# 211

Women

[QUOTE who="Mahmood"]

01. You need to quit claiming god of the Koran aka allah, as the almighty creator of this universe. Neither you nor any other muslim has been able to prove this, so dont even go there. What you believe and what is reality are not one and the same.

02. : Irrelevant - we are talking about brains not brawn and value of human life. Obviously women lack the upper body strength as a result they do not compete again men.

03 : What do you mean by "fake gender" equality? Why should laws apply differentl to men? You guys claim that Islam treats women as equals, but the sad truth is that it does not. It's laws and edicts are based on the 7th CE Hijazi mindset.

Read the Koran, and the verses are primarily patriarchal. Does the Koran specify the rewards awaiting the devout female muslimah? 78:33 - "And voluptuous women of equal age". [/QUOTE] Ans.

01. Why should I disregard Allah as sole Creator of this Universe? Is there any one else? Please give me His name!! 02. You need both brains and brawn to evaluate some one. And that is what Allah has done, otherwise, why Allah did create males and females? Why He did not made humans as Unisexual to have no problem at all? 03. Fake gender equality is what you people are putting forward to bring males and females in direct conflict and creating havoc with the society.

Muslim women will get reward what they like best! And any good and pious women wha she likes best and what she likes would be given in the heavens. As what those whores, strip tease joint hostesses, lesbians and live show formers like bestyou can visit them in hell and see what they are getting. Paradise and heaven is the place for good and pious people, not for scum of the society.

04. Quran is neither patriarchal not matriarchal , it is revelation from Allah. Allah mentio rewards for men, because they are incharge of family and says that equal rewards sha be given to women also.

220 of 277

Questions and Answers with Non Muslims Vol-8

Q#21 Women 2

[QUOTE who="Mahmood"]

I've been told that the same law applies in murder cases as well. How true that is, I do not know. Even if it's limited to financial matters, this goes to show that "Allah" does no have confidence in women as he has in men. Zakir Niak says men are more adept in financial matters than women which is not quite true. Muslim scholars make all kind of excuses in this regard such as, emotional condition of a female is not to be trusted women can get confused etc etc. The bottom line is that Islam discriminates & is conceptually patriarchal. The blood money paid for the murder of a female is half of that paid for murder of a man. [/QUOTE] Ans. 01. It is not the question of Allah having less confidence in women in Financial MattersAllah KNOWS what is the best role for female.

He is the Creator and He knows the physiology and psychology of fames and males and i is He who has made the division of roles. What Dr. Zakir Naik has said is true if we take account of all males and females on this earth. Just having a few women here and there will not change the general rule.

Muslim scholars try to find justification as to why Allah has ruled such, they might be tru or they might not be true.but things stay as Allah has decreed. This is the philosophy of Islam 02. Islam does discriminate and every system does discriminate..I have yet to see sports in which men and women compete against each other, athletics, boxing, football.almost every sport, why do you not hold mix competitions. Why males have been given more physical powers than women? You just cannot side track this issue and keep on talking about fake gender equality.

Islam discriminates based on just and natural grounds. There is no stigma on women taki care of home and children. The family and house is as important as earning bread for the family.

The western system has muddled the issue, with the result that women are overburdene They have to work on both the fronts, and since they cannot cope up with both tasks, they have to pay less attention to one.

That that is the reason for broken homes, sky rocketing divorces and single parent homes and domestic violence, mental sickness and every problem that we see in western
221 of 277

Questions and Answers with Non Muslims Vol-8

Q# 213

Women

[QUOTE who="Frijoles"] Hey MUQ I am still waiting for your Quranic justification for locking women up in the home. I hope this isnt yet another case of "Allah Knows Best". I suspect that this is merely a tradition, not a Quranic command, but either way it makes your behavior appear barbaric. You would benefit from a few woman Imams. [/QUOTE] Ans. A. Quranic guidance that a womens place in her house: 01. It is mentioned in Surah AHZAB, (Ch. 33 V 33)addressing the wives of prophet and stay quietly in your houses and make not a dazzling display , like that of the former times of ignorance. Since wives of Prophet are role models for other Muslim ladies, this could be viewed as a general order also. 02. In the same Surah AHZAB (Ch. 33 V 59) , it is mentioned:

O Prophet tell your wives and daughters and to believing women that, they should cast their outer garments (JILBAB) over their persons (when abroad), That is most convenient , that they should be known (as such) and not harmed (That shows that a women should not go out of her house, unless there is a pressing need, and even then she should cover her body, so that it is not exposed to gazes of strangers). 03. It is mentioned in Surah NOOR (Ch. 24 V 31) , advising them of Hijab: And tell to the believing women, that they should lower their gaze, and guard their modesty.. and they should not strike their feet in order to draw attention to their hidden ornaments..

That shows even the sound of their ornaments should not be heard by strangers this impli that they should not go out unless there is pressing need. 04. It is mentioned in Surah NISAA (Women, Ch 4, V 34):

Men are the protectors and maintainers of women, because Allah has given the one more (strength) than the other, and because they support them from their means.. That shows that a women is not burdened to earn her living, it is the duty of her husband
222 of 277

Questions and Answers with Non Muslims Vol-8

Chapter-9 On Jews and Related Topics

223 of 277

Questions and Answers with Non Muslims Vol-8

224 of 277

Questions and Answers with Non Muslims Vol-8

Q# 214

Circumcis ion

[QUOTE who=" "] [/QUOTE] Jewish groups oppose circumcision ban in US city by Hannah Dreier Hannah Dreier 1 hr 24 mins ago SAN FRANCISCO (AFP) Jewish groups and others are up at arms over an attempt to outlaw male circumcision in San Francisco by putting the issue to a popular vote. Self-described "intactivist" Lloyd Schofield has been collecting signatures for a voter initiative that would criminalize infant circumcision in the Californian city.

After two months of collecting names, he claims to be more than half way toward getting the 7,168 signatures he needs by late April to put the matter on the November ballot.

Schofield and a growing community of anti-circumcision activists say that infants should not be forced to participate in what is essentially culturally accepted genital mutilation. They claim that the procedure can cause health risks and diminished sexual function and compare it to the clitoridectomies performed on girls in parts of Africa. "This is a human rights issue," he said. "What you're doing is you're taking an infant and removing the most sensitive part of their body."

Jewish organizations have pledged to fight the measure should it be placed on the ballot. Anti-Defamation League director Daniel Sandman called Schofield's effort discriminator and misguided. "This is hurtful and offensive to people in the community who consider this a coveted ritual," he said. Abby Porth of the Jewish Community Relations Council charged Schofield with wasting city resources for an inappropriate political stunt that was unlikely to become law. "This is one of the most fundamental practices to our tradition of over 3,000 years," she said. "It's symbolic of our covenant with God."

Porth said the Jewish community would form a coalition against the initiative with medic professionals and Muslims, who also practice circumcision.

"It's very similar to those of the Jewish faith," said Omar Nawaz of the Bay Area-based Zaytuna College, one of the nation's only Muslim colleges. "It's a religious tradition and i important for us." Both pro- and anti-circumcision advocates make health claims, but the medical research
225 of 277

Questions and Answers with Non Muslims Vol-8

Q# 215

Ishmael

[QUOTE who="Ex-Mess ]

Righto. When Ishmael was blessed, his numbers became numerous and he needed a lot o territory, so now he has 57 nations. When Isaac was blessed, his name was changed to Israel and the twelve tribes and they were given one small land. So why does Ishmael not be thankful for all his blessings and leave Israel alone?

[/QUOTE] Ans. You do not know the story of Ishmael and Isaac in its totality. God did promise the land of Canaan to Abraham and his descendants after him. So Ishmael and his children have as much right on the land of Canaan as the children of Isaac (if we speak strictly thru the book of Genesis).

When the Children of Isaac were strong enough to defend their territories..the children of Ishmael did not interfere with than land

However when the Abraham legacy well into the hand of others (Romans, Greek and Pagans) and they drove out children of Isaac from the promised lands in 70 AD..and 60 years passed and there was NO WAY in which Children of Isaac could come back to take their lands from squatters.

It was then that children of Ishmael, decided to re-possess their birth right.they drove o all those foreigners from the land and cleansed it from all outside interference and they toll possession of 100 % of all lands , which feet of Abraham, Ishmael, Isaac, and Jacob had touched during their entire lives!!

So Muslims indeed did a favor to Jews by re-claiming the Land of Palestine from Roman Empire.

Then Muslims never claimed sole possession of that holy landbut they maintained peace and tranquility in holy land for 1300 years and they allowed Jews to come and live these lands as and when they wished.

But Jews tied to repay the same hospitality..after 1917, when they termed their elder cousins as Squatters and forced them into exiles with the help of same outsiders wh were responsible for their exile in 70 AD. What a strange world we do live in!!
226 of 277

Questions and Answers with Non Muslims Vol-8

Q# 216

Palestine

Biiguy: That is true. Unfortunately, fire first is what the Palis do to the Israelis. Then, when the Israels finall shoot back, they're accused my many, includng you, it seems, being 'aggressors'.

MUQ: This taking of sides makes one biased.... first of all these Palestinians have genuin problmes, and Israelis have not made any efforts to solve them.

So instaed of taking sides and giving "clean chit" to one party and putting "all blame on other party, we should go to the root of problem and try to solve that. And this "balanced attitude" is missing in most posts, including your above post. BIIGUY:

Unfortunately, Israel has made many efforts to aid the Palis, and made many offers. the only result of Israeli concessions has been more demands & more terror. Never concessions to the Israelis. As a semi-outsider & do take a balanced view. To those f whom the Palis are never wrong & the Israelis never right, this does seem 'unbalance but it's still the truth. MUQ:

They never talk to solve "the basic problems"... all their offers of talk are "for camera" an a sort of "cosmetic surgery". That is why there is never any "real progress" in talks. Before 1952 it was to return to "1947 Borders". Then in 1956 it was return to "pre-1952 borders" Then upto 1967, it was return to "pre 1956 borders". Then post 1967, it was return to "pre 1967 borders".

Now talking of 1967 borders is a "four letter word to Israeli Military and politicians" These are the "peace terms and negotiations on which Israeli wants Palestinians to agree". And this game is being played since 1917, Jews getting more and more and Palestinians asked to concede more and more. And do you think any "real peace" is possible in this climate of take and take by one party and give and give by another? BIIGUy:
227 of 277

Questions and Answers with Non Muslims Vol-8

Q# 217

Scripture To: Frijoles If you ask any "LEARNED" Christian, he will say that this OT is nothing but "Jewish" Scriptures, And if you ask "More Learned" Christians, he will say that OT is same which is going around by the name of "Tanakh" these days by Jews.

And if you ask "Extremely Learned Christian"....he will say, most of OT and Tanakh boo are written , copied and edited by "unknown hands"

And if you ask "Common Bible Thumping Christians", he will say that Entire Bible from Genesis upto revelation is nothing but "Inspired Word of God".

So, you can have your pick of Christians to whom you ask the question and let us be done with it.

228 of 277

Questions and Answers with Non Muslims Vol-8

Q# 218

Zionism

[QUOTE who="JeJ"]

01. The history of these events is well understood. When political Zionism began in earne in the late 19th century, there were only about 15,000 Jews in Palestine.29 In 1893, f example, the Arabs comprised roughly 95 percent of the population, and though und Ottoman control, they had been in continuous possession of this territory for 1300 years.30 Even when Israel was founded, Jews were only about 35 percent of Palestines population and owned 7 percent of the land.31 02. The mainstream Zionist leadership was not interested in establishing a national state or accepting a permanent partition of Palestine. The Zionist leadership was sometimes willing to accept partition as a first step, but this was a tactical maneuver and not their real objective.

03. As David Ben Gurion put it in the late 1930s,After the formation of a large army in the wake of the establishment of the state, we shall abolish partition and expand to th whole of Palestine.32

04. To achieve this goal, the Zionists had to expel large numbers of Arabs from the territory that would eventually become Israel. There was simply no other way to accomplish their objective. Ben Gurion saw the problem clearly, writing in 1941 that it is impossible to imagine general evacuation [of the Arab population] without compulsion, and brutal compulsion.33 Or as Israeli historian Benny Morris puts it, the idea of transfer is as old as modern Zionism and has accompanied its evolution and praxis during the past century.34

05. This opportunity came in 1947 48, when Jewish forces drove up to 700,000 Palestinians into exile.35 Israeli officials have long claimed that the Arabs fled because their leaders told them to, but careful scholarship (much of it by Israeli historians like Morris) have demolished this myth. In fact, most Arab leaders urged t Palestinian population to stay home, but fear of violent death at the hands of Zionist forces led most of them to flee.36 After the war, Israel barred the return of the Palestinian exiles. 06. The fact that the creation of Israel entailed a moral crime against the Palestinian people was well understood by Israel s leaders. As Ben Gurion told Nahum Goldmann, president of the World Jewish Congress,

If I were an Arab leader I would never make terms with Israel. That is natural: we have taken their country.... We come from Israel, but two thousand years ago, and what is that to them? There has been ant Semitism, the Nazis, Hitler, Auschwitz, but was tha their fault? They only see one thing: we have come here and stolen their country. Wh should they accept that?37 http://www.lrb.co.uk/v28/n06/john-mearsheimer...

229 of 277

Questions and Answers with Non Muslims Vol-8

Q# 219

Angels

[QUOTE who="Killuminati"]

In Hebrew language "angel" is "mal'ach" . It is written and sounds almost the same to the word "melacha" which means "craft" "doing" . So when we look at angel from that angle , it can get totally different meanings . I haven't mentioned some of their names and what each name means . "Gabriel" (in quran "Gibril") is just a name for you guys , like the names Bill,George and Jimmy but in Hebrew , names have meaning and sometimes more than one . [/QUOTE] Ans. Thanks for your info, you know Arabic and Hebrew are sister languages.

The term for Angels in Arabic is Malak (and its plural is Malaikah) the term Malak is used for prophet also, so the Book of Malacchi in OT collection means My Prophet and it gives mostly the qualities of the last and Final prophet.

If Malak in Hebrew means craft or doing, there is nothing wrong, because angels act and do manage the affairs of the Universe as commanded by their Creator. Quran says about these angels They do not disobey Allah and do what they are commanded I think Michael and Gabriel are mentioned in OT books also. Most of the Arabic names also have meanings just like in Hebrew. But proper names are proper names, they do point out to a particular individual, man, angel of Jinn!! Thanks for your info once again.

230 of 277

Questions and Answers with Non Muslims Vol-8

Q# 220

Anti Semite

[QUOTE who="Rick Moss"]

1. You know PRECISELY what this word means and where it comes from. I'll answer th for others who might not know.

2. The word "anti-Semite ", while not anthropologically accurate, has been used for over 100 years to describe collected hatred specifically against Jews. The term was n coined by Jews at all -- but by Germans.

3. These German writers used the term "Semite" to refer exclusively to Jews in a pejorative manner. The purpose of the term was to highlight the non-European origin and customs of Jews and Judaism to isolate them and target them for abuse. 4. Because of it's popular usage, it has achieved the definition that it has today. Changing the meaning of a common usage term doesn't alter the fact that specific hatred of Jews as opposed to other Semitic peoples exists. To pretend that it does is ingenuous..

[/QUOTE] Ans.

1. I thank you for pre-judging me!! I told in the start with that the name is a misnomer an the word does not convey what is actually says. 2. If the name Anti Semite is not correct, what prevents us from correcting it now? Why you want to persist with the mistake? It does not matter if the term was coined by Germans or Australians or any one else, a mistake is a mistake and once brought to our notice, we should correct it. When the hate is against Jews, why hide the word Jews and cover it up with term like Semite? Arabs are also Semite and they outnumber Jews by many orders of magnitude, why they should be removed from this protection?

3. What Germans writer did is neither here nor there. We have outgrown every thing that those Germans did, why we want to persist with this Anti Semite terminology? If those German writers did mistake, we should correct it rather than persist with it? 4. Have you ever thought why people should systematically hate only Jews? What are Jews own contribution to cultivate that hate?

Why of all people and races to choose from, only Jews are singled out for this hate..the
231 of 277

Questions and Answers with Non Muslims Vol-8

Q# 221

Anti Semite

[QUOTE who="Frijoles"]

. "Jews link with every thing subversive to world peace, is well known. So there is nothin to know much about their involvement." That is an anti-Semitic comment. I guess you just blew your theory.

1. If according to your own logic conquerors have the last right - and the wave of 1880s Zionism led to the settlement /"conquering" of the land, why is Israel a Muslim land again?

And yes, a few posts ago you referred to Israel as "Muslim land" - after I asked you about Muslim government, and you said it was appropriate for Muslim land only, an then listed all the lands.... [/QUOTE] Ans. 1. I told you I do not agree that there is anything called Anti Semitism. The term is a misnomer, first properly define the term and then use it. If it is OK to paste hate message against Arabs and Muslims, why it is wrong to post a hate message against Jews and what Semitism has to do with it? Are Jews the ONLY Semites left in the world? 2. The Only correct argument for Jews getting the land is might is right

I have NO OBJECTION if instead of giving all those logics and reasons justifying Jew claim over Land of Palestine, they only say one thing We took is by force. But this is a double edged sword! If they took it by force, others also have a right to take back by force.

So let the struggle continue and see who is the winner in the end. (And going by their pas histories, it is the Jews who would be the ultimate loosers).

I do not know why, perhaps you could give the reasons, why Jews are always the loosers They seem to win every battle and in the end it transpires that they have lost the war! It happened in 580 BC , it happened in 70 CE.could it not happen again?

232 of 277

Questions and Answers with Non Muslims Vol-8

Q# 222

Billionaire [QUOTE who=" "] s [/QUOTE] I'm investigating Jewish Democrat billionaires. Help me out here. How are your boycotts coming, LimpRider? Jewish Billionaires Lawrence Ellison Net Worth:$25.0 billion Sheldon Adelson Net Worth:$20.5 billion Sergey Brin Net Worth:$18.7 billion Larry Page Net Worth:$18.6 billion Michael Dell Net Worth $17.2 billion Steve Ballmer Net Worth $15.2 billion Carl Icahn Net Worth $14.5 billion Michael Bloomberg Net Worth:$11.5 billion George Soros Net Worth:$9.0 billion

233 of 277

Questions and Answers with Non Muslims Vol-8

Q# 223

Chosen people

[QUOTE who="arbitrageur"]<quoted text> So, who are God's chosen people?[/QUOTE] Ans.

The God's chosen people are really the prophets and messengers of God and any one who follows in their footsteps.

There is no label like "I am God' Chosen people" which any one can put on his or forehea and march towards heaven.

God look at our hearts and our deeds and our intentions and he does not look at our bodie or our genealogies or the color of our skins, or the languages we speak or the countri or race we are born into. This is my understanding on who are God's Chosen people. Allah Knows Best

234 of 277

Questions and Answers with Non Muslims Vol-8

Q# 224

Egypt

[QUOTE who="Resonator"] Hi MUQ, not quite sure why my post has upset you so much - it is simply the historical TRUTH.

Current research strongly suggests that Judaism is a 'ritualized normative inversion' of the Egyptian religion at the time the Jews were enslaved there. Pristine Judaism was based on animal-slaughter & immolation in the Temple ie. 'burnt offerings' to Yahweh. Whether you like it or not; that is Judaism Mk-I; Christianity is Judaism Mk-II; Islam is Judaism Mk-III.

I am happy to concede that in many respects the theology of Islam is less nonsensical tha the theology of Christianity. Both however are derivatives of Judaism Mk-I. [/QUOTE] Ans. No I am not upset, I am amazed at the creative minds of these scholars and the way they find the missing links between these Mark-1 and 2 and 3!! We on the other hand are much advanced than you scholars and believe that All religions are related to each other and not only that they are essentially the same!! All the old religions were teaching the same fundamentals, believe in One God, follow your prophet and prepare for the Last day!! There was slight local color depending upon the country and prevailing customs.

The Big Deviation came only in Christianity when they introduced the concept of Trini and God Own Begotten Son and some one dieing for the sin of whole mankind etc. These concepts have nothing to do with religions of the old prophets and was from the Greek and Roman Mythology. But the way your scholars trace the link and make mountains out of molehills and smallest similarity in rituals is taken as ultimate proof of copying and borrowing is really amazing.

Your scholars have spent too much time in Copyright and Patent departments and they fail to realize that entire human race is after all descendents of same father and mother
235 of 277

Questions and Answers with Non Muslims Vol-8

Q# 225

Israel

[QUOTE who="Seaside Soon"]Quran confirms Israel belongs to the Jews, but most Muslims don't read it...Khaleel Mohammed Mohammed attracted attention for a 2004 interview in which he stated that Sura 5 verse 2 of the Qur'an, and the medieval exegetes of the Qur'an, say that Israel belongs to the Jews. He translates it thus: He states that he was once accused of being racist, for having stated that "95% of contemporary Muslims are exposed to anti-Semitic teachings".[3] [/QUOTE] Ans. A. Quran and Jews claim on Palestine: I have seen this one sided view of Quranic confirmation that God gave land of Palestine to Jews, but this is not a full story!! Quran refers to Jews and Palestine in many places and not only in Chapter 5, it has more say on this issue in Chapter 17

You are right, Allah gave Palestine to the Jews..... But not for an "Everlasting possession"..... It was a conditional offer.... So long as they remained true and walked in the ways of God....they will prosper and they will be custodians of the land. God remained true to His promise....He gave them possession of the land and they prospered there. B. First Punishment:

But Jews started to "backtrack" on their part of promise and they broke the laws of God and they worshipped false gods and did every type of abomination in the land.....God was patient with them, and He sent many prophets to them to warn of God; wrath.....centuries passed but Jews did not heed to this warning.

Finally the time came for God wrath and He caused the Jews to be defeated by the Babylonian king Nebuchadnezzar , who destroyed the temple at Jerusalem and took Jews as captives to Babylon. That was their First Punishment. C. Reprieve: Then Jews repented of their crimes and inequities and after 70 years or so God again had mercy on them. Jews were removed from captivity by the Persian king Cyrus and they came back to Palestine and rebuilt their temple and started again walking in the ways of God.

236 of 277

Questions and Answers with Non Muslims Vol-8

Q# 226

Israel

[QUOTE who="Chuy"]

1. MUQ I know Israel is sometimes brutal and will build settlements in palestine .But the have been the only democracy there for a long time. (And it's hard not to love your own kind).We kind of have to support democracy , when we find it. 2.

You know MUQ, most who worship GOD. Will accept "The will of GOD." I think that after several wars w / Israel ,the Arab nations have done their very best already ,may a "jewish nation " is the will of GOD?

[/QUOTE] Ans. A. Israel and Democracy:

1. Democracy is not something which is inherently good and is no reason that you suppor any country which has some outer sign of democracy. 2. People should support truth, justice and equity, irrespective of whatever sort of Government those countries have.

3. And correctly speaking there is no real democracy in Israel as there is no real democra in USA. Just because people vote in general elections, does not mean that their rulers are chosen because of their free will.

4. In our Modern Party based democracy, you have to select one of the two idiots that a placed before you. Who selects them and how democratically they are elected, no on knows. B. Will of God and Creation of Israel: 1 . Nothing happens in the world except by the Will of God but no one really knows what is the Will of God. 2. Why Western nations continued their compaign against communism for more than 70 years and did not accept it as Will of God. The same goes for their struggle against Nazism and Fascism etc. 3. Do you remember Crusaders occupied Jerusalem for 99 years, then they were evicted from there. So we do not know what was the Will of God.

237 of 277

Questions and Answers with Non Muslims Vol-8

Q# 227

Israel

[QUOTE who="biiguy"]

They bought it from the local owners, be they Arabs, Turks or whatever. as to it's being a 'local Palestinian matter', why is the whole muslim world involved in it? Regarding usurpers, yes, the pali usurpers will be removed eventually... Hand of God, yes, since God promised the Jews that they will be punished for their sins, but that they will eventually return to their land. Now is the time. [/QUOTE] Ans. All Muslims in the world are involved in it (as a moot spectators) as Jews all over the world are involved in every Jew who is killed any where in the world. And the way every Christian in the world is involved when any Christian is killed in the world.

The whole Muslim world is watching the atrocities being committed against our brothers Palestine and the "double standard" world uses in dealing with Israel in every matter

For offenses on which they put economic sanctions, political blockade and even downrig attack are ignored in the case of Israel.

What I feel is that this is a test for both Muslims and Jews. For Muslims to learn as to wh it does to them when they become disunited amongst themselves and "request" the same USA and UNO who "created Israel and are their biggest political economic and arms suppliers" to "restrain Israel".

For Jews it is a test as to how they misused this opportunity which God them to get one state of their own. Instead of that being an Ideal Jewish state, it turned into a Materialistic Atheist state based on the principle of Apartheid from the very first day The atrocities they have committed and are committing is adding to their bag of inequity....when it tips over the limit....they are due for the same chastisement as was done to them on two previous occasions.

They do not have guts to invite all people to their religion and reason with people that the way is best and they should follow it, so that every one becomes followers of religion in their area.

Muslims are all the time ready that if these Jews become Muslim, they can have all the land of Palestine and much more. Islam is not the fight for any piece of land. It stand for principles. What is astonishing is that despite trying every thing possible, they are yet to break the spirit and resolve of the Palestinian people. It is growing stronger with each passing
238 of 277

Questions and Answers with Non Muslims Vol-8

Q# 228

Jerusalem

[QUOTE who="Wozza"] 01. I don't think this is the case, No peaceful god would have anything to do with Jerusalem as its a violent place of unrest. 02. During its long history, Jerusalem has been destroyed twice, besieged 23 times, attacked 52 times, and captured and recaptured 44 times. 03. A good judge of what people will do in the future have a look at the past.

04. Its absolutely pathetic and ridiculous how that lot over there carry on, fancy two race so close hating each other with a passion and then having there holy sites side by sid its the biggest joke of the world. [/QUOTE] Ans. 01. This could be your point of view about Jerusalem. But Jews and Zionists (and Christians before that during Crusade) are dieing to have possession of it. Even the US congress passed a Bill for US to move its embassy from Tel Aviv to Jerusalem!! 02. Yes Jerusalem has seen a lot of action.the place must have some attraction for people! Muslims have the maximum period of ruling it, undisturbed than Jews and Christians combined.

03. During Muslim rule only, Jerusalem was a city of peace and Muslims want to maintai it as such. But when Jews and Christians were its masters, it was a city that attracted all its enemies to that city. 04. You are right that two groups of people so closely related to each other are fighting over a piece of land is pathetic.. .But

More pathetic is the stand of onlookers, who are supporting and goading only one group t have a go at other. Their duty as a peace maker should have been to ask both parties get away from each other and decide with equity and justice in their disputes.

But they have taken side with one group and ad fuel to the fire by their one sided support

239 of 277

Questions and Answers with Non Muslims Vol-8

Q# 229

lobby

[QUOTE who="Matty"]

Yes... Israel does have a powerful lobby in the US.... SO does the arab world and their by spouting organizations like CAIR and ISNA. [/QUOTE] Ans. Jewish Lobby Versus Palestinian and Arab Lobby in USA: I know there is supposed to be something which is called Arab and Palestinian Lobby in USA, let us compare their strengths: 1. The Jewish lobby forced US President to get up from his bed at 1:00 AM in the night and recognize the Newly Created Israel of Israel in 1948, and in 62 Years Arab and Palestinian lobby is not able to convince USA that there is a place called Palestine!! 2. In past 62 years Jewish Lobby forced USA to veto more than 32 times in UNO any resolution that criticises Israel and Arab and Palestinian lobby is only wishing that some time US will vote for any resolution that is helpful to them.

3. Jewish lobby forced US House of Commons and Senate pass a resolution to ask US Govt. to declare Jerusalem as eternal capital of Israel and US to move its embassy there and Arab and Palestinian lobby is yet to get any US President to even know tha any Palestinian lives in Jerusalem.

4. Jewish lobby forced US to even discuss or acknowledge the involvement of Jews and Israel in accidents like USS Liberty, Spying on US Nuclear and Industrial secrets etc Arab and Palestinian lobby is not able to convince that they are not involved in these cases.

5. Jewish lobby forces every US president during election compaign to tie them in knots and growl before them and each candidate trying to excel each other in what favour h will do to Israel and Arab and Palestinian lobby considers themselves lucky if any U presidential candidate will give them five minutes time to them.

6. Jewish lobby in USA got them to cover up the true investigation of 9/11 incident and p all the blame of Al Qaeda and Taliban and despite all proofs to the contrary accept One Plane One Tower Theory Arab and Palestinian lobby could only babble Trut about 9/11 and got no listeners.

7. Jewish lobby forced USA to involve Iraq, in 9/11 incident and got USA to attack them 2003 and is all but forcing USA to strike against Pakistan, Yemen, Syria and Iran and Arab and Palestinian lobby just passed resolution that US should not attack any country after Afghanistan.
240 of 277

Questions and Answers with Non Muslims Vol-8

Q# 230

Messianic

[QUOTE who="ELH"]<quoted text> The most disturbing thing is that Messianic Jews seem top pick and choose and randomly practice which ever book works for them. And animal sacrifice? Really? It's the 21st century! Also I have two adopted children who where born into a crazy (and deeply f*cked up) "Messianic Jewish" cult. Hyper r" religious" parents the physically and mentally abused then (both in prison) and a sect that did so much emotional damage to them that it's taken four years to re-establish normalcy in our home. So (obviously) my feelings about this issue are very complex but no matter how I look at it, I just can't agree that anyone, even a decent and sincere person can actually be a "true" Christian and a "true" Jew at the same time. [/QUOTE] Ans. A. Animal Sacrifice: What 21st Century has to do with Animal Sacrifice? Have people stopped eating flesh and meat of animals in 21st Century? Do people not enjoy Chicken, Mutton and beef meat in your diets? And why killing of animals should only be inside confines of slaughter houses? Sacrificing Animal as part of religious rituals (if ordained in religious scriptures) is permitted and I do not find any fault with it. But sacrificing animals is NOT the Main job of religious scriptures or it is not the main purpose of any religion. The meat of animals sacrificed is eaten by poor members of community or even the household of person sacrificing the animal.

The real purpose of religious scriptures is to correct our relations with our Creator and ou fellow human beings and lead a pure and pious life, so we pass the test of this life an earn favor and blessings of our Creator and so that we can enter the paradise for an everlasting life. But as usual people are more concerned with the exterior things and ritual and forgetting the real purpose of religion and religious scriptures. B. Messianic Jews only follow Selected Portion of Law: If they do it, then they are obviously wrong. While no one can claim to be able to do 100 of scripture law into ones personal life, but we should not reject any part of it and try to fulfill best that we can do.

241 of 277

Questions and Answers with Non Muslims Vol-8

Q# 231

Palestine

[QUOTE who="H. Patriot"] 01. I think Israel should stop building colonies and help out the Arabs in their lands. 02. This will help it to gain favour among other ME countries. From the beginning, Palestinians have been driven out and have been homeless. 03. Moreover declaring it as a Jewish State in the middle of Arab lands has probably greatly annoyed the Arabs.

Israel has done great for itself but I feel that it has to remove the 'Jewish state' tag and become completely secular and foster friendship through help rather than arrogantly doin nothing to help Palestinians. 04. It also brags that the US is its bitch. [/QUOTE] Ans. Your post shows that you are very nave and inexperienced in these things and you do not even know the whole purpose of creating Israel and what it stands for.

You are also unaware of what Zionism means and what are its aims and goals and why Christian Zionists are More eager and in haste to help Jews in their task of Re building Israel.

Jews are Christians as their cats paws because their logic is totally different from that Christians and in the end, Jews will get all the benefits and rewards and Christians would be left holding the bag, after doing all the dirty works for jews. I will post short answers to your points, just to show how nave you are on this matter. 01. Israel should stop building colonies:

What else they should do? It is their aim to totally vacate the whole land of Palestine from Arabs and make it 100 % Jewish. They are moving slowly but surely to this agenda.

And the West opposition and rejection is only for the camera and news channels. Whe any time of action comes the bid daddy is ready with its veto to block it.

Just see the west response to that Libyan vote, within a few hours of UN vote, strikes started on Libya. How many security Council resolution are biting dirt in UN for decades without any action. 02. It will gain favor from other ME states:
242 of 277

Questions and Answers with Non Muslims Vol-8

Q# 232

Palestine

[QUOTE who=" "] [/QUOTE] Balfour Declaration of 1917

Arthur James Balfour. The Balfour Declaration of 1917 (dated 2 November 1917) was a formal statement of policy by the British government stating that

His Majesty's government view with favour the establishment in Palestine of a national home for the Jewish people, and will use their best endeavours to facilitate the achieveme of this object, it being clearly understood that nothing shall be done which may prejudice the civil and religious rights of existing non-Jewish communities in Palestine, or the right and political status enjoyed by Jews in any other country."[1]

The declaration was made in a letter from Foreign Secretary Arthur James Balfour to Baron Rothschild (Walter Rothschild, 2nd Baron Rothschild), a leader of the British Jewish community, for transmission to the Zionist Federation of Great Britain and Ireland The letter reflected the position of the British Cabinet, as agreed upon in a meeting on 31 October 1917. It further stated that the declaration is a sign of "sympathy with Jewish Zionist aspirations."

The statement was issued through the efforts of Chaim Weizmann and Nahum Sokolow, the principal Zionist leaders based in London; as they had asked for the reconstitution of Palestine as the Jewish national home, the declaration fell short of Zionist expectations

The "Balfour Declaration" was later incorporated into the Svres peace treaty with Turke and the Mandate for Palestine. The original document is kept at the British Library. The anniversary of the declaration, 2 November, is widely commemorated in Israel and among Jews in the Jewish diaspora as Balfour Day. This day is also observed as a day of mourning and protest in Arab countries.[3][4][5]

http://en.wikipedia.org/wiki/Balfour_Declaration_of_1917

243 of 277

Questions and Answers with Non Muslims Vol-8

Q# 233

Palestine

[QUOTE who="Biiguy"]

There were ALWAYS Jews living in the land of Israel, even if the numbers were small. And The Jews Never gave up the hope of returning... Just take a look at their prayers.... As to Muslims, Jews & Christians living in 'peaceful coexistence' under th muslims, only partly true, & only when non-muslims were willing to be 3rd-class citizens. the 'permanent source of hate & violence' is from the muslims, not the Jews, inasmuc as wherever muslims go there is hate & violence.. look at India, Europe today, even Thailand.... [/QUOTE] Ans. Jews living in small numbers in Palestine is OK, and they were living peacefully....they might have any dreams or hopes of clearing their lands from outsiders....we have no problem with that either....many Muslims also dream of ruling all over the world. But when it comes to ground realities, the Jews who left their lands or homes (or were forced by their enemies) and living outside for 2000 years have no "automatic" right come back and displace persons living there for past 1300 years.

This idea is very absurd in the first place and this logic has NEVER been used for any other people or nation of the world. Why should Jews be the only people for whom a special law be made?

And when every one knows what problem it would create in near future and long term ....there will never be peace in the land, because the Arabs so displaced will always b fighting for their homes and lands.

The ONLY solution was to "win over" and make permanent peace with these people and do that Jews had to bent and treat those Arabs as their equals....but to treat any one as their equals is not in the blood of Jews, so they wanted to solve this problem by the use of force and coercion and bullying them into submission.

What we have seen for past 62 years is the running commentary of these jewish tactics. I wish they were successful....but my reading of world history is that every nation and or regime based on oppression, tyranny, mischief and injustice has but a short leash o life. And the present state of Israel has all these ingredients built into its very foundation and existence. It is to be seen, whether they will be able to prove the history wrong and its principles as wrong. It is an interesting fight for survival.

On one hand there is Israel armed with latest weapons and technology and having direct /
244 of 277

Questions and Answers with Non Muslims Vol-8

Q# 234

Tanakh

[QUOTE who="Frijoles"] Ha Shem means "The Name" in Hebrew. Thank you for having no objection as to what we Jews call our God. This warms me no end.

Shhh ..don't turn everyone else on to our secret. Next thing we know everyone will wa our recipe for Gefilte fish too!!! [/QUOTE] Ans. Thanks for the info. Ha Shem stands for The Name as Allah stands for The Ilah or The god or God in Arabic.

If the word Ha Shem is used in Hebrew Scriptures for God, then it obviously would have beautiful meaning, that is sure.

Even if any one gets your secret recipe of Gefilte Fish, what use it will be to him? He wil still be a Gentile and Ha Shem of Jews will have nothing to do with a Gentile!! And in USA you could take him to court for being an anti Semite!!

245 of 277

Questions and Answers with Non Muslims Vol-8

Q# 235

USA

[QUOTE who="Jeanie"]<quoted text> I love it !!! AMEN !!! I love and pray for peace of your country Israel !!!! [/QUOTE] Ans.

With Jews vowing to be "friends and Protectors of Mighty USA"....why should they look for any other Enemy? After all what did USA GAINED in past 10 years after 9/11? I searched and I searched and I could only see and count their losses, there was no gain whatsoever. Want to have a tour of their losses ? a. They LOST four fully functional Commercial Jet airliners on the eve of 9/11 with so many persons and trained crew on board.

b. They LOST Three WTC Towers, WTC-1 and WTC-2 and WTC-7 (which collapsed ou of "shame"; how it could stand when its elder "brothers" had fallen to the ground). c. They LOST ~ 3000 humans of all religions (very few Jews of course) in those WTC towers.

d. They LOST close to 300 Fully trained Army personnel in Pentagon and good part of th "safest structure" in the world (a slap in the face really). e. They LOST close to 4500 soldiers and injured 25000 soldiers (not counting civilian casualties) in Iraq war. f. They LOST close to 2000 soldiers and injured 50000 soldiers in 10 years of warfare in Afghanistan and the toll is always increasing, and no end in sight.

g. They LOST trillions of dollars in doing this war and ruined their economy and econom of so many countries in Europe with still no light under the tunnel.

h. They LOST their status as leaders of Free nation by attacking Iraq on false ground and earned the hatred of most people on the earth.

All this is on the lost side, if you look thru Microscope, you could find a few Plus points:

A. They Got rid of Saddam Hussain and made Iraq a fully "Democratic" nation. (please d not ask what Democracy means in Iraq). B. They toppled Taliban Regime in Afghanistan and installed a "democratically elected"
246 of 277

Questions and Answers with Non Muslims Vol-8

Chapter-10 Miscellaneous Topics

247 of 277

Questions and Answers with Non Muslims Vol-8

Q# 236

Animal Rights

[QUOTE who="EDL"] The problem is Hinduism, Animism and religions of Native Americans are very difficult for common people to understand. It has very similar beliefs.

Currently, it's well understood by ecology is that if you disturb something in nature everything has irreversible consequence. Snake was very important because it kills th rodents in the farms. Thus are the small animals which are feeders for larger animals Ancient Indian thinkers didn't want to explain this but made it at a religious belief because Hinduism used to be philosophies of all Indian people before it was coined a religion. Same native Americans considered them as spirit. Our ancestors were much smarter than us they might have learned it from being practical, being with nature an their self-observation.

We shouldn't be killing this animals but try to co-exist. Now, people are urbanizing t forest and killing animals, we should know that the land we try to "steal" is originally their. So g*d, spirit or not, we need to pay them respect. Give them alternate space. [/QUOTE] Ans.

I am sympathetic to what you say, and this is precisely what happens, when men decide o their own thinking about matters they know nothing about.

It is where we need divine guidance and it was to correct all that misconceptions that God sent His prophets and messengers and books to all people and to all nations. Animals and every thing in this world is created for human beings, they are a gift of God and they should be treated as such. To be used for humans benefits and not to be taken as an article of worship. A man bowing down before a cow, snake, elephant or Monkey is really degrading his position as Best of Creations. Men are created to bow down and humble themselves before Their Creator and Lord of This Universe and not before any other creature, real or imaginary.

That is the message of islam, which was preached by all prophets of God to All nations a in all ages.

248 of 277

Questions and Answers with Non Muslims Vol-8

Q# 237

Atheism

[QUOTE who="WN"] 01. You forget that I am not persuaded by ANY holy book, yours or anyone else's. I tell you my point of view from OUTSIDE your religion and you get angry that I don't agree with you.

02. I am not angry at you personally, MUQ. I am upset that you and others of all religion need the crutch.

03. I hope there will come a day when all people realize that the divine, just like that whic is not divine, lives inside us, not outside us. 04. This negates the need for that crutch if we are able to take the responsibility of trying right the wrongs around us. It's just that simple. [/QUOTE] Ans. 01. If you are not guided by ANY book, then it is your problem and is a very wrong approach too.

In Quran we are advised to Ask people who know, if you do not know anything and thi is a very prudent advice too. When you do not know anything yourself , you consult specialist. If you think that what is purpose of your life and what will happen to you when you die, are trivial matters, then it is your view. You have no knowledge and what stand you take based on your own conjecture, so I cannot call it a scientific or reasonable approach.

02. I do not care, if you are angry with me or not, on these threads , we leave most of our emotions outside, otherwise we cannot stand one day.

What you call as crutch is not really crutch. It is the aid which you need, because you d not have knowledge about these issues.

03. That Divine which lives inside us, in no Divine, it is only our Soul, which resides inside our bodies and it is this Soul which is on test in this life. And it is this soul which will be questioned on the last day and the day we die about what it did while living on thi earth. 04. You will always be need of crutches if you want to reach where you cannot reach.

You make your own crutches of flimsy material, which break down when you put any weight on it (just like a spiders web)and our crutches are made of sold materials that ca bear our weight and weight of any one who leans on it for support (URWATHUS WUSQ
249 of 277

Questions and Answers with Non Muslims Vol-8

Q# 238

Civil

[QUOTE who=" "] [/QUOTE] Ans.

If we learn how to spell correctly, then I think it is a very plus point for us, in the fag end of our lives. We are that much ahead in the game.

But a falsehood shall always remain a falsehood irrespective of its correct spelling and be presentation of style and poise.

And truth shall always remain truth, even if it is misspelled or not presented in so elegant language.

If I speak in uncouth language then let me say that a "Prostitute shall not turn into queen, because some one has placed crown on her head". PS: Please Sir, Please check the spelling in the above post and tell me where to correct it!!

250 of 277

Questions and Answers with Non Muslims Vol-8

Q# 239

Death

[QUOTE who="Lilya"] 1. Death is change. Material returns to material with nothing vanishing. Soul returns to where it originated from and it too stays intact. Spirit (life) is recyclable and is used over and over in life - no matter where or in what.

2. The Bible states that God is the only one who can destroy the soul forever. Ancien beliefs stated the same. The 'spirit of life' doesn't die and kisses everything in life.

3. I believe when we depart from our earthly clothes we go to a place of refreshing to remove all the evidence of a material existence. Then we go thru the door to whence we came. Maybe these excursions are to teach the soul lessons. Knowledge is limitle so maybe that is the purpose of a soul.

4. The Bible talks about a void you cross over and you no longer have any knowledg of things past, not knowing relatives and friends, nor knowledge of wives, husbands, children, no marriages, no death, no wants, no bad events. Yet in Revelations it clear states and shows that those who were slaughtered for God's sake (Jews only residing under the throne of God) are looking down on earth with God seeing all the wickedness of men and they whine to God about when He will revenge their death an He said soon. These Jewish individuals could talk, felt anger and demanded revenge on those wicked people. 5. Yet we are told you remember nothing of the past. So many lies but they don't distract from the fact that life is eternal.

6. This is my belief. I'm not in the business of wanting converts or change what other believe. We will reach that same place no matter what we believe. Then truth will really be set free. I am waiting for the journey to return home. [/QUOTE] Ans.

1. Death is a change, the material body disintegrates but you are wrong that soul is als recycled. If soul also vanishes, then there is no purpose of our life on this earth. It does not matter whether we do good or bad.death will level every thing this is your own reasoning and thinking.

2. Yes God is Creator of Soul and He can destroy it also. But by the common death, He preserves the soul for giving its account of deeds on a Final day of Account Taking

3. This could be you own assumption and about these things we do not know any thing, w should believe what the Creator of Soul and Body has told us.our own assumption and mere wishes. Could be true or could be false. The soul does remember of wha it did in its earthly life and is either happy or sad for the coming day of Judgement.
251 of 277

Questions and Answers with Non Muslims Vol-8

Q# 240

Death

[QUOTE who="Chuckey"]

You just explain it in you Quote NEAR DEATH EXPERIENCE not death near death and can remember at about third grade I nearly drown and my past flashed before my eye I am now sixty two years the reason I say this is, people are dying now that have never died before, hypothetical no one that have been declared dead for years have came back and told us anything about death

and yes I believe also that the soul do not die and Beatrice do you believe there is A God after getting you life back? and I truly believe we all on this earth will loose our life whether reincarnation or not at A specific time we all shall die. and that is my point!! [/QUOTE] Ans. It was very interesting for you to talk about near death experience. There are many people who were declared dead by doctors and yet came back to life after many hours.

Experiences of many of such people have been published and could be found on net if yo search. People close to death in accidents and violent form of deaths have also reported their feelings.

But you are right, no one has come aback from the other side and told us what happene to him. That veil of secrecy has never been broken. Though people see those who are dead in dreams. But any thing you see in dream is subjective and very unreliable. You are right, soul does not die. And indeed there is God, who created every thing in this Universe

And yes, we are on this life undergoing a test.. our life time is the period given for this test. And when we die, all of us will be resurrected, brought back to life and give our account to our own Creator. Any one who passes that test is the ultimate winner. Not the one who gains wealth, power, and fame in this world. This is the Islamic point of life and death and personal accountability.

252 of 277

Questions and Answers with Non Muslims Vol-8

Q# 241

Equality

Is My Wife My Equal?

(This question was asked to me in my personal capacity on another thread , so the answer is also in the same sense and should not be used in any serious or theological discussions), I would like to share it with people on other threads also Or Am I Equal to my wife? I wish that there was some way, in which I would be able to convince my wife to accept me as her equal.!!

Whenever I talk to her about this equality issue and ask her to share some of my load she says, This is all a western propaganda , they want to see that we women be permanently enslaved by men.

When I look at it, I see this is so unfair, every thing is tilted in her favor, there is no justic and there is no equality!! Let us analyze this graded inequality in my case, which should be same as millions of people like me. A. Physical Outlook:

The problem starts from our looks itself. And I think I should lay some blame to God also as to How he did make us this way. Any one looking at me for the first time thinks it is enough for his life time.

While in the case of my wife it is reverse!! People would try to look again and again at m wife, even when she is wearing Islamic Hijab!! It is so unfair, can any one suggest a remedy? B. Earning Bread for Family: Here also I see, total inequality. I have to get up early in the morning and after prayers, make myself ready for going to the job. How hard and how painful is go to work in winter months and during rainy season, people can realize. But my wife is snuggly in her bed, smiling at my problem and commenting from time to time Hurry up, you will again be late today and when I am ready to go says, Please close the door gently and put on the coffee machine!!

253 of 277

Questions and Answers with Non Muslims Vol-8

Q# 242

Evolution

[QUOTE who="Whoops"] 01. You are a rare type of muslim since you did respond properly and honestly to my enquiry within my post. I do appreciate that.

02. However, your perceived idea of belief in evolution as being a "problem" is misplace

I do believe in God, but not religions. Then I believe GOD created all physical matters an physical laws whereby evolution occurs. 03. Evolution is no longer a theory but Genesis has now been proven to be a story.

04. I have met many individuals who are far more moral in their lives (belief and practice than many "believers of faith/s". [/QUOTE] Ans. 01. Type of Muslim: No I am not a rare type of Muslim. There are more than a Billion out there. Islam asks every Muslim to speak truth, be polite and ready to accept his or her mistakes when pointed out. May be you had very little interactions with Muslims and got biased from media reports and negative hype which is created by media against Islam and Muslims. 02. Evolution and religion:

This is a very big topic you have opened and I doubt it can be finished in one or two post so let me just give you a few outlines.

a. You said that you believe in God and that He Created all physical matters and physical laws, that is very commendable and should I say generous of you) and God will fe greatly Honored by this).

b. Then you say that you do not believe in religion. What are religions but a set of social laws from the same God who created those Physical laws. Why is it so illogical from God, not to provide you guidance about your purpose of life and why you are on this eart But let us leave that topic right now.

c. Then you said that Evolution occurs according to the laws which are made by God. Th is exactly what we call as Creation!. Things happen and take course as God Wills and Wishes it. There are set laws for it.

254 of 277

Questions and Answers with Non Muslims Vol-8

Q# 243

Evolution

[QUOTE who="DAKO"]

01. Fact: We humans are of the Ape Family. Those who lie may be required to answer to the real GOD for this lie that humans have not evolved from apes. Those short of lies do have many adverse effects on human societies. O! MUQ!

02. Cooking food was a breakthrough which advanced the evolution in a particular direction. Perhaps the first cooked food was accidental one as natural foods did get cooke in the natural forest fires. [/QUOTE] Ans.

01. If it is FACT that humans evolved from the apes, then why this FACT was not known to our earliest forefathers? It is no FACT but people imagination and nothing else.

02. And if one looks at the way man prepares food and animals prepare it, would prove th they are totally different from each other. If both man and ape saw the effect of fire on foods from that accidental fire, why no ape took any action on it and EVERY Human learned from it. The apes and other animals are observing jungle fires (both natural and artificial) for millions of years and yet, not a single one has tried to use it to prepare the food the way humans do.

That is why Quran asks us to open our eyes and look around us and we see plenty of proo everywhere to guide us. But people act like blind and do not use their power of seeing an analyzing.

255 of 277

Questions and Answers with Non Muslims Vol-8

Q# 244

Family Planning

A. Chinas Progress versus Compulsory Family Planning:

1. Now an expert on International affairs has claimed that Chinas progress is the result of their Compulsory Family Planning.

2. This is far from true, China was led on the path of progress when its leaders diluted the controls of Communism and allowed people to own private property and gave them freedom in trade and business.

3. The Compulsory family planning was introduced as an imitation of Western Powers in fact shall spell doom for China in the long run.

4. They are already seeing its ill effect as number of Girls in China is going down and the are re-thinking of abolishing One Child Policy. 5. The real reason of Chinas progress is their hard work, less corruption and their huge manpower, whom they employ in the manufacturing processes. 6. Unless you have a strong manpower base, you cannot succeed in manufacturing.

7. That is the success story of China, they went for manufacturing and India on the other hand went for easy route of becoming giants in Software.where you do not have to produce anything. Just sit in front of your computer and write programs. A thing which suites most of educated Indians.

8. In India human population is not seen as a Resource, but as a burden. Most of India poor people are without Jobs and even when they work, they are poorly paid, most o the money goes to the middle men and the poor remains poor.

And our economic thinkers think that just by forced and compulsory family planning all its ills will be solved.

B. It is ONLY Muslims who are holding back India from Compulsory Family Planning: 1. And another WRONG impression is given that it is only Muslims who are against Compulsory Family planning. 2. Do they not look at their own politicians and functionaries and see how many of them have 3 or more than 3 children? 3. And what stops 90 % of Indians to follow family planning, even if 10 % Muslims are opposed to it? Why do we need a law to force family planning?

256 of 277

Questions and Answers with Non Muslims Vol-8

Q# 245

Gender Equality

[QUOTE who="Dragnet"]

It is a well documented fact that wherever the females are empowered, the rate of population growth is very healthy. Females who are socially and educationally empowered and have relative economic freedom tend to have lower children than the ones who are not. [/QUOTE] Ans. Family Planning and Women Empowerment: Now people are giving their own theories based on their own logics than based on hard facts and proofs.

The main point is that how many children a couple needs or wants is their own matter and state or society has no business there. This is the most natural solution and that is ho it should rest.

What are the problems with Ambanis and Modis and Tatas and other wealthy business houses to raise and produce as many children as they want. They can educate them a they can take care of them and there is no restriction of any kind on them

But we see that these people are the first to be affected by this We do not need more children virus. They do not want to have more children, when there are no restrictions on them from any side.. and the same is the case with High and upper Middle class and lower Middle classes of the society. These are people that provid much of the IQ content of the society.

When these people produce less than 2.5 Children per family, the IQ content of the societ starts decreasing and the society starts loosing its power of innovation and trying to explore new fields and the result being that it starts loosing its position in the world. The poor and poorest of the poor members of the society needs more children, because they do not have to spend much on their food and education and they add to the economics generating members of the society. That is why their numbers and percentage starts increasing in the society and the IQ level of the society goes down further.

There should be no restriction on women getting education and knowledge of any kind, b if the aim of that education is to earn money and compete and fight with men in each and every field, that shall destabilize further the bond of family which is so much needed for a happy and satisfied family and household.

You just see and check the condition of children of those families where both husband an
257 of 277

Questions and Answers with Non Muslims Vol-8

Q# 246

God

[QUOTE who="LWS"] 1. I don't see how any of the interpretations you make re. Islam/Mohamed in the ..

2. AND, MUQ, it seems to me that the supreme being, creator of the entire universe shou be far beyond any human perception or concept of having a "personality". God of the OT pretty much described Himself as "the One who Is"...Not something so trivial as a "personality". 3. But the Jews never were "replaced". Not all Jews left their homeland. Many did, .. .

04. Islam is falling on its own bloody sword, MUQ. The question for the future of Islam i for Muslims to answer. Muslims have to figure out a way to be Muslims without butchering each-other and without alienating the "infidel" world. .. [/QUOTE] Ans. 01. Prophesey and Specific personalities:

a. I had mentioned earlier that prophesies are word pictures of a future event that might happen many centuries after the prophesies are made. b. They are never specific and never name person and or persons who would fit that prophesey. c. Scholars are on the look out to try and fix prophesies to particular persons and events that bear close resemblance to the wordings of prophesy.

d. But if some one thinks that Another Powerful Prophet will come into the world and set things right, they are mistaken and that door is closed for ever now. 02. God and personality:

a. I never claimed that humans can ever encompass God and understand Him fully, God and His attributes and His power and His Wisdom is beyond the limits of understanding o humans. b. I used the term Persons and Personality as used by Christians to make God into a Triune God with three Distinct Persons. 03 Jews Replacement:
258 of 277

Questions and Answers with Non Muslims Vol-8

Q# 247

Hoax

[QUOTE who=" "] [/QUOTE] Ans. An "official Communiqu" from the Office of Son of Man: Dearly beloved,

I wanted to keep my "revisit plan" hidden from people, so that it should be a surprise to all....that is why I did not tell it to any one, but hide it in code words in "my book"....and I also told very clearly the following: "Matt. 24: 37 But as the days of Noah were, so shall also the coming of the Son of man be. 38 For as in the days that were before the flood they were eating and drinking, marrying and giving in marriage, until the day that Noah entered into the ark, 39 And knew not until the flood came, and took them all away; so shall also the coming of the Son of man be. "

Only ONE MAN found out (after many false starts and hit and trials) and, decoded m cipher, WHEN is the Coming of Son of? And THAT man is Sir Harold Camping, Esq. Reverend of Church of Lay Preachers.

And now THAT man has betrayed this Secret Knowledge to a bunch of his dedicated followers....so that the "return plan" of 'Son a Man" is no longer a Secret and Mystery any more!!

And therefore I have changed my plan and will no longer return on May 21, 2011. M return date will therefore will continue to be a secret (as originally planned) and would not be announced any more. All data s that exists in the existing "official book" stands obsolete and ineffective, with immediate effect. Yours Lordly, Signed: H.H.H.H.E. The Son of Man (Official Seal of Son of Man)
259 of 277

Questions and Answers with Non Muslims Vol-8

Q# 248

Hypocrisy

[QUOTE who="LWS"]

But MUQ...YOU "stick to prove your own beliefs, right or wrong by any way, fair or foul..." You don't use foul language, which I admire, but you accuse me of doing the same thing you have been doing for these three years. THAT is what's known as "hypocrisy"...Or do you believe that it isn't hypocrisy if a muslim is being hypocritical? [/QUOTE] Ans. Let me explain, there is a FUNDAMENTAL difference in your arguments and my arguments. 1. In my arguments, first I establish that I hold the Book of Allah, i.e. Quran and authenticated sayings of our Prophet as absolute truth. 2. This is not blind faith, because there are basic proofs which support that Quran is a revealed book of Allah which has reached to us in the same form as it was revealed and that our Prophet Mohammad (PBUH) is a true prophet of Allah.

3. If some one is interested in arguing me about these two issues, I am always ready to do it, provided you do it in civil and proper manner and listening to your opponent.

4. Once I have agreed that Quran is a book of Allah and our prophet is a True Messenger of Allah, it would indeed be hypocritical for me, to reject any of their express saying 5. That is why I always hold the Basic principles of Islam as true and logical and reasonable and practical..

6. Except Quran and established principles of Islam, I do not blindly defend actions of an Muslim person, or nation or group. This you can see in my various posts. 7. Your approach on the other hand is based on YOUR OWN VIEWS and without any authority or accuracy of your sources. That is why you speak sometimes in contradictory terms and use foul language and / or abuses to silence your opponent some how, and you think it as your victory.

8. You forget that truth has a power of its own and every logic, or reason or principle howsoever good it may look to us is not truth unless it is confirmed as truth by analyzing it with the absolute standard of truth, Quran and Prophets sayings!!

260 of 277

Questions and Answers with Non Muslims Vol-8

Q# 249

India Partition

[QUOTE who=" "] [/QUOTE] Ans. The partition of India in 1947 was not a solution for Whole of India. It was only for the Muslim majority states in the Western Frontier and Bengal in the east. There was a very strong group of Indians which was led by Maulana Azad, Rafi Anmad Kidwai, Ulama of Deoband, Nadwa etc, who were apposed to this Partition plan.

It was never a solution that All Muslims should go to Pakistan and all Hindus should go to India, otherwise the division of India should have been 30 % for Muslims and 7 % for Hindus (as per their population in 1947). So it is very wrong to say that Muslims "lost the right to live in India after 1947".

There was no one to Give any right to Muslims, they were and are as part of India as any Hindu, Sikh, Christian, Parsee or Buddhist. India is not the "personal property" of any religious group. PS: The people who are really responsible for Partition are the right wing and extremist Hindus, who drove that "secular and activist for Hindu Muslim Unity" Jinnah to demand separate land for Muslim majority state.

Almost any one who has written a history of partition of India, puts more blame on Nehru Patel and other right wing extremist Hindu organizations for partition of India.

Read ANY book, except those emotional ones written by RSS people, if you want to kno the truth of partition.

261 of 277

Questions and Answers with Non Muslims Vol-8

Q# 250

memoirs

On Great Hakims and Not So Great Hakims A lot can be said on this topic, but nothing is better than ones own testimony, so I shall relate something from my own experience. A. Great Hakim, the Late Hakim Abdul Hameed, Patron of Hamdard Labs.

1. It was in year 1975, when I, a young fresh graduate in Electronics and Telecom Engg. Joined BEL Ghaziabad, a well known Govt. of India Undertaking dealing in Telecom and Radar systems for Ministry of Defence and Police forces. 2. From my college days I had sensitive Tonsils and any change of weather etc. would cause swelling and pain in the neck and coughing etc. But it was always minor and I did not had to go to a doctor for treatment. 3. But it so happened that in 1975, suddenly my Tonsils gave me a lot of pain and I was unable even to speak. I had to take emergency leave from my Job. I was new to Ghaziabad and in those days it was a small city, it was only recently made a Dist. 4. I went in search of an ENT specialist and found one Young Doctor who had very recently opened his clinic in the city.

He told me that my tonsils are filled with puss and they have to be cleaned. He removed a lot of puss and I got immediate relief from the pain and he charged Rs. 125 (not a small amount then) and gave me some medicine.

5. When I was leaving, he told me that I have an affliction known as QUINCY in Medica terms and I must get my Tonsils removed, otherwise this problem would be recurring He said that I should get the operation done within six months.

6. I was half convinced with the doctors advice (after seeing how serious it had become) when some one told me that Hakim Abdul Hamid sees the patients in his Hamdard Head office located on Asif Ali Road New Delhi, very close to Ram Lila ground.

Why should not I get a second opinion?. They also told me that I must reach there before AM, otherwise I would not get a chance to get my turn. 7. So next day I started very early in the morning from Ghaziabad and reached his clinic before 7 AM, I found the queue was already big and they had put benches in snake form to accommodate as many people as possible in the waiting room. 8. Promptly at 9 AM, Hakim Sb. Walked in and the serpentine queue started moving slowly, untill it was my turn to see Hakim Sb. And put my case.

9. I told Hakim Sb. About my infliction and recent operation and the advice of Doctor to get my tonsils removed, I told him that I have some to him for his advice. He said Inshallah (i.e. God willing, a phrase which every Pious Muslims says when speakin
262 of 277

Questions and Answers with Non Muslims Vol-8

Q# 251

Miracle

[QUOTE who="Pete-O"]

01. A miracle is an event created by religious people to explain what they don't understan about the natural world. Hopefully you can understand this definition. Here's another: 02. A highly improbable or extraordinary event, development, or accomplishment.

[/QUOTE] Ans.

As usual your first definition is too defensive and totally meaningless, so there is nothin to comment about it.

You have opened up a little with your second definition. It is also not 100 % correct, bu we can call it a Lay mans definition, happening of something Improbable, Extraordinary event, extraordinary development, extraordinary accomplishment This could be starting point of our discussions. Now I will give you an example (hypothetical of course, do not be alarmed, I am not pulling your leg) In your opinion, how much is the weight a man can lift?

Let me help you out, most people can lift 100-150 KG weight, but trained professionals can lift 400-450 Kg and Worlds strongest man Mark Henry had lifted 500 Kg s

So I think 1000 KG should be the fair limit (during our life time) that a man can ever lift.

Now suppose a man right in front of you, lifts a rock which weighs more than 100,000 K and puts in another place, would you call that a Miracle? It is a simple question, I am not pulling any trick, so do not be defensive and just answer simple yes or no.

263 of 277

Questions and Answers with Non Muslims Vol-8

Q# 252

Modern World

[QUOTE who=" "] [/QUOTE] Ans.

Is only one party to be blamed? What was the root cause of the problem? I do not know Any one even touched that? Pruning leaves and branches and leaving the roots untouched?- MUQ Man had sex with drunk 12-year-old girl By TONY BLAIS, QMI Agency http://cnews.canoe.ca/CNEWS/Crime/2011/04/22/...

EDMONTON - An Edmonton man who had sex with an intoxicated 12-year-old girl and then let her leave in the middle of a winter night with no shoes or jacket on has "no remorse."

That's what a city judge said Thursday while sentencing Gerlin Romero-Moncada, 21 to four years in prison. "The accused did not care for her," said provincial court Judge Ferne LeReverend. "She was vulnerable, not only by her age, but from her consumption of alcohol and marijuana, and the accused took advantage of her." LeReverend also noted that Romero-Moncada said, "I don't care about this f---ing case" while being cross-examined after testifying in his own defence at trial.

Romero-Moncada was found guilty of sexual assault and sexual interference on Apri 8.

Court has heard the 12-year-old girl went with two girlfriends to a party at RomeroMoncada's north Edmonton home on Dec. 30, 2008, and she consumed pot and booz

She testified that, after her friends left and she was alone at the party with a number o males, she had sex with one guy before later having sex with Romero-Moncada.

Court heard some other females showed up at the party and told the girl to leave and she went out without any shoes or jacket on. A security guard at the building saw her and called police and Romero-Moncada was arrested.

Romero-Moncada denied having sex with the girl and testified he thought at the time that she was 17.

264 of 277

Questions and Answers with Non Muslims Vol-8

Q# 253

Movies Top 25 Ground Breaking Movies (of Modern times) http://technology.canoe.ca/Microgalleries/MovieBreakthroughs/home.html?&pic=24 01. Movie: Tron Year: 1982

Breakthrough: Famous for its high-speed Lightcycle race, Tron pioneered the use o CGI in live-action film. While not the first to use the technology, it was the first to us it extensively. However, Tron gets more credit than may be deserved, as only about 2 minutes of the movie used CGI. Does it stand the test of time? No way, it NEVER impressed us. 02. Movie: Who framed Roger Rabbit? Year: 1988

Breakthrough: Live actors and animated "Toons" freely interact in this tribute to all of t great cartoons of the 1940s. Artists added hand-painted images to the film, and used sophisticated shading techniques to make the lighting on the cartoons mirror the real actors. Does it stand the test of time? Yes. It looks great, even by today's standards. 03. Movie: The Abyss Year: 1989

Breakthrough: While not the first to use "morphing" in a major film (1984's The Golden Child owns that record), The Abyss perfected the techique with its "pseudopod" water tentacle that mirrored Mary Elizabeth Mastrantonio's face. This "character" was the first t be in 3D CGI. Does it stand the test of time? Looked good then, but now... not so much. (Especially those damn aliens!) 04. Movie: Back to the Future: Part II Year: 1989

Breakthrough: This film was the first to have interacting characters played by the same actor. Michael J. Fox plays three roles in the famous "instant pizza" dinner scene, thanks a computer-controlled camera system called "VistaGlide" that allows for a moving boundary between sections of a split screen. Does it stand the test of time? Yes. It looks great, even by today's standards. 05. Movie: Terminator 2: Judgment Day Year: 1991

Breakthrough: Robert Patrick's liquid-metal T-1000 terminator has the ability to turn int
265 of 277

Questions and Answers with Non Muslims Vol-8

Q# 254

Pakistan

[QUOTE who=" "] Pakistan is not doing enough [/QUOTE] Ans.

What Pakistani President said was very correct that Most people in US Govt. and Congre do not know what they are talking when it comes to other countries and civilizations They do not know what are the ground situations are in India and they keep on repeating like a parrot "Pakistan is not doing enough". "Pakistan has not done enough".

I think what Pakistan has done, no other country can do. It has surrendered its sovereignty to US and NATO forces. They come with ease and bomb any place they want and no questions asked. What happened in Davis case, shows the extreme haughtiness os US people towards an independent country. A US secret service man, shoots two Pakistani nationals in broad day light.....and US "orders" Pakistani Court and Govt to "release" their man.

Can any Pakistani embassy man doing the same "crime" in US would be dealt in the sam way. This dual standard of Justice will be the failings of US and European people in the end. How can they distinguish on the basis of color of skin and nationality.

US and European people use a very shallow approach when dealing with other people an that is the main reason why there is so much mistrust and misunderstanding in their relations with other nations.

But when the issue any "orders" to Israel....it is "thrown back" on their faces with contem and they remain a moot spectator and can do nothing.

This is another anomaly in their relationships with other nations, they have a separate law to deal with Israel....much lenient than their own laws!!

266 of 277

Questions and Answers with Non Muslims Vol-8

Q# 255

Religions

[QUOTE who="Mt. Sinai"] 01. Our Group from Mt. Sinai are a small group of Free Thinkers and have come to this conclusion of what's right and what's wrong in this Mad Crazy World of Religious Extremists and Fanatical Radicals. 02. That mostly all religions think that they have the definitive religion, based on some charlatans theory and interpretation of the Truth.

03. We've never interpreted the Old Testament prophecies of the coming of Jesus Christ i the Old Testament, 04. Just look around u and see all the problems religions have caused in the past and the problems it will be causing in the future.

05. Free Thinkers are independent Spirits, that are dedicated to the One True Creator of A Creations and we do not serve any religion,

06. Jesus Christ was just a very Special Man, who set the fine Example for All to Follow, for the undeserved Gift of Everlasting Life.. 07. Now to digress for just a second and wed like everyone to listen up to what we are about to post very closely,..... 08. Neither was Moses, Abraham, Noah and all the Hebrews and Israelites in Genesis.. And that's real sad for them, when their pseudo minds are denying themselves from accurate We agree, nothing could be fairer, but to wait for Judgement Day. Peace! [/QUOTE] Ans. You have posted a long post and raised many points, I would try to cover them in my answer: A. Free Thinkers: The name you chose, is itself wrong. Free Thinkers are like free electrons, they would sway from one extreme to another extreme, when the force is applied to them.

Any thinking without having laying down foundations would lead you into error, as I can
267 of 277

Questions and Answers with Non Muslims Vol-8

Q# 256

Saudi Arab

[QUOTE who="RL"] Muq, Why are Saudis so racist, Saudi women are prohibited from marrying non-Arabs except with a special dispensation from the King. Something wrong with Muslim Indian men?.

[/QUOTE] Ans.

First of all I am an Indian Muslim working in Saudi Arabia, so if you are mistaking me fo being a Saudi, you should correct your mistake. Second thing is that what Saudis or Saudi Govt. does has no de-facto authority in Islam.

Islam is not what Saudis or Saudi Govt. does, but what Quran and Prophet of Allah said o did. In Islam there is no distinction between one human being and another and there is no limitation on one women marrying any Muslim man. But there are local customs and local practices. Saudi Arabia is a tribal society for many, many centuries. And in tribal society marriage outside their clans is not looked with favour.

And now, since the discovery of Oil, whole world is flocking to Saudi Arabia and getting too much interest in Saudi women (whom they would ignore, if there was no oil an wealth), so they fear about their local customs and their existence itself. That is why they have made all these rules You can see there is how much noise in western countries about growing influence of Islam and Islamization of Europe and USA You can say how much will be the fear of Saudi Arabian people if they allow free marriage But this is a local law and not any direction from islam.

268 of 277

Questions and Answers with Non Muslims Vol-8

Q# 257

Saudi Indian

[QUOTE who=" "] [/QUOTE] Ans. Unite to change social order: Al-Sudais By ARAB NEWS Published: Mar 27, 2011 23:19

NEW DELHI: Sheikh Abdul Rahman Al-Sudais, imam of the Grand Mosque in Makkah, laid the foundation stone for the Students' Islamic Center in New Delhi Sunday and urged the student community to work together to change the social ord and bring about progress and prosperity.

The imam commended the Students Islamic Organization of India (SIO), which is buildin the center, for its efforts in spreading the message and culture of Islam among the student community. The SIO has branches in most Indian states as well as in GCC countries.

I am very happy to learn that the SIO is playing an important role in the Indian society, the imam said. The student community is a vital source of development and progress for any country, he said. He wished every success for the student center project. Earlier, SIO President Muhammad Azharuddin welcomed the imam and introduced his organization. We are greatly honored to have Your Excellency to launch this Islamic center project, he said. The SIO works for preparing students and youth for the reconstruction of society in the light of divine guidance, Azharuddin said. The SIO believes that the real function of education should be to impart genuine knowledge about life, existence and future, he added.

The ceremony at the SIO headquarters was attended by President of Indian Jamaat-e-Islam Syed Jalaluddin Umari, the Saudi Ministry of Islamic Affairs Abdul Aziz Swaleh, Jamaa vice presidents Siddique Hassan and Muhammad Jaafar, SIOs secretary-general P.M. Saleh and other dignitaries.

269 of 277

Questions and Answers with Non Muslims Vol-8

Q# 258

Scholar [QUOTE who="scuter pie"]i'm still hung up on the fact that someone is under the assumption that camping is a scholar of anything much less of the bible! my cat, who sleeps on a bible from time to time, is a better scholar![/QUOTE] Ans. It is mostly the "Scholars" that swerve out of the way and mislead themselves and thousands of their followers. Thru out the religious history of every religion, we find 'Scholars" who divide into sects and create divisions into the religious community.

And most of these "scholars" are who "ponder too much" on scripture and try to find hidden meanings and decipher clues which were not seen or were clear to those who came before them.

The Best Scholars are those, who abide by the teachings of their earliest people, who wer direct disciples of the their prophets and those who were following their path.....these scholars will always call you to come back to scriptures and follow your prophet....these scholars are what we know are reformers. The other type of scholars lead us into "research" in unknown territories and find hidden meaning and open secret doors and predict what will happen in future and similar bizarre things. I do not know in which category Mr. Camping belongs to, can any one help me?

270 of 277

Questions and Answers with Non Muslims Vol-8

Q# 259

Secularis m

[QUOTE who="DAKO"]

Response to part of my post and a highly suspect response. Secularism is acceptance of other faith believers and also non-believers such as agnostics, atheists, etc. Secularism also means that rights of all citizens are respected as per UN Declaration of Human Rights, not according to any one belief faith. [/QUOTE] Ans. A. Secularism in Indian Context:

Secularism (please note that there is no Uniform definition of Secularism accepted by everyone) In Indian context means that state shall not discriminate between its citizen based on religion, language, color, caste or ethnicity. Government shall not have any religion of its own.

Every section of the society shall get equal opportunity in jobs and economy and educatio etc.. PS: The above does not restrict Govt. from giving some special privileges to Dalits and give protection to Minorities. This is the working definition of Secularism in Indian context. B. Anti Secular acts done by the Govt.: Govt. shall not have any religion, but you see all these BHOOMI PUJANS Coconut Breaking Lamp lighting and Chanting of Mantras at Govt. functions and no one complains that it is against Secularism. We see all those religious dramas and Astrology and Pujas and Bhajans being shown on TV and Channels and no one complains. It means when you are in majority, your religion becomes the de-facto religion of the whole country.

271 of 277

Questions and Answers with Non Muslims Vol-8

Q# 260

Sex

[QUOTE who=" WN]

I doubt that any all-knowing god would give people the urge to have sex and then punish them for it. I think people who have indiscriminate sex without protection get punished quite enough here. That's another reason I don't believe in gods. That would just be either very ignorant on his part, or very evil.

[/QUOTE] Ans.

God will not punish any one for fulfilling his or her sexual desiresbut He would punish those who do it thru illicit means (and do not repent it)!! God has given men and women legitimate means to fulfill their sexual desires. If they limit themselves to these outlets, there is no blame of any kind on them. In Islam fulfilling you sexual desire thru legitimate means is approved and appreciated. There is no Monastism in Islam . However when people exceed the legitimate means and fall into illicit means they are discouraged and punished.

Same way God has given the urge of eating and drinking to every human and He will not punish any one for fulfilling these desires.but He will punish if people do it thru wrong means, i.e. by stealing and robbing others!!

You do not believe in God, because you have been fed up with wrong concept of God by the so called various religions of the world.

If you want to know about the Real qualities of God and His attributes..then you should study Quran, where you fill find Most accurate information on this subject. God has qualities of Mercy as well as Justice with Him. He does no injustice to any one and He does not like that any one should do injustice either.

Justice at every level is the Basic Law and Basic Pillar of His Code of Conduct to human beings.

272 of 277

Questions and Answers with Non Muslims Vol-8

Q# 261

Soap

[QUOTE who=" "] [/QUOTE] Ans.

Brushing your teeth with soap


Written by: DR. GIFFORD-JONES, Special to QMI Agency

http://chealth.canoe.ca/columns.asp?columnistid=6&articleid=31590&relation_id=3224

Do you enjoy paying dental bills? Or having dentists scraping plaque from your teeth? If it's a pleasure, there's no need to read this column. But I've never enjoyed these regul checkups. Now there's a way to retire dentists, prevent cavities, protect gums and rid teeth of plaque, using cheap, ordinary soap. My first reaction when I read this report was, "Come on, Dr Judd, you must be kidding! Who would ever brush their teeth with soap?" But Dr. Gerald F. Judd is no nut. He's retired Emeritus Professor of chemistry at Purdue University.

I admire people who have the intestinal fortitude to question well-established theories tha may be wrong. Besides, I discovered he and I both believe dentists are wrong on another issue.

Dr. Judd reports that acid destroys enamel and that cavities would vanish if people rinsed acids from their mouths quickly. Tap water is all that's needed to do the job.

He also claims that bacteria cannot damage the tooth's hard outer enamel that is compose of calcium hydroxy phosphate. The proof is that bones and teeth are resistant to earth bound organisms. After all, we've all seen pictures of skeletons that have been unearthed after hundreds of years with teeth still intact. But why use soap to clean teeth? Judd says glycerine is present in all toothpastes and it's sticky that it requires 27 washes to clean it off. This means that teeth remain coated with a film and cannot rebuild enamel. And if they're not clean, adenosine diphosphatase cannot provide phosphate to enamel.

His next point is what I wanted to hear. Brushing with soap destroys bacteria and viruses. No professor at The Harvard Medical School told me about that. Or that brushing wi ordinary bar soap not only cleans teeth but also removes hard plaque stuck to the bottom of enamel.

Removing plaque from teeth is vital as it invades gums, separating them from teeth. This sets the stage for gingivitis, poorly anchored teeth and eventually possible loss of teeth. It's shocking that 25% of North Americans over age 43, and 42% of those over
273 of 277

Questions and Answers with Non Muslims Vol-8

Q# 262

Toyota

Toyota to pay $32.4M in extra fines


Dec 21, 2010 By KEN THOMAS, Associated Press

WASHINGTON Toyota Motor Corp. has agreed to pay the government a record $32.4 million in additional fines to settle an investigation into its handling of two recalls at the heart of its safety crisis.

The Transportation Department said late Monday the civil penalties will settle investigations into how Toyota dealt with recalls over accelerator pedals that could g trapped in floor mats, and steering relay rods that could break and lead to drivers losing control. The latest settlement, on top of a $16.4 million fine Toyota paid earlier in a related investigation, brings the total penalties levied on the company to $48.8 million. It caps a difficult year for the world's No. 1 automaker, which recalled more than 11 million vehicles globally since the fall of 2009 as it scrambled to protect its reputation for safety and reliability. Toyota's board of directors agreed to pay the fines on Tuesday at the company's board meeting in Japan, according to an official familiar with the case, and the company said it agreed to the penalties without admitting to any violations of U.S. laws. However, that does not free Toyota from potential civil and criminal penalties in private lawsuits and other federal investigations.

The person had spoken earlier Monday on condition of anonymity ahead of the formal announcement.

Steve St. Angelo, Toyota's chief quality officer for North America, said in a statement tha the company has "worked very hard over the past year to put these issues behind us and s a new standard of responsiveness to our customers. These agreements are an opportunity turn the page to an even more constructive relationship with NHTSA (National Highwa Traffic Safety Administration)."

He said Toyota was grateful to its customers for "their confidence in the quality and reliability of our vehicles."

In April, Toyota agreed to pay the maximum fine allowed under law for a single case
274 of 277

Questions and Answers with Non Muslims Vol-8

Q# 263

War

[QUOTE who="Arbitrageur"] Indian troops fought well against the Japanese and were highly respected.

Are you simply anti-war or were you in favor of the Nazis taking over Europe, Afric Russia, the Middle East, and Central Asia? You may just have a fondness for the Turks, Muq. I don't know. [/QUOTE] Ans. India at that time was under the control of British and HAD to fight against Japan.

Most of Indians were happy at the defeats which Germany was executing on British force

As a slave nation, Indians were craving for freedom from yokes of Britain and as enemy of your enemy is your friend. Many Indians were happy that Germany was winning.

And it was because of Germany that India won freedom from Britain so quickly after the end of WW-2.

But in no way I or any Indian (except a few of Extremely Right Wing Hindu Nationalists supported Nazi Philosophy of German Racial Superiority. Our support to Germany was only moral and no way corporal. Turkey being the Islamic nation was a different thing. Even though Indian Govt. was fighting Turkey in WW-1, Indian Muslims were against that war.

They only could support Turkey in providing humanitarian aid and which they did as mu as they could do. Being ruled by another country, does ties your hands in a big way. May be you have no experience of that, but we Indians do have that experience. Thank you for providing this diversion from our usual posts.

275 of 277

Questions and Answers with Non Muslims Vol-8

Q# 264

West

[QUOTE who="Frank"]

LOL ... may be so, but those dumb people youre so awkwardly referring to are responsib for the richest and most advanced Country in history! Chances are that you, yourself, wer educated right here in the USA! Apparently you missed joining a debate team learning how to argue effectively ... [/QUOTE] Ans. A. Wests Progress in the world: You have raised a very important point here and apparently there is lot of misunderstanding in the mind of western people on this issue.

They think that once you have marvelled in Science and Technology and Won scores of Nobel Prizes, you become Expert in every field and there is no one above you in any field But this is not true. Expertise in one field does not mean that you are experts in all field. There is no denying the fact that most of Western people are honest and hardworking people and they are very sincere in the jobs they do. It is their hardworking and sincerity which is the prime reason for them getting so much progress in science, technology and medicine. B. Perennial Weakness of Western People:

But despite all these achievements, there is one thing in which they lack and that is thinking and analysing and planning for issues that are not related to Science, technology and Industry.

In these field they have given free reign to their Politicians and their media and they simp accept what their politicians and their media tells them.

Anyone controlling media can mould and change the public opinion much more easily th possible otherwise, and that is why the control of media (newspaper, Magazine, Films, TV Channels, cartoons, Fiction books etc.) is such an important matter in the Western World

Another field thru which Western people can be controlled and molded is thru controlling their Finances, their Banks, Stock Exchange, Insurance and Mortgage firms. If any group controls BOTH media and Finance of the Western people, he can simply
276 of 277

Questions and Answers with Non Muslims Vol-8

Q# 265

India

[QUOTE who=" "] [/QUOTE] Ans. Some one said Muslims never ruled "Whole of India"!! What is whole of India and where the whole of India is defined? Under Aurangzeb, the Rule extended from Afghanistan (Kabul) , and upto Burma (Myanmar of present).

It was the biggest size of "Whole India" that India ever witnessed. Small pockets of local chiefs here and there do not count. Marathas and Sikhs during the time of Mughal Empires are like we see "Naxalites" and "PWG" in Bihar and AP.

Do you mean to say that because of these separatist movements, Indian Government is no ruling "Whole of India"? We know the situation as it is.....may be after 200 years or so some one will make "More than life size image" of these Naxalites and PWG leaders and said that "They were heroes who defeated the might of Indian Government and every one in India was terrified when they heard their names".

In reality, it is these leaders who are running away from place to place and being terrified at the sight of Indian Army. This is what I call mutilation of History and distorting it and twisting it to suit your purpose. And that is what we see in these "heroics" of Shivaji and Rana Pratap and Sikhs....minor irritants blown over many fold to look like "giants"

277 of 277

También podría gustarte